Book For Practice V2.01 Quantitative Ability

Download as pdf or txt
Download as pdf or txt
You are on page 1of 112

Table of Contents

Quantitative Aptitude - Handout ........................................................................................................................ 2


Quantitative Aptitude Handout Homework ..................................................................................................... 11
Quantitative Aptitude Handout Homework - Solutions ................................................................................... 13
Quantitative Aptitude ....................................................................................................................................... 17
Chapter 01 - Number System ....................................................................................................................... 17
Chapter 02: Percentages .............................................................................................................................. 21
Chapter – 03: Profit & Loss, Partnership ................................................................................................... 25
Chapter 04: SI & CI ..................................................................................................................................... 28
Chapter 05: Averages................................................................................................................................... 32
Chapter 06 Ratio & Proportion ................................................................................................................... 36
Chapter 07: Mixtures & Allegation ............................................................................................................. 40
Chapter 8: Time & Work.............................................................................................................................. 43
Chapter 09: Time Speed & Distance............................................................................................................ 48
Chapter 10 Pipe & Cistern........................................................................................................................... 54
Chapter 11 - Mensuration ............................................................................................................................ 57
Chapter 12 - Permutation and Combination................................................................................................ 61
Chapter 13 - Probability .............................................................................................................................. 64
Quantitative Aptitude Solutions ....................................................................................................................... 68
Chapter 01: Number System ........................................................................................................................ 68
Chapter 02: Percentages .............................................................................................................................. 72
Chapter 02: Percentages .............................................................................................................................. 76
Chapter – 3: Profit & Loss, Partnership ..................................................................................................... 78
Chapter 04: SI & CI ..................................................................................................................................... 81
Chapter 05 Averages .................................................................................................................................... 85
Chapter 06: Ratio & Proportion .................................................................................................................. 89
Chapter 07: Mixture & Allegation ............................................................................................................... 93
Chapter 08 : Time & Work........................................................................................................................... 97
Chapter 09: Speed, Time & Distance......................................................................................................... 103
Chapter 10: Pipe & Cistern ....................................................................................................................... 106
Chapter 11 - Mensuration .......................................................................................................................... 107
Chapter 12 - Permutation and Combination.............................................................................................. 109
Chapter 13 - Probability ............................................................................................................................ 110

For details on our programs: - Contact: +91 9930202329, +91, 8291924300 or visit www.campuscredentials.com (1)
d) 15:49 e) Can't be determine
Quantitative Aptitude - Handout 𝒂 𝟑 𝟐𝒂−𝟑𝒃
iii) = , Find the value of
𝒃 𝟐 𝟒𝒂𝟑 −𝒃

1-Equations, Ratio, Proportion & a) 13:47 b) 0 c) 22:47


d) 15:49 e) Can't be determine
Variation
𝟑
1) A student ask to find th of a number and he instead 9) An amount of Rs.1560 was divided among A, B and C in the
𝟕
𝟕 𝟏 𝟏 𝟏
multiplied it by th . As a result, he got an answer, which was ratio : : . Find the share of C in this amount.
𝟑 𝟐 𝟑 𝟒
more than the correct answer by 1680. What was the number? a) Rs.300 b) Rs.320 c) Rs.280
a) 882 b) 273 c) 840 d) Rs.360 e) None of these
d)1684 e) None of the these
10) The ratio of present age of A and B is 11/4. 15 years ago,
2) A and B each has some money. If A gives Rs.30 to B then B the ratio of their ages was 8/1. Five years ago, the ratio of the
𝟑
will have twice the money left with A. But if B gives Rs.10 to A, ages of B and C was . What is C's present age?
𝟐
then A will have thrice as much as is left with B. How much a) 15 years b) 10 years c) 22 years
money does B have? d) 25 years e) None of these
a) Rs.20 b) Rs.28 c) Rs.24
d) Rs.34 e) None of these 11) A varies directly with B when C is constant and inversely
with C when B is constant. Given that A is 16, when B is 28 and
3) X says to Y, "I am twice as old as you were when I was as C is 7. Find the value A, when B is 9 and C is 6.
old as you are". The sum of their present age is 63 years. Find a) 6 b) 7 c) 8
the present age of X. d) 9 e) None of these
a) 43 years b) 39 years c) 36 years
d) 42 years e) None of these 12) Two men Ashok and Baban have the ratio of their monthly
incomes as 6/5. the ratio of their monthly expenditure is 3/2.
4) A question paper consists of 50 questions. Each correct find the ratio of their monthly savings.
answer fetches 3 marks and 1 mark is deducted for each wrong a) 1:1 b) 3:5 c) 3:10
answer. A student who attempted all the questions, scored 90 d) 3:8 e) Can't be determine
marks. Find the number of questions answered by him correctly
a) 35 b) 25 c) 15 Directions for questions 13 and 14:
d) 30 e) None of these There are two colleges in the town - college A and College B.
There are 500 students more in college A than in college B. The
5) The sum of 3 single digit numbers is 15 less than their ratio of the boys to that of the girls in college A is 3:2 and that
product. If we subtract 2 from first given number then sum of in college B is 4:1. The ratio of number of Science, Humanities
these numbers will become 7 more than their product. The and Commerce student in college A and College B are
product of given 3 numbers will be? respectively 2:5:3 and 2:3:3. The number of commerce students
a) 6 b) 8 c) 12 in both the colleges is the same.
d) 24 e) 30 13) How many students are there in college A?
a) 2000 b) 2500 c) 3000
6) If a carton containing a dozen mirrors is dropped, which of d) 3500 e) None of these
the following cannot be the ratio of broken mirrors to unbroken
mirrors? 14) How many girls are there in two colleges together?
a) 2:1 b) 3:1 c) 3:2 a) 1400 b) 1600 c) 1700
d) 1:1 e) 7:5 d) 2000 e) None of these

7) The total cost of producing a certain number of units of an 15) The distance travelled by freely falling body is directly
article is partly fixed and partly varies directly with the number proportional to the square of the time for which it falls. A body
of units of that articles produced. The average cost of fell 95 m in the 10th second. Find the distance it fell in the 14 th
producing 15000 units of that article is Rs.2 while that of second.
producing 25000 units of that article is Rs.1.5. Find the a) 54m b) 116.5m c) 108m
variable cost of each unit of that article produced d) 135m e) None of these
a) Rs.1.25 b) Rs.0.75 c) Rs.1.50
d) Rs.2.50 e) None of these 16) Cost of precious stone varies directly as the square root of
its weight. A certain precious stone broke into 3 pieces whose
𝒂 𝟑 𝟓𝒂+𝒃
8) i) = , Find the value of weights are in the ratio 1:4:4. As a result its value went up by
𝒃 𝟕 𝟒𝒂+𝟓𝒃
a) 15:44 b) 22:35 c) 15:49 Rs.12000. Find its initial value.
d) 22:47 e) Can't be determine a) Rs.9000 b) Rs.15000 c) Rs.12000
𝒂 𝟑 𝟓𝒂𝟐 +𝒃 d) Rs.18000 e) None of these
ii) = , Find the value of
𝒃 𝟐 𝟓𝒂−𝒃
a) 13:47 b) 47:13 c) 22:47

For details on our programs: - Contact: +91 9930202329, +91, 8291924300 or visit www.campuscredentials.com (2)
a) 1225 b) 1450 c) 1425 d) 1275
2-Percentage, Profit & Loss
12) The cost price of 20 articles is the same as the selling price
of x articles. If the profit is 25%, then the value of x is:
1) Dipin's score is 15% more than that of Rafi. Rafi's score is
10% less than that of Chandar. If the difference between the a) 15 b) 16 c) 18 d) 25
scores of Dipin and Chandar is 14, what is the score of Rafi?
13) An article was sold at a profit of 20%. If both the cost price
a) 180 b) 360 c) 120 d) 480
and selling price decreases by Rs.3000, then the profit would
be 25%. Find the original cost price.
2) Anil spent 16.66% of his income on rent. 25% of the
remaining on food, 60% of the remaining on education. If he a) Rs.15000 b) Rs.30000 c) Rs.45000
saves ₹4500 per month, find his income d) Rs.60000
a) Rs.16,000 b) Rs.17,000
c) Rs.18,000 d) Rs.19,000 14) The loss incurred by selling 16m of a cloth equals the
selling price of 4m of that cloth. Find the loss percentage.
3) Pravin purchased the articles for Rs 123684. He sold 60% a) 15% b) 20% c) 33.33% d) 25%
of those at a profit of 16.66% and rest at a loss. Find the loss
percentage on the remaining if the overall loss is 14%? 15) A trader mixes 25% kerosene to his petrol and then he sells
a) 20% b) 60% the whole mixture at the price of petrol. If the cost price of
kerosene be 50% of the cost price of petrol, what is the net profit
c) 33.33% d) 66.66%
percent?
𝟏 𝟏 𝟏
4) Numerator of the fraction increase by 418% and a) 11 % b) 12 % c) 9 % d) 20%
𝟗 𝟗 𝟏𝟏
denominator increase by 117% then the value of the fraction is
𝟑
. what is the original fraction? 16) A shopkeeper sells his goods at the same price as what he
𝟑𝟏
𝟏𝟏 𝟑 𝟓 𝟐 pays his supplier. But when he buys from his supplier, he takes
a) b) c) d) 10% more than the indicated weight and when he sells to his
𝟖𝟖 𝟕𝟒 𝟕𝟔 𝟖𝟑
customers he gives 10% less than the indicated weight. Find his
5) A dealer gives as much discount (in percent) as the markup profit percentage.
(in percent) above the cost price. What is the profit or loss 𝟐 𝟐 𝟐
a) 18 % b) 22 % c) 20 % d) 20%
percent? 𝟏𝟏 𝟗 𝟗

a) 10% b) 1% c) 4%
d) can’t be determined 3-Partnership, SI & CI
6) Two articles are sold at the same price. One at a profit of 1) A and B invest in a business in the ratio 3:2 for the same time
20% and another one at a loss of 20%. What is the overall profit period. If 5% of the total profit goes to charity and A's share is
or loss percentage? Rs. 855, the total profit is.
a) 4% loss b) 4% profit c) No profit, No loss a) Rs. 1425 b) Rs. 1500 c) Rs. 1537.50
d) 2% loss d) Rs. 1576 e) None of these

7) There are five equal glasses containing milk in the ratio 2) A, B and C jointly thought of engaging themselves in a
3:4:5:6:7. How many glasses are at least 50% full of milk if the business venture. It was agreed that A would invest Rs. 6500
total volume of milk in the glasses is 60% of the total volume of for 6 months, B, Rs. 8400 for 5 months and C, Rs. 10,000 for 3
the glasses? months. A wants to be the working member for which, he was
a) 2 b) 3 c) 4 d) 5 to receive 5% of the profits. The profit earned was Rs. 7400.
Calculate the share of B in the profit.
8) Price of the article increase by 12%, 20% and 25% a) Rs. 1900 b) Rs. 2660 c) Rs. 2800
successively. What is the effective percentage increase in the d) Rs. 2840 e) None of these
price of the article?
a) 65% b) 68% c) 72% d)76% 3) A, B and C enter into a partnership in the ratio 7/2:4/3:6/5.
After 4 months, A increases his share 50%. If the total profit at
9) The length and the breadth of rectangle are increase by 12% the end of one year be Rs. 21,600, then B's share in the profit
and x% respectively, thereby area of rectangle increase by is.
40%. What is the value of x? a) Rs. 2100 b) Rs. 2400 c) Rs. 3600
a) 25 b) 28 c) 31 d) 34 d) Rs. 4000 e) None of these

10) A sugar trader declares that he sells sugar at the cost price. 4) The simple interest on a certain sum for 3 years is Rs.8178.
However, he uses a weight of 450g instead of 500g. His The rate of interest are 7%, 10% and 12% per annum for the
percentage profit is: first, second and third year respectively. Find the sum.
𝟏 𝟐 a) Rs. 28200 b) Rs. 29200 c) Rs. 22800
a) 10% b) 11 %c) 12 % d) 12%
𝟗 𝟗
d) Rs. 22900 e) None of these
11) The difference between CP and SP of a table fan is Rs 175
where it gives the profit of 14%. What is SP of that fan (in Rs.)?

For details on our programs: - Contact: +91 9930202329, +91, 8291924300 or visit www.campuscredentials.com (3)
5) In how many years does a sum amount to four times itself at 2) 16 men went to hotel. 15 of them paid Rs.80 each and the
25% p.a. simple interest? 16th man paid Rs.75 more than the average bill of all the sixteen
a) 15 years b) 12 years c) 10 years men. Find the total bill
d) 17 years e) None of these a) Rs.1020 b) Rs.1280 c) Rs.1360
d) Rs.1360 e) None of these
6) A certain sum yields Rs.840 more simple interest if it is
invested for two years at 18% p.a. than at 12% p.a. Find the 3) The Average age of husband, wife and their child 3 years ago
principal was 27 years and that of wife and child 5 years ago was 20
a) Rs. 7500 b) Rs. 8500 c) Rs. 8000 years. The present age of husband is.
d) Rs. 7000 e) None of these a) 35 years b) 40 years c) 50 years
d) 45 years e) None of these
7) Amit borrowed a sum of money from Yogesh at simple
interest. The rate of interest is 10% per annum for the first 2 4) The average age of 24 students and the principal is 15 years.
years and 12% per annum for the next 3 years and 15% per When the principal’s age is excluded, the average age
annum thereafter. If amit paid Rs.5332 as interest after 7 years, decreases by 1 year. What is the age of the principal?
then find the sum that he borrowed. a) 38 b) 40 c) 39 d) 37 e) None of these
a) Rs. 6800 b) Rs. 6600 c) Rs. 6200
d) Rs. 6400 e) None of these 5) Sachin Tendulkar has a certain average for 9 innings. In the
10th inning, He scores 100 runs thereby increases his average
8) A sum of money doubles in 3 years at compound interest. In by 8 runs. His new average is
how many years does it amount to 16 times itself? a) 20 b) 24 c) 28 d) 32 e) None of these
a) 25 b) 18 c) 30 d) 12
e) None of these 6) The average length of any 4 fingers of my left hand is 600mm.
Then the average length of all the five fingers of my left hand
9) The compound interest, interest being compounded is.
annually, on a certain sum for the second and the third year are a) 800mm b) 750mm c) 480mm
Rs.2880 and Rs.3398.40 respectively. Find the rate of interest d) 600mm e) Cannot be determined
per annum.
a) 18% b) 14% c) 20% 7) Vessel A contain 5 litres of milk and vessel B contains 5 litres
d) 16% e) None of these of water. One litre of milk is taken from A and is poured into B.
one litre of the mixture in B is then poured into A. if the present
10) What is the compound interest on Rs.8000 at 20% p.a. for quantity of milk in B and water in A are X and Y respectively,
9-month, interest being compounded quarterly? then which of the following holds true?
a) Rs. 1281 b) Rs. 1261 c) Rs. 1241 a) X>Y b) X=Y c) X<Y
d) Rs. 1221 e) None of these d) Cannot be determine

11) What is the difference between compound interest on 8) Fresh grapes contain 84% water while raisins contain 20%
Rs.12000 at 20% p.a. for one year when compounded yearly water. How many kg of raisins can be made from 80 kg of fresh
and half yearly? grapes?
a) Rs. 140 b) Rs. 120 c) Rs. 130 a) 16 kg b) 18 kg c) 20 kg
d) Rs. 110 e) None of these d) 22 kg e) None of these

12) Palak borrows money on compound interest and return it 9) 8 litres are drown from a cask full of wine and is then filled
in two equal half - yearly instalments of Rs 4410 each. Find the with water. The operation is performed three more times. The
interest charged if the rate of interest is 10 % p.a. compounded ratio of quantity of wine now left in the cask to that of water is
half yearly. 16:65 How much wine the cask holds originally?
a) Rs 600 b) Rs 820 c) Rs 620 a) 18 litres b) 24 litres c) 42 litres
d) Rs 660 e) None of these d) 32 litres e) None of these

13) A sum of Rs 7500 is to be paid back in three equal quarterly 10) A cup of milk contains 3 parts of pure milk and 1 part of
instalments. How much is each instalment if the interest is water. How much the mixture must be
compounded quarterly at 8% p.a.? withdrawn and water substituted in order that resulting mixture
a) Rs. 2600.66 b) Rs. 2700.06 c) Rs. 2701.58 may be half milk and half water.
d) Rs. 2706.08 e) None of these 𝟑 th 𝟏 th 𝟐 rd
a) mixture b) mixture c) mixture
𝟒 𝟒 𝟑
𝟏 rd
d) mixture e) None of these
4-Average, Mixture & Alligation 𝟑

11) In what ratio must a grocer mix two varieties of pulses


1) The average of 11 consecutive natural numbers is 'x'. If 6 th
costing Rs.15 and Rs.20 per kg respectively so as to get a
number is 12, find x
mixture worth Rs.16.50 per kg?
a) 9 b) 10 c) 18 d) 11 e) None of these
a) 3:7 b) 5:7 c) 7:3 d) 7:5 e) None of these

For details on our programs: - Contact: +91 9930202329, +91, 8291924300 or visit www.campuscredentials.com (4)
12) Two cans A and B contains milk worth Rs.7 per litre and
Rs.9 per litre respectively. If the contents of A and B are 7) 33 binders can bind 2200 books in 24 days. Find the number
transferred to another can C in the ratio 3 : 7 then the cost per of binders required to bind 1800 books in 18 days.
litre of the mixture in can C is: a) 33 b) 30 c) 40 d) 36 e) None of these
a) Rs.9.40 b) Rs.10.10 c) Rs.7.40
d) Rs.8.40 e) None of these 8) Thirty-six men can do a work in 20 days. In how many days
can 24 men do the same work, given that the time spent per day
13) A milk vendor has 2 cans of milk. The first contains 25% is decreased by 1/3rd of the previous time?
water and the rest milk. The second contains 50% water. How a) 42 b) 39 c) 48 d) 45 e) 52
much milk should he mix from each of the containers so as to
get 12 litres of milk such that the ratio of water to milk is 3 : 5? 9)10 men and 8 women can do a work in 10 days, 8 men and 19
a) 4 litres, 8 litres b) 6 litres, 6 litres women can do the same work in 8 days. In how many days can
c) 5 litres, 7 litres d) 7 litres, 5 litres 4 men and 6 women do the same work?
e) None of these a) 20 b) 5 c) 15 d) 10 e) 25

14) The quantity of sugar costing Rs.6.10 per kg must be mixed 10) Two men and four women can complete a piece of work in
with 126 kg of sugar priced at Rs.2.85 per kg, so that 20% may four days. One man and two women take the same time as five
be gained by setting the mixture at Rs.4.80 per kg. women to complete the work. Find the time by one woman to
a) 126 kg b) 89 kg c) 69 kg complete the work
d) 84 kg e) None of these a) 20 days b) 25 days c) 30 days
d) 40 days e) 45 days
15) Two qualities of rice at Rs.63 per kg and Rs.67.50 per kg
are mixed with another quality of rice in the ratio 2:2:3. The 11) A contractor employs 280 men to complete a work in 10
final mixture sold at Rs.76.50 per kg then the rate of third days. But after 3 days it was found that only 1/4 th of the work
quality rice per kg was: was done. How many additional men are required to finish the
a) Rs.87.50 b) Rs.91.50 c) Rs.81.50 d) Rs.99.50 work on time?
a) 360 b) 120 c) 140 d) 80 e) 120
5-Time and Work
12) Three pipes, of which two are filling and the third is
1) A can do a work in 12 days and B can do the same work in draining, are fitted to a tank. The two pipes can fill an empty
tank in 30 minutes and 40 minutes and the draining pipe
36 days, in how many days can they complete the work, working
together? empties the full tank in 60 minutes. In how many minutes is the
empty tank completely full when all the three pipes are opened
a)15 b) 9 c) 24 d)18 e)12
simultaneously?
2) P and Q can do a work in 15 days and 21 days respectively. a) 36 b) 60 c) 30 d) 48 e) 24
P joins Q after some days and the total work is completed in 14
days. After how many days did P join Q? 13) Pipes A and B can fill an empty tank in 20 minutes and 30
minutes respectively, whereas C can empty the full tank in x
a) 8 b) 7 c) 6 d) 9 e)12
minutes. When pipes A, B and C are opened simultaneously the
3) A, B and C can do a work in 10 days, 15 days and 20 days tank will be filled in 15 minutes. What is the value of x?
a) 50 b) 55 c) 45 d) 60 e) 30
respectively. They work together and completed the work,
earning ₹312. What is the share of C?
14) Pipe A can fill a tank in 3 hours. A can fill the tank in 5
a) Rs.138 b) Rs.98 c) Rs.72
d) Rs.112 e) Rs.108 hours if it works along with the outlet pipe B. If B can empty the
tank at 6 litres per minute, then what is the capacity of the tank?
4) Mr. P and Mr. Q can build a wall in 10 days; Mr. Q & Mr. a) 2850 litres b) 3450 litres c) 3200 litres
R can take 14 days to build the same wall and Mr. P and Mr. R d) 3600 litres e) 2700 litres
can do it in 8 days. Who among them will take more time when
they work alone? 6-Time, Speed and Distance
a) P b) Q c) R d) data inadequate
1) A car covers a distance from town A to town B at a speed of
5) M and N together finish a work in 30 days. If they work 70 kmph and covers the distance from town B to town A at a
together for 20 days and then N left. The remaining work was speed of 90 kmph. What is the approximate average speed of
done by M alone in the next 20 days. N alone can finish the the car?
work in a) 70 kmph b) 79 kmph c) 80 kmph
a) 48 days b) 70 days c) 40 days d) 90 kmph e) None of these
d) 50 days e) 60 days 2) Without stoppages a train travels a certain distance with an
6) Lalit and Bipin can do a work in 3 days and 7 days average speed of 60 km/h and with stoppages with an average
respectively. If they work on alternate days, then find the speed of 40 km/h. How many minutes per hour does the train
minimum number of days required to complete the work? stops?
𝟏 𝟏 a) 30 minutes b) 20 minutes c) 10 minutes
a) 4 b) 4 c) 7 d) 3 e) None of These
𝟑 𝟕

For details on our programs: - Contact: +91 9930202329, +91, 8291924300 or visit www.campuscredentials.com (5)
d) 15 minutes e) None of these d) 20 metre e) None of these

3) A car starts from a city X at 9 am and travels towards another 12) In a race Palak gives Asha a head start of 350 m and still
city Y at 70 km/hr. Another car starts from the city Y at 10 am 𝟏
beats her by 50 m. If palak's speed is 1 times Asha's speed,
𝟒
and travels towards the city X at 85 km/hr. If the distance
What is the length of race?
between the cities X & Y is 690 km then at what time do they
a) 1 km b) 2 km c) 4 km
meet?
d) 1.5 km e) None of these
a) 1 pm b) 1.30 pm c) 2 pm
d) 2.15 pm e) None of these
13) Three runners A, B and C run a race, with runner A
finishing 12 meters ahead of runner B and 18 meters ahead of
4) Two cyclists simultaneously start from city A to city B and
runner C, while runner B finishes 8 meters ahead of runner C.
city B to city A respectively. After crossing each other first
Each runner travels the entire distance at a constant speed.
cyclist reaches city B in 4 hours while the second cyclist
What was the length of the race?
reaches city A in 9 hours. Find the ratio of the speed of two
a) 36 meters b) 72 meters c) 60 meters
cyclists.
𝟐 𝟑 𝟒 𝟑
d) 48 meters e) None of these
a) b) c) d) e) None of these
𝟑 𝟐 𝟑 𝟒
14) P, Q, R run around a circular track 1200 m long with speed
5) A police van travelling at 60 kmph crosses an escaping thief of 9, 18, 27 kmph. If they start at the same point and at the same
travelling in opposite direction at 48 kmph. The police van has time in the same direction, when will they meet again at the
to travel for a further 5 minutes before it can find a gap in the starting point?
median where it can take a U tern and start chasing the thief. a) 480 sec b) 360 sec c) 240 sec
After how much time police van catch the thief? d) 300 sec e) None of these
a) 25 minutes b) 45 minutes c) 50 minutes
d) 32 minutes e) None of these 15) A can run one full round of a circular track in 6 min and B
in 15 min. If both A and B start simultaneously from the same
6) A train travelling at 36 kmph completely crosses another starting point then How many times would they met in the time
train having half its length and travelling in the opposite B has completed 10 rounds when running in same direction,
direction at 54 kmph, in 12 seconds. If it also passes a railway and In opposite direction?
platform in 60 seconds, the length of the platform is : a) 15, 10 b) 25, 30 c) 15, 35
a) 300 metres b) 350 metres c) 450 metres d) 35, 10 e) None of these
d) 500 metres e) None of these
7-Numbers
7) Two trains running in the opposite direction cross a man 1) Find unit digit
standing on platform in 31 sec and 23 sec respectively. Two
i) 687567
trains cross each other in 29 sec. What is the ratio of their
a) 7 b) 3 c) 9 d) 1 e) None of these
speeds?
ii) 𝟕𝟔𝟑4539
a) 3:1 b) 4:3 c) 2:1 d) 3:2 e) None of these
a) 3 b) 1 c) 9 d) 7 e) None of these
iii) 568564
8) Two trains running at 45 kmph and 54 kmph cross each other
a) 8 b) 4 c) 2 d) 6 e) None of these
in 12 seconds when they run in opposite directions. When they
run in the same direction, a person in the faster train observe
2) i)If a number is represented by N=84×192×217×301. What
that he crossed the other train in 32 seconds. Find the lengths
will be remainder when N is divided by 27?
of the two trains?
a) 7 b) 6 c) 5 d) 9 e) None of these
a) 250 m, 90 m b) 260 m, 70 m c) 240 m, 90 m
ii) What will be remainder when 568+5468+2543+65259 is
d) 250 m, 80 m e) None of these
divided by 13?
a) 10 b) 11 c) 12 d) 13 e) None of these
9) A man can row a certain distance downstream in 2 hours
3) i) Which of the following number is divisible by 11, 42, 72?
while he takes 3 hours to come back. If the speed of the stream
a) 8736 b) 6240 c) 10032
be 6 km/hr then the speed of the man in still water is:
d) 7392 e) None of these
a) 15km/hr b) 30km/hr c) 25km/hr
ii) Which number we have to add in 2456210 to make it divisible
d) 29km/hr e) None of these
by 11?
a) 9 b) 7 c) 3 d) 5 e) None of these
10) A boat takes 7 hours to cover 24 km distance and comes
iii) Which number we have to subtract from 468951 to make it
back. And, it can cover 2 km with the stream in the same time
divisible by 9?
as 1.5 km against the stream. The speed of the stream is:
a) 14 b) 15 c) 17 d) 18 e) None of these
a) 1 km/hr b) 2 km/hr c) 3 km/hr
iv) What is the smallest number by which the number 222264 is
d) 4 km/hr e) None of these
to be divided such that the quotient becomes a perfect cube?
a) 3 b) 4 c) 7 d) 8 e) None of these
11) In a 100 metre race. A beats B by 20 metre and C by 30
meters, then in the same race B beats C by how many metres?
4) i) The remainder when 6100 is divided by 7, is:
a) 10 metre b) 12.5 metre c) 15 metre
a) 3 b) 0 c) 1 d) 2 e) None of these

For details on our programs: - Contact: +91 9930202329, +91, 8291924300 or visit www.campuscredentials.com (6)
ii) The remainder when 54457 is divided by 63 is, a) 130 b) 140 c) 150 d) 160 e) None of these
a) 19 b) 17 c) 13 d) 18 e) None of these
16) In a large school auditorium, the students are made to sit
5) Three number are in the ratio of 3 : 4 : 5 and their L.C.M. is to watch the programmes. If the teachers make a row of
2400. Their H.C.F. is: students of 16 each, there will be 12 students left. If they make
a) 40 b) 80 c) 120 d) 200 e) None of these rows of 24 each, then there will be 20 students left, if they make
rows of 25 each, there will be 21 students left and if they make
6) LCM and HCF of 2/7,3/14 and 5/3 is rows of 30 each, there will be 26 students left. What is the
𝟏 𝟑𝟎 𝟏 minimum number of students present in the school?
a) 45, b) 35, c) 30,
𝟏𝟒 𝟒𝟐 𝟒𝟐
𝟑𝟎 a) 1216 b) 1784 c) 1196
d) 25, e) None of these d) 2396 e) None of these
𝟏

7) The greatest possible length which can be used to measure 8-Geometry & Mensuration
exactly the lengths 7 m, 3 m 85 cm, 12 m 95 cm is:
a)15 cm b) 35cm c) 27cm
1) The wheel of a motorcar makes 1000 revolutions in moving
d) 52cm e) None of these
550m. Find the diameter of wheel.
a) 18 cm b) 20 cm c) 17.5 cm d) 18.5 cm
8) 6 different sweet varieties of count 32, 216, 136, 88, 184, 120
were ordered for a particular occasion. They need to be packed
2) In parallelogram ABCD, AP and BP are the angle bisectors
in such a way that each box has the same variety of sweet and
of DAB and ABC. Find APB.
the number of sweets in each box is also the same. What is the
a) 850 b) 900 c) 700 d) 800 e) 950
minimum number of boxes required to pack?
a) 129 b) 64 c) 48
3) In a trapezium PQRS, PQ is parallel to RS and PQ=10cm
d) 97 e) None of these
and RS=20cm. What is the length of the line UV which is
parallel to PQ and RS and divides the distance between them
9) i) If p×q = 361, p, q are integers then the value of p + (q-1)2
in the ratio 2:3 respectively?
can be:
a) 15 cm b) 12 cm c) 14 cm
a) 343 b) 111 c) 109 d) None of these
d) 16 cm e) 10 cm
ii) If p×q = 289 and p, q are integers then find the integer value
of p/q.
a) 289 b) 1 c) a & b d) None of these

10) i) Simplify: [(1113 + 626)2 + (1113 - 626)2] / (1113x1113


+ 626x626)
a) 1 b) 2 c) 11132 d) 4872 e) None of these 4)
ii) Simplify: (1004 x 1004 x 1004 + 382 x 382 x 382)/ (1004 x In the above figure, AB = 10 cm, CD = 40 cm, Find EF.
1004 – 1004 x 382 + 382 x 382) a) 5 b) 6 c) 8 d) 4
a) 1386 b) 622 c) 1004 x 382
d) 1 e) None of these 5) Sum of the interior angles of a polygon is 1620. How many
sides does polygon have?
11) If the sum of two numbers is 55 and the H.C.F. and L.C.M. a) 12 b) 11 c) 10 d) 9 e) 8
of these numbers are 5 and 120 respectively, then the sum of
the reciprocals of the numbers is equal to: 6) In the diagram find the radius of the circle.
a) 55/601 b) 601/55 c) 11/120 a) 10 cm b) 50 cm c) 40 cm
d) 120/11 e) None of these d) 60 cm e) 30cm

12) i) Find the number of factors of 4320.


a) 40 b) 42 c) 36 d) 48 e) None of these
ii) Find the sum of factors of 4320.
a) 13172 b) 15120 c) 10890
d) 12144 e) None of these
13) Find the smallest 3 digit number which when divided by 4,
7 and 11 leaves a reminder of 3 in each case.
a) 308 b) 305 c) 311 d) 309 e) None of these

14) Find the smallest number which when divided by 5, 7, 11


and 13 leaves respective remainder of 2, 4, 8 and 10
a) 5005 b) 5002 c) 5500 7) T is an obtuse angle triangle. Two of its sides are 7 cm and
d) 5020 e) None of these 13 cm. How many possibilities exist for T such that the third
side has an integral measure?
15) Find the largest number with which when 437, 857 and a) 12 b) 7 c) 16 d) 9 e) 15
1557 are divided, leaves the same remainder in each case

For details on our programs: - Contact: +91 9930202329, +91, 8291924300 or visit www.campuscredentials.com (7)
c) 4√47𝑓𝑡 d) None of these
8) The area of similar triangles ABC and XYZ are 54 sq.cm and
150 sq.cm respectively. Find the perimeter of triangle XYZ, if 18) A peacock is sitting on a 19 m long pole, a snake is
the perimeter of the triangle ABC is 36 cm. approaching the hole which is at bottom of the pole, the snake
a) 100 cm b) 30 cm c) 50 cm is 27 m away from the hole, if their speeds are same, find the
d) 60 cm e) 70 cm distance from the hole at which the peacock pounces over the
snake.
9) How far from the centre of a circle of diameter 170 cm is the a) 3.4 m b) 6.8 m c) 5.9 m d) 7.3 m
chord 26 cm long?
a) 84 cm b) 85 cm c) 82 cm 9-Permutation & Combination
d) 83 cm e) 81 cm
1) If Naresh has 5 different shirts and 7 different pairs of pants,
10) A horse is put outside a fenced rectangular plot 60m×20m
how many different combinations could he wear?
and is tethered to one of the corners of the plot by rope of length
a) 35 b) 30 c) 40 d) 45 e) None of these
30m. Find the total area that it can graze.
a) 600𝝅 sq.m. b) 750𝝅 sq.m. c) 700𝝅 sq.m. 2) There are 8 stations between Ernakulum and Chennai. How
d) 500𝝅 sq.m. e) None of these many second-class tickets have to be printed, so that a
passenger can travel from one station to any other station?
11) A solid sphere is cut into 8 identical pieces by three a) 70 b) 80 c) 90 d) 100 e) None of these
mutually perpendicular cuts. By what percentage is the sum of
total surface area of the eight pieces more than the total surface 3) How many 4-digit numbers can be formed using digits 1, 2,
area of the original sphere? 3, 4 and 5?
a) 125% b) 150% c) 175% d) 200% i) Without repetition of digits
a) 240 b) 420 c) 625 d) 120 e) None of these
12) If the distance between the tops of two poles with lengths ii) With repetition of digits
13.42 m and 8.484 m both standing erect, is 6.17 m. Find the a) 420 b) 625 c) 120 d) 240 e) None of these
distance between their bases.
a) 1.234 m b) 3.702 m c) 4.936 m 4) How many 4-digit numbers can be formed using digits 0, 1,
d) 2.468 m e) None of these 2, 3, 4 and 5?
i) Without repetition of digits
13) Radius and height of a right cylinder are each increase by a) 60 b) 120 c) 360 d) 300 e) None of these
10%. Find the percentage increase in its volume. ii) With repetition of digits
a) 30% b) 33.33% c) 33.1% a) 625 b) 1080 c) 750 d) 36 e) None of these
d) 300% e) None of these
5) How many 4 digit numbers can be formed using digits 0, 5,
14) A conical cup when filled with ice-cream forms a 6, 7, 8 and 9 which are divisible by 4?
hemispherical shape on its open end. Find the approximate i) Without repetition of digits
volume of ice-cream. If the radius of the base of the cone is 3.5 a) 36 b) 72 c) 96 d) 108 e) None of these
cm and the vertical height of cone is 7 cm. ii) With repetition of digits
a) 165 cm3 b) 185 cm3 c) 170 cm3 a) 240 b) 360 c) 270 d) 72 e) None of these
3 3
d) 175 cm e) 180 cm
6) In how many ways 4 girls and 4 boys can seat in a row such
15) The minute hand of a clock is 24.5 cm long. Find the area that no 2 boys and no 2 girls are together?
swept by it between 10:10 am and 10:40 am of the same day. a) 20240 b) 1152 c) 576
a) 900 sq.cm b) 948.75 sq.cm d) 40320 e) None of these
c) 943.25 sq.cm d) 953.25 sq.cm
7) How many ways 8 persons can seat around a circular table
16) Four points P, Q, R and S lie on a straight line in the XY facing the centre such that 3 particular persons are always
plane, such that PQ=QR=RS and the length of PQ is 5 meters. together?
A man wants to go from P to S. but there are dogs tethered with a) 5040 b) 40320 c) 241920
5 metre chains at point Q and R. The man would not go within d)30240 e) None of these
the reach of any dog. Minimum distance in meters the man must
travel to reach the point S is. 8) A letter lock consists of three rings each marked with six
𝟐𝟎𝝅 𝟒𝟎𝝅
a) 5(π+1) b) c) d) 15√𝟐 different letters. The number of distinct unsuccessful attempts
𝟑 𝟑
to open the lock is at the most?
17) A mosquito is flying in a room having dimensions 8 ft × 6 a) 120 b) 216 c) 360 d) 215 e) None of these
ft × 10 ft. It has to fly from one corner to the farthest opposite 9) How many 7 lettered words without repetition can be formed
corner of a room to collect food. It collects the food and returns using the letters of the word "MISTAKE" so that no two vowels
to its original spot. Find the minimum possible distance are together?
covered by the mosquito? a) 1240 b) 1220 c) 1420
a) 20√2𝑓𝑡 b) 20 ft. d) 1440 e) None of these

For details on our programs: - Contact: +91 9930202329, +91, 8291924300 or visit www.campuscredentials.com (8)
a) 48 b) 64 c) 24
10) A committee of 6 members is to be formed out of 10 d) 60 e) None of these
members. How many ways can a selection of six members made
so that, 19) In how many ways can Anil uncle distribute 10 identical
i) A particular person is always included? chocolates in three children A, B and C?
a) 126 b) 84 c) 210 d) 96 e) None of these a) 410 b) 310 c) 66
ii) A particular person is always excluded? d) 63 e) None of these
a) 210 b) 120 c) 84 d) 126 e) None of these
10-Probability
11) In how many ways you can arrange the letters of the word
i) BINOCULARS 1) Four fair coins are tossed simultaneously. What is the
𝟏𝟎!
a) b) 9! c) 10C10 d) 10! e) None of these probability of getting exactly 2 tails?
𝟓!
𝟑 𝟓 𝟕 𝟏
ii) ASSISTANTS a) b) c) d) e) None of these
𝟏𝟎! 𝟏𝟎! 𝟏𝟎! 𝟖 𝟖 𝟏𝟔 𝟐
10
a) b) c) d) P10 e) None of these
𝟐!×𝟒!×𝟐! 𝟔! 𝟖!
iii) MISSISSIPPI 2) Four fair coins are tossed simultaneously. What is the
𝟏𝟎! 11 𝟏𝟏! 𝟏𝟏! probability of getting at least 2 heads?
a) b) P10 c) d)
𝟐!×𝟐!×𝟒! 𝟏𝟎! 𝟐!×𝟒!×𝟒! 𝟗 𝟓 𝟏𝟏 𝟏𝟑
e) None of these a) b) c) d) e) None of these
𝟏𝟔 𝟖 𝟏𝟔 𝟏𝟔

12) In the figure given bellow, the lines represent the one-way 3) If a die is thrown once, what is the probability of getting a
roads allowing cars to travel only eastwards or northwards. In prime number?
𝟐 𝟏 𝟏 𝟏
how many ways car travel from the City A to the City B? a) b) c) d) e) None of these
𝟑 𝟔 𝟐 𝟑

4)Find a probability of getting a total more than 7, when


sequentially throw a pair of dice?
𝟐 𝟑 𝟓 𝟕
a) b) c) d) e) None of these
𝟏𝟐 𝟏𝟐 𝟏𝟐 𝟏𝟐

5) Three dices are thrown up simultaneously. What is the


probability of having all the three different faces on the top?
a) 330 b) 304 c) 420 𝟒 𝟏 𝟓 𝟐
d) 840 e) None of these a) b) c) d) e) None of these
𝟗 𝟑 𝟗 𝟑

13) Rohit attempted a multiple-choice question paper 6) Two cards are drown from a standard deck of 52 playing
consisting of 10 questions and each question having 4 choices. cards. What is the probability the both the cards are honour
The number of ways in which he can attempt the entire paper if cards?
he is making the answers at random is 𝟑𝟔 𝟏𝟔 𝟐𝟎 𝟏𝟖
a) b) c) d) e) None of these
𝟐𝟐𝟏 𝟐𝟐𝟏 𝟐𝟐𝟏 𝟐𝟐𝟏
a) 10P4 b) 410 c) 10C4 d) 104 e) None of these
7) i) Two cards are drown from a standard deck of 52 playing
14) There are 15 points in a plane of which 8 of them on a
cards one after another with replacement. What is the
straight line. Then by how many ways,
probability the both the cards are face cards?
i) Straight lines can be formed? 𝟗 𝟏𝟖 𝟏𝟓 𝟏𝟐
a) 105 b) 21 c) 78 d) 84 e) None of these a) b) c) d) e) None of these
𝟏𝟔𝟗 𝟏𝟔𝟗 𝟏𝟔𝟗 𝟏𝟔𝟗
ii) Triangles can be formed?
a) 399 b) 400 c) 434 d) 235 e) None of these ii) Two cards are drowned from a standard deck of 52 playing
cards one after another without replacement. What is the
15) The total number of selections of 5 fruits which can be made probability the both the cards are black cards?
𝟐𝟕 𝟐𝟓 𝟑𝟏 𝟐𝟏
from 4 oranges, 3 apples and 2 bananas taking at least one of a) b) c) d) e) None of these
𝟏𝟎𝟐 𝟏𝟎𝟐 𝟏𝟎𝟐 𝟏0𝟐
each kind is:
a) 44 b) 55 c) 98 d) 56 e) None of these 8) A card is drawn from a standard deck of 52 playing cards.
What is the probability that the card is an ace or a king?
16) In how many ways selection of at least 1 ball be made from 𝟑 𝟐 𝟒 𝟓
6 distinct balls? a) b) c) d) e) None of these
𝟏𝟑 𝟏𝟑 𝟏𝟑 𝟏𝟑
a) 26 b) 96 c) 63 d) 120 e) None of these 9) A card is drawn from a standard deck of 52 playing cards.
17) There are 4 identical books on Maths, 5 identical books on What is the probability that the card is a red card or a king?
𝟔 𝟏𝟓 𝟖 𝟏𝟓
Science, and 3 identical books on English. In how many ways a) b) c) d) e) None of these
𝟏𝟑 𝟐𝟔 𝟏𝟑 𝟐𝟔
one or more books can be selected?
a) 120 b) 119 c) 4096 10) A box contains 12 electric bulbs, of which four are
d) 4095 e) None of these defective. Two bulbs are drawn at random from the bag. What
18) Five persons A, B, C, D and E are standing in a row facing is the probability that both are defective bulbs?
north. Find the number of possible arrangements in which A is 𝟒 𝟑 𝟐 𝟏
a) b) c) d) e) None of these
to the left of B? 𝟏𝟏 𝟏𝟏 𝟏𝟏 𝟏𝟏

For details on our programs: - Contact: +91 9930202329, +91, 8291924300 or visit www.campuscredentials.com (9)
11) The probability that it rains on a certain day is 0.6 and the
probability that it rains on certain another day is 0.8. What is
the probability that it will not rain on both days?
a) 0.04 b) 0.08 c) 0.12
d) 0.16 e) None of these

12) Out of all the 2-digit integers between 1 and 100, a number
is selected at random. What is the probability that the selected
number is not divisible by 7?
𝟏𝟑 𝟕𝟗 𝟖𝟎 𝟕𝟕
a) b) c) d) e) None of these
𝟗𝟎 𝟗𝟎 𝟗𝟎 𝟗𝟎

13) A seven-letter word is written at random with the letters of


the word 'REPLACE'. What is the probability that it is
REPLACE?
𝟏 𝟏 𝟏 𝟏
a) b) c) d) e) None of these
𝟔𝟑𝟎 𝟏𝟐𝟔𝟎 𝟐𝟓𝟐𝟎 𝟓𝟎𝟒𝟎

14) Six persons sit in a row randomly. What is a probability


that three particular persons always appear together?
𝟏 𝟐 𝟑 𝟒
a) b) c) d) e) None of these
𝟓 𝟓 𝟓 𝟓

15) Probability that a speak truth is 7/10 and the probability


that b speaks truth is 11/15. What is the probability that both
speak false while making a statement?
𝟔 𝟒 𝟐 𝟖
a) b) c) d) e) None of these
𝟐𝟓 𝟐𝟓 𝟐𝟓 𝟐𝟓

16) The probability that M hits the target is 0.6, N hits the target
is 0.65 and K hits the target is 0.55. What is the probability that
at least of the person hits the target?
a) 0.877 b) 0.897 c) 0.917
d) 0.937 e) None of these

17) A bag contains four red, three green, two yellow and five
blue marbles. Three marbles are drawn at random from the bag
i) What is the probability that all the marbles are of the same
colour?
𝟏𝟓 𝟏𝟗 𝟐𝟑 𝟐𝟕
a) b) c) d) e) None of these
𝟑𝟔𝟒 𝟑𝟔𝟒 𝟑𝟔𝟒 𝟑𝟔𝟒
ii) What is the probability that all the marbles are of different
colours?
𝟏𝟏 𝟏 𝟏𝟓 𝟏𝟏
a) b) c) d) e) None of these
𝟐𝟔 𝟐 𝟐𝟔 𝟓𝟐
iii) What is the probability that at least one green coloured
marble is drawn?
𝟏𝟓𝟗 𝟏𝟕𝟗 𝟏𝟗𝟗 𝟐𝟎𝟗
a) b) c) d) e) None of these
𝟐𝟔𝟒 𝟐𝟔𝟒 𝟐𝟔𝟒 𝟐𝟔𝟒

For details on our programs: - Contact: +91 9930202329, +91, 8291924300 or visit www.campuscredentials.com (10)
5) A vendor bought toffees at 6 for a rupee. How many for a
Quantitative Aptitude Handout rupee must he sell to gain 20%?
Homework a) 3 b) 4 c) 5 d) 6

1-Equations, Ratio, Proportion & Variation 3-Partnership, SI & CI


1) A rope of 77 meters is cut into 2 pieces such that one piece is 1) Amol started a business with Rs.2000 and after 5 months,
𝟒⁄ th of the other piece. What is the length of 𝟑⁄ th of the Ashok joined him with Rs.4500. Amol received Rs.45250
𝟕 𝟏𝟒
longer piece in meters? including 10% of the profits as commission for managing the
a) 22.5 b) 245 c) 17.5 business at the end of one year. What amount did Ashok receive?
d)10.5 e) None of these a) Rs.47250 b) Rs.47500 c) Rs.47750
d) Rs.48000 e) None of these
2) Father is aged three times more than his son Sunil. After 8
years, he would be two and a half times of Sunil's age. After 2) A starts business with Rs. 3500 and after 5 months, B joins
further 8 years, how many times would he be of Sunil's age? with A as his partner. After a year, the profit is divided in the
a) 4 times b) 5 times c) 2 times ratio 2 : 3. What is B's contribution in the capital?
d) 3 times e) None of these a) Rs. 7500 b) Rs. 8000 c) Rs. 8500
d) Rs. 9000 e) None of these
3) In a regular week, there are 5 working days and for each day,
the working hours are 8. A man gets Rs. 2.40 per hour for 3) If Rs.8000 has been lent at 10% p.a. the interest being
regular work and Rs. 3.20 per hours for overtime. If he earns compounded annually, what is the interset for the fifth year?
Rs. 432 in 4 weeks, then how many hours does he work for? a) Rs.1171.28 b) Rs.1161.28 c) Rs.1141.28
A) 160 b) 175 c) 180 d) Rs.1151.28 e) None of these
d) 195 e) None of these
4) The simple interest and compound interest at a certain rate
4) Weight of the solid right circular cone of a certain material on a certain sum for 2 years are Rs.800 and Rs.960 respectively.
varies directly as the square of its radius when its height is Find the rate of interest as well as sum
constant and varies directly as its height when its radius is a) 30% p.a.; Rs.2000 b) 40% p.a.; Rs.2000
constant. The weight of one such cone 12kg, its radius is 2 cm c) 40% p.a.; Rs.1000 d) 45% p.a.; Rs.1000
and its height is 4 cm. Find the weight of another such cone e) None of these
whose radius is 4 cm and whose height is 3 cm.
a) 24kg b) 30kg c) 36kg 4-Average, Mixture & Alligation
d) 39kg e) None of these
1) The average of 4 consecutive even numbers is 103. What is
2-Percentage, Profit & Loss the product of the smallest and the largest number?
a) 10400 b) 10504 c) 10605
1) Two students Sharan and Karan took the test. Sharan got 65% d) 10600 e) None of these
of the maximum marks and Karan got 55% percent of the
maximum marks in the test. The difference between marks obtain 2) A can contains 200 litres of pure milk. 20 litre was taken out
by Sharan and Karan is 48. What are the maximum marks of the and replace with water. How many times should
test? this procedure be followed for the can to contain 145.8 litres of
a) 360 b) 400 c) 440 d) 480 pure milk?
a) 2 b) 3 c) 4 d) 5 e)None of these
2) There are 4 containers W, X, Y and Z, each of which can hold
a maximum quantity of 200 kg of a particular item. Container W 3) In what ratio, a liquid A of cost Rs.31 per litre should be
has 40% more than X, X has 40% more than Y and Y has 30% mixed with liquid B of cost Rs.36 per litre, so that cost of liquid
less than Z. If W has 102.9 kg of contents, then what percentage of mixture is Rs.32.25 per litre?
of full quantity did Z has? a) 2:1 b) 3:1 c) 3:2 d) 4:3 e) None of these
a) 37.5% b) 12.9% c) 45.8% d) 82.4%
4) A shopkeeper sells wheat partly at 4% profit & at 16% profit.
3) The length and the breadth of the rectangle are increase by How much quantity should be sold at 4% profit if he has 600kg
15% and 20% respectively. What is the percentage increase in of wheat and wants want to make an overall profit of 11%?
the area of rectangle? a) 250 kg b) 120 kg c) 350 kg
a) 38% b) 42% c) 46% d) 50% d) 520 kg e) None of these

4) Marked price of the article is Rs.100. It sold with first


discount of 10% and second discount was half of the previous
discount then find the selling price after the two successive
discounts.
a) Rs.87.5 b) Rs.90 c) Rs.85.5 d) Rs.85

For details on our programs: - Contact: +91 9930202329, +91, 8291924300 or visit www.campuscredentials.com (11)
5-Time and Work a) 4.12 minutes b) 5.28 minutes c) 3.92 minutes
d) 4.08 minutes e) None of these
1) A is twice as good a workman as B and working together they
finish a work in 14 days. In how many days can A alone finish 7-Numbers
the work?
a) 19 b) 17 c) 43 d) 21 e) 42 1) The HCF and LCM of two numbers are 13 and 455
respectively. If one number lies between 85 and 125, then that
2) Arun and Vinay can do a work in 12 days and 36 days number is :
respectively. If they work on alternate days, beginning with a) 99 b) 88 c) 91 d) 110 e) None of these
Arun, then in how many days will the work get completed?
a) 24 b) 22 c) 18 d) 20 e) 25 2) A rectangular courtyard 3.78 meters long 5.25 meters wide is
to be paved exactly with square tiles, all of the same size. what
3) 12 men or 16 women can do a work in 18 days, in how many is the minimum number of the tile which could be used for the
days can 9 men and 24 women do the work? purpose?
a) 12 b) 8 c) 10 d) 18 e) 14 a) 400 b) 450 c) 500
d) 600 e) None of these
6-Time, Speed and Distance
3) i) The sum of squares of two numbers is 80 and the square of
1) Had a person travelled 3 kmph faster he would have taken 2 their difference is 36. The product of the two numbers is.
a) 44 b) 22 c) 58 d) 116 e) None of these
hours less to cover a certain distance. Had he travelled 4 kmph
slower he would have taken 5 hours more to cover the same
distance. Find the distance. ii) The sum of the squares of the three numbers is 138. While
sum of their products taken two at a time is 131. Their sum is.
a)180 km b)120 km c)140 km
d)160 km e) None of these a) 20 b) 30 c) 40 d) 50 e) None of these

2) A train, 350-meter-long, crosses a man, running in the same 4) Find the smallest number which when divided by 7 leaves a
direction in 7 sec. The same train crosses a pole in 5 sec. What remainder of 6 and when divided by 11 leaves remainder of 8
a) 37 b) 43 c) 47 d) 41 e) None of these
will be the speed of man?
a) 14 m/s b) 36m/s c) 25m/s
5) Find the largest number with which when 906 and 650 are
d) 20m/s e) None of these
divided they leave respective remainders of 3 and 5.
3) Two bombs were exploded at a place P with a time interval a) 129 b) 127 c) 131 d) 133 e) None of these
of 40 minutes. A person moving away from P heard the first
explosion at a point A and second explosion when he was at a 8-Geometry & Mensuration
point B. if he heard the explosions at an interval of 41 minutes
and the speed of the sound is 331m/s, what is the distance 1) Find the perimeter of the semicircle whose radius is 35 cm.
between A and B? a) 110 cm b) 150 cm c) 180 cm
a) 15240 m b) 18660 m c) 19860 m d) 20356 m d) 220 cm e) None of these
𝟏
4) A motorboat can cover 10 km in 1 hour in still water. And it
𝟑
2) A swimming pool 100 m long and 40 m wide is 1 m deep at
takes twice as much as time to cover up than as to cover down
the shallow end and 5m at deep end. Find the volume of water
the same distance in running water. The speed of the current is:
𝟒 𝟏 contain in the pool?
a) 3 km/hr b) 2 km/hr c) 4 km/hr a) 10000 cu.m. b) 12000 cu.m c) 15000 cu.m
𝟗 𝟑
d) 3 km/hr e) None of these d) 12500 cu.m e) 13500 cu.m

5) Two toy trains start at same time from the same point on a 3) A cylindrical vessel of radius 21 m and height 5 m is 60%
circular track of circumference 1452 meters and run in opposite filled with water. How many pebbles of diameter 2 m are
direction at 9 km/hr and 7.5 km/hr, respectively. They will meet approximately required to fill the vessel?
in: a) 540 b) 340 c) 662 d) 750

For details on our programs: - Contact: +91 9930202329, +91, 8291924300 or visit www.campuscredentials.com (12)
New values 23(15% 12(20% 276
Quantitative Aptitude Handout increase) increase)
Homework - Solutions
% increase in the area of Rectangle is 38%.
1-Equations, Ratio, Proportion &
4) Marked price = Rs.100
Variation 1st discount = Rs.10(10% of the marked price)
2nd discount = Rs.5(Half of the first discount i.e. Rs.10)
1) Suppose one piece is x then other piece is (4/7)x So, at the end marked price becomes Rs. 85
x+(4/7)x=77 then x=49
Longer piece is 49m and shorter piece is 28. 5) A vendor bought toffees at 6 for a rupee.
(3/14) of 49 = 10.5 Cost price of 1 toffee is Rs.1/6.
% profit = 20%
2) If sunil's age is x then fathers age is x+3x=4x. 𝑺𝒆𝒍𝒍𝒊𝒏𝒈 𝒑𝒓𝒊𝒄𝒆−𝑪𝒐𝒔𝒕 𝒑𝒓𝒊𝒄𝒆
% profit = ×100
After 8 years, 𝑪𝒐𝒔𝒕 𝒑𝒓𝒊𝒄𝒆
(4x+8)= 2(1/2)(x+8)
By solving this x=8. From here we get the selling price of one toffee as 1/5.
Then after further 8 years father would be twice the age of So, in 1 rupee we can sell 5 toffees.
sunil.
3-Partnership, SI & CI
3) Without doing over time he earn 384
So total he earns 432 - 384 = 48 by doing overtime. 1) the total time period is 1 year. Amol started the business
So number of hours for which he did the overtime = 48/3.20 and Ashok joined him 5 months after. So, Amol invested the
=15 capital for 12 months and Ashok invested the capital for 7
Regular working hours = 60 hrs months.
over time =15 hrs Partners Amount of Time Ratio of
Total number of hours = 75 hours investment period profit
Amol 2000 12 16
4) W α r2 and W α h
Ashok 4500 7 21
By joint variation W α r2.h
Soppose total profit at the end is 100. Then A receives 10% as
Equation becomes
𝑾𝟏 𝒓𝟐𝟏.𝒉𝟏
a commission then remaining 90 rupees get distributed among
= Amol and Ashok in the ratio 16:15.
𝑾𝟐 𝒓𝟐𝟐.𝒉𝟐
𝟏𝟔
So, amount received by Amol becomes ×90 + amount
𝟑𝟕
W1= 12 W2 = ? 𝟏𝟔
r21 = 2 r22= 4 received by Amol as a commission(10) and that ( ×90+10)
𝟑𝟕
h1 = 4 h2 = 3 corresponds to 45250(Amount received by Amol with 10%
Put values in above equation and get the value of W2. commission) then amount received by ashok when we assumed
𝟐𝟏
as 100 is ×90 that corresponds to what?
𝟑𝟕
2-Percentage, Profit & Loss If we solve this them we will get answer as 47250.

1) Maximum marks of the exam is suppose M 2) A starts business with Rs. 3500 and after 5 months, B joins
Then marks got by Sharan and Karan becomes 65% of M and with A as his partner by investing x rupees suppose. Then A
55% of M. Difference between their marks is 48. invested for 12 months and B invested for 7 months and at the
65% of M - 55% of M = 48 end of the time period they share the profit in the ratio of 2:3.
,𝟑𝟓𝟎𝟎×𝟏𝟐 𝟐
Solving we will get value of Maximum marks as 480. So, = Solve this and get the value of A that is amount
𝒙 ×𝟕 𝟑
invested by B in the business.
2) If content in the container Z is 100 kg. Value of x will be 9000.
Then content in the container Y is 70 kg.
Content in the container X is 98 kg. 3) the interest for the fifth year = Amount after 5 years -
Content in the container W is 137.5 kg. Amount after 4 years.
But actual content in the container W is 102.9 kg that Rate of interest (r) = 10% p.a.
corresponds to 137.5. Principal (P) = 8000
Then actual content in the container Z that corresponds to 𝟏𝟎 5 𝟏𝟎 4
100. So, Interest for 5th year = 8000(1+ ) - 8000(1+ )
𝟏𝟎𝟎 𝟏𝟎𝟎
So, actual contain in the container Z will be 75 Kg and that is If we solve above equation the we will get answer as 1171.28
37.5% of its maximum capacity.
OR
3) Length×Breadth=Area of Rectangle
Length Breadth Area C.I. For 1st year = 800
Assumed 20 10 200 C.I. For 2nd year = 880
values C.I. For 3rd year = 968

For details on our programs: - Contact: +91 9930202329, +91, 8291924300 or visit www.campuscredentials.com (13)
C.I. For 4th year = 1064.8
C.I. For 5th year = 1171.28

4) Simple interest is same every year and Simple interest and


compound is same for first year.
So,
Simple Interest Compound Interest
1st Year 400 400
2nd Year 400 560
Total Interest 800 960
The % increase in the compound interest in between any 2 3)
consecutive year is same as the rate of interest. So, liquid A and Liquid B are mixed in the ratio of 3.75 to 1.25
So, % increase in the compound interest from 1st to 2nd year i.e. 3:1 so that cost of mixture becomes Rs.32.25 per litre
is 40%. Hence rate of interest is 40%.
Rate of interest is interest for one year on Principal. 4) Suppose shopkeeper sells x kg at 4% profit and y kg at 16
So, 40% of Principal = 400 % profit then total quantity after mixing these 2 types wheat
Hence Principal is 1000. will be 600 kg.
So, x + y = 600

4-Average, Mixture & Alligation


1) If numbers are in the Arithmetic progression and even
number of terms are there then average of all such terms will
be the average of first and last term or average of second and
second last term or average of third and third last term and so
on.
The average of 4 consecutive even numbers is 103. So average
of first and last or average of second and third numbers is
103. Then2 consecutive even integers are there whose average
is 103 The numbers are 102 and 104. So, second and third
numbers are 102 and 104.
All 4 numbers are 100, 102, 104 and 106.
Multiplication of first and last number is 10600. Ratio of x and y must be 5:7 and x+y=600.
So, 250 kg must be sell at 4 % profit and 350 kg must be sell at
2) P = Initial quantity of Pure liquid. 16% profit.
Q = Quantity replaced every time.
n = Number of times procedure is repeated. 5-Time and Work
Then after n number of such operations,
𝑷−𝑸 ^n
a) Concentration of pure liquid in the final solution = [ ] 1) A is twice as good a workman as B. Means if B completes
𝑷
b) % Concentration of pure liquid in the final solution = the work in 2x number of days then A completed the same
𝑷−𝑸 ^n amount of work in x number of days.
[ ] ×100
𝑷 Work done per day
𝑷−𝑸 ^n
c) Quantity of pure liquid in the final solution = [ ] ×P A = x days 1/x
𝑷
B = 2x days 1/2x
Now in Question we have to find out the value of n-1(because A and B = 14 days 1/x+1/2x = 1/14
we have to find out number of times such procedure is 1/x+1/2x=1/14
followed, its already perform one time at the start). If we solve above equation then value of x will be 21. So A
Q = 20 litres alone can finish the work in 21 days.
P = 200 litres
Put these values in formula (Formula c) 2)
Then we will get equation Work done per day
𝟐𝟎𝟎−𝟐𝟎 ^n Arun = 12 days 1/12
[ ] × 200= 145.8
𝟐𝟎𝟎
Vinay = 36 days 1/36
By solving above equation we get the value of n as 4.
So, 3 more times same procedure must be followed.
If they work on alternate days, beginning with Arun, then on
odd number of days Arun will work and even number of days
Vinay will work. So in a block of two days work completed by
𝟏 𝟏 𝟏
them = + = .
𝟏𝟐 𝟑𝟔 𝟗
𝟏
So, in 2 days they complete th work so, number of days
𝟗
required to complete the total work would be 18.

For details on our programs: - Contact: +91 9930202329, +91, 8291924300 or visit www.campuscredentials.com (14)
𝑴𝟏.𝑻𝟏 𝑴𝟐.𝑻𝟐 𝑴𝟑.𝑻𝟑 point B. So, time taken by the sound of bomb to travel from
3) = ==
𝑾𝟏 𝑾𝟐 𝑾𝟑
point A to B is 1 minute.
M = Number of men to complete the work
Distance between point A and B = Speed of Sound × 1
T = Time required to complete the work
minute(60 seconds)
W = Amount of work
= 331 × 60 = 19860 metres.
M1 = 12m M2 = 16w M3 = 𝟏
9m+24w 4) A motorboat can cover 10 km in 1 hour in still water.
𝟑
T1 = 18 days T2 = 18 days T3 = Speed of boat in still water is Sb = 31/3 kmph.
? It takes twice as much as time to cover up than as to cover
W1 = X W2 = X W3 = X down the same distance in running water.
(Work is same in all the case.) Now distance is constant.
𝟏𝟐𝒎.𝟏𝟖 𝟏𝟔𝒘.𝟏𝟖 𝟗𝒎+𝟐𝟒𝒘
= = TU = Time taken to cover the distance Upstream
𝑿 𝑿 𝑿
TD = Time taken to cover the distance Downstream.
By equating 1st and 2nd term we get the relation between
TU = 2.TD
number of men and number of women. 𝑻𝑼 𝑺𝑫 𝟐
3m = 4w So, = =
𝑻𝑫 𝑺𝑼 𝟏
After that either equate 1st and 3nd term or 2nd and 3rd term Where, SD = Downstream speed = Sb+Ss
to calculate value of T3. SU = Upstream speed = Sb-Ss
If we solve then we get value of T3 as 8 days. Sb = 31/3 kmph
Put these values in above equation and find out value of S s we
𝟒
6-Time, Speed and Distance will get it as 3 .
𝟗

1) Usual time taken and usual speed of a person is s kmph and 5) Two toy trains start at same time from the same point on a
t hours. circular track of circumference 1452 meters and run in
Now in the question distance travelled by a person is same in opposite direction at 9 km/hr and 7.5 km/hr, respectively.
all the cases. So, time taken by 2 toy trains to meet for the first time =
Had a person travelled 3 kmph faster he would have taken 2 𝑫𝒊𝒔𝒕𝒂𝒏𝒄𝒆

hours less to cover a certain distance. 𝑺𝒑𝒆𝒆𝒅

Equation becomes, Distance = 1452 meters


𝟓
s×t = (s+3)(t-2)____(1) Speed = (9+7.5) × m/s
𝟏𝟖
Had he travelled 4 kmph slower he would have taken 5 hours If we put these values in above equation then we get the value
more to cover the same distance. of time taken as 5.28 minutes.
Equation becomes,
s×t = (s-4)(t+5)____(2)
7-Numbers
By solving 1 and 2 simultaneous equations we will get the
value of s and t as 12 kmph and 10 hours.
1) If A and B are two integers then,
So, distance = s × t = 12 × 10 = 120 km
A × B = LCM(A,B) × HCF(A,B)
A × B = 13×455
2) Suppose speed of man = x m/s
If we factorise this then it becomes 13×5×7×13
And speed of the train is = s m/s
One number lies between 85 and 125. So that one number
A train, 350-meter-long, crosses a man, running in the same
must be 91 and other number we get 65.
direction in 7 sec.
In this case 2 objects are Train and man.
2) A rectangular courtyard 3.78 meters long 5.25 meters wide
So,
is to be paved exactly with square tiles, all of the same size. In
Distance = Length of train = 350 meter
this case dimension of square tile must be the factor of the
Time = 7 seconds
𝑫 𝟑𝟓𝟎 length and width of courtyard. But minimum number of such
Speed = s - x = = = 50 m/s tiles are required then we have to find the highest factor of
𝑻 𝟕
S - X =50____(1) length(378 cm) and width(525 cm) of the rectangular
The same train crosses a pole in 5 sec. courtyard.
In this case 2 objects are Train and pole. So, HCF(378,525) = 21cm
So, Square tile must me of the size 21cm × 21cm.
Distance = Length of train = 350 meter 𝑨𝒓𝒆𝒂 𝒐𝒇 𝑪𝒐𝒖𝒓𝒕𝒚𝒂𝒓𝒅
Minimum number of such tiles =
Time = 5 seconds 𝑫𝒊𝒂𝒎𝒆𝒏𝒕𝒊𝒐𝒏 𝒐𝒇 𝒔𝒊𝒏𝒈𝒍𝒆 𝒕𝒊𝒍𝒆
𝟑𝟕𝟖×𝟓𝟐𝟓
𝑫 𝟑𝟓𝟎 = = 450
Speed = s = = = 70 m/s 𝟐𝟏×𝟐𝟏
𝑻 𝟓
Put value of s in the 1st equation then we will get speed of So, 450 tiles are required to paved the area with square tile.
man = x = 20m/s
3) i) The sum of squares of two numbers is 80 and the square
3) Two bombs were exploded at a place P with a time interval of their difference is 36. The product of the two numbers is.
of 40 minutes. A person moving away from P heard the first Let the 2 numbers are a and b,
explosion at a point A and second explosion when he was at a (a2+b2) = 80
(a-b)2 = 36
So, (a-b)2 = (a2+b2) - 2ab

For details on our programs: - Contact: +91 9930202329, +91, 8291924300 or visit www.campuscredentials.com (15)
Putting the values above we get the product of two numbers as 8-Geometry & Mensuration
22
1) Perimeter of semi circle = Half of the perimeter of circle +
ii) The sum of the squares of the three numbers is 138. While
Diameter of circle
sum of their products taken two at a time is 131.
= 𝝅r. + 2r
Let the numbers are a, b and c.
= 𝝅(35) + 2×35
(a2+b2+c2) = 138
= 180 cm
ab+bc+ca = 131
we have to find out a+b+c = ?
2)
(a+b+c)2 = (a2+b2+c2) + 2(ab+bc+ca)
Putting the values in above equation we get value for a+b+c
= 20

4) Find the smallest number which when divided by 7 leaves a


remainder of 6 and when divided by 11 leaves remainder of 8.
Solution:
Take the larger of the two given divisors i.e. 11 in this case.
The required number when divided by 11 gives a reminder of
8. We know that a number when divided by 11 giving
remainder of 8 is of the form 11k+8, which means we are Volume of pool = Area of trapezium with parallel sides 5 and
looking for a number of the form 11k+8. 1 meter × Width of pool
𝟏
= × (1+5) × 100 × 40
Since the same remainder, when divided by 7 gives a 𝟐
After solving we will get answer as = 12000 cu.m
remainder of 6, this number 7k+6 when divided by 7 gives a
remainder of 6. We know that if there is a remainder in a
division, by subtracting the remainder from the given number,
the resulting number will then be exactly divisible by the
divisor. This means if 6 is subtracted from 11k+8, the
resulting number, i.e.11k+2 will be exactly divisible by 7. We
should now give value of 0, 1, 2, 3... to k and find out for what
value of k, 11k+2 will be divisible by 7.
3)
The smallest value of k which satisfy the above condition, we A cylindrical vessel of radius 21 m and height 5 m is 60%
notice, is 3 and hence k=3 will give us a number that we are filled with water. So, 40% of the cylinder is vacant.
looking for. Since the number, we said, is 11k+8 number is Suppose n numbers of pebbles are required to fill the vessel
11× 𝟑+8 i.e 41. So 41 is the smallest number which satisfies completely then,
the two given conditions. Number of pebbles required to fill this vessel completely =
40% of the volume of the cylinder.
5) Find the largest number with which when 906 and 650 are n × volume of single pebble = 40% of the volume of the
divided they leave respective remainders of 3 and 5. cylinder.
The largest number with which the number P, Q, or R are 𝟒 𝟒𝟎
n × × 𝝅 × r3 = × 𝝅 × r2 × 𝒉
divided giving remainders of S, T and U respectively will be 𝟑 𝟏𝟎𝟎
𝟒 𝟒𝟎
the, n× ×𝝅× (1)3 = × 𝝅 × (21)2 × (𝟓)
𝟑 𝟏𝟎𝟎
HCF of the three numbers of the form (P-S), (Q-T) and (R-U). n = 661.5
So, answer is, HCF[(906 - 3),(650 - 5)] = 129 So, approximately 662 pebbles are required to fill the vessel
completely.

For details on our programs: - Contact: +91 9930202329, +91, 8291924300 or visit www.campuscredentials.com (16)
Quantitative Aptitude
Chapter 01 - Number System
Number System & Simplification
Numbers can be classified in various categories as follows

REAL NUMBERS : An integer is any number of the set {…., -3, -2, -1, 0, 1, 2, 3, ….}
Corresponding to every natural number, there is a ‘negative’
Numbers that can be represented on a number line are called number. These negative numbers and natural numbers form the
Real Numbers. set of integers along with 0. A set of integers is denoted by ‘I’.
IMAGINARY (COMPLEX) NUMBERS : The order of relation in the set I. … < -3 < -2 < -1 < 0 < 1 < 2
A number of the form k.i. where k is a real number, k  0 and i < 3 < ……
= √−𝟏 is called an imaginary number. A number of the form a
+ bi, where a, b are real numbers is a complex number. NATURAL NUMBERS :
Note : All the non negative counting numbers are known as natural
(i) i = √−𝟏 numbers.
(ii) A number a + bi ; if a = 0 the number is a purely imaginary e.g. 1, 2, 3, 4, 5,…….. is a set of natural numbers. 0 is not
number, if b = 0, the number is a purely real number included in the set of natural numbers.
(iii) Any real number can be written in the form of a complex
number. WHOLE NUMBERS :
All the Natural numbers including 0 forms the set known as
CONJUGATE OF A COMPLEX NUMBER : whole numbers.
It is obtained by changing the sign of the imaginary part. e.g. 0, 1, 2, 3, 4, 5 …….. is a set of whole numbers.
e.g., conjugate of (2+3i) is (2 – 3i) Here, we can say that all natural numbers are also whole
RATIONAL NUMBER : numbers.
Rational numbers can be expressed in the form of p/q, where p
and q are integers and q  0. Rational numbers can be both EVEN NUMBERS :
positive or negative. All integers that are divisible by 2 are known as even numbers.
e.g. -2, -4, 12,16 ….. etc are even numbers.

e.g. ODD NUMBERS :


Every rational number when expressed in decimal form is All integers that are not divisible by 2 are known as odd
expressible in either terminating decimals or repeating numbers.
decimals. e.g. -3,1,-5,7,-11 etc are odd numbers.

FRACTION :
The fraction is a part of an integer. e.g. p/q, The denominator of
a fraction cannot be Zero as division by zero is not defined.
IRRATIONAL NUMBERS :
Kinds of fractions :
Numbers those when expressed in decimal form are neither
Proper Fractions : A proper fraction is a fraction whose
terminating nor repeating are known as irrational numbers.
denominator is greater than its numerator.
𝒑 𝟐
i.e. such that q > p. e.g. is a proper fraction.
𝒒 𝟑
Improper Fractions :
INTEGERS :

For details on our programs: - Contact: +91 9930202329, +91, 8291924300 or visit www.campuscredentials.com (17)
An improper fraction is a fraction whose denominator is less
𝒑 𝟑 (7) Divisibility by 11 : A number is exactly divisible by 11. If the
than its numerator. i.e. such that q < p. e.g. is a proper
𝒒 𝟐
difference between the sum of digits at odd places and the sum
fraction.
of digits at even places is either 0 or divisible by 11.
Mixed Fraction :
e.g. 121
It is combination of two parts, an integer and a fraction.
𝟏 𝟏
Sum of digits at odd places = 1 + 1 = 2
e.g., 𝟏 is a mixed fraction. 1 is an integer and is the fraction. Sum of digits at even places = 2
𝟐 𝟐
A mixed fraction can be expressed as an improper fraction and 2–2=0
vice-versa.  121 is divisible by 11.
𝟏 𝟑 𝟑 𝟏 𝟏 e.g. 563574
𝟏 = 𝒂𝒏𝒅 = 𝟏 + = 𝟏
𝟐 𝟐 𝟐 𝟐 𝟐 Sum of digits at odd places = 5 + 3 + 7 = 15
Sum of digits at even places = 4+5+6= 15
PRIME NUMBERS : 15 – 15 = 0
Those numbers which are greater than 1 and having exactly two  563574 is divisible by 11.
factors, i.e. 1 and the number itself, are known as prime
numbers. FACTOR AND MULTIPLES :
e.g. 2, 3, 5, 7, 11, 13, ……. etc are prime numbers. A number is a factor of another number if it divides that number
Note : exactly i.e. A is a factor of B if
(i) 2 is the smallest prime number and it is the only even prime A exactly divides B.
number. e. g. 30 is exactly divisible by 5.
(ii) If a number has no factor equal to or less than its square Hence, 5 is a factor of 30.
root, then that number must be prime. If A is a factor of B, then B is called a multiple of A.
(iii) 1 is neither prime nor composite e.g. 30 is multiple of 5.

COMPOSITE NUMBERS : SOME IMPORTANT FORMULAE :


Natural numbers greater than 1 that are not prime, are known (a + b)² = a² + b² + 2ab
as composite numbers. (a - b)² = a² + b² - 2ab
e.g. 4, 6, 9, 10, 12 …….. etc. are composite numbers. (a + b)² - (a - b)² = 4ab
(a + b)² + (a - b)² = 2 (a² + b²)
RELATIVELY PRIME NUMBERS (a² - b²) = (a + b) (a - b)
Two positive integers are said to be relatively prime to each (a³ + b³) = (a + b) (a² - ab + b²)
other if their highest common factor is 1. (a³ - b³) = (a - b) (a² + ab + b²)
e.g. 14 and 15 are relatively prime numbers. a³ + b³ + c³ - 3abc = (a + b + c)  (a² + b² + c² - ac – bc - ab)
If a³ + b³ + c³ - 3abc = 0 ; then either a + b + c = 0.
DIVISIBILITY RULES :
(1) Divisibility by 2 : A number is exactly divisible by 2, if the Ex. 1 : Express ̅̅̅̅̅̅̅̅̅
𝟎. 𝟑𝟒𝟓 as a fraction.
last digit of that number is 0, 2, 4, 6 or 8. ̅̅̅̅̅̅̅̅̅ ̅̅̅̅̅̅ = 1000x- x
Sol. : Let x = 𝟎. 𝟑𝟒𝟓 = 1000x = 345.𝟑𝟒𝟓
e.g. 16,18,24,36,42….. etc. are exactly divisible by 2. ̅̅̅̅̅̅ ̅̅̅̅̅̅̅̅̅
=345. 𝟑𝟒𝟓 - 𝟎. 𝟑𝟒𝟓
= 999x = 345 = x = 345/999
(2) Divisibility by 3 : A number is exactly divisible by 3, if the
sum of all its digit is divisible by 3.
Ex. 2 : Find the total number of factors of 462.
e.g. 462  4+6+2= 12
Sol. : The factorized form of 462 is (2×3×7×11)
12 is divisible by 3.
So the total number of factors is (1 + 1) (1 + 1) (1 + 1) (1 + 1)
462 is divisible by 3.
= 2 × 2 × 2 × 2 = 16
(3) Divisibility by 4 : A number is exactly divisible by 4, if the
Ex. 3 : What is remainder if 723 is divided by 6 ?
last two digits of that number are either 00 or divisible by 4.
e.g. 1728. Here, last two digits are 28. 28 is divisible by 4.
Hence, 1728 is divisible by 4.
Sol. :
(4) Divisibility by 5 : A number is exactly divisible by 5, if its 7 divided by 6 leaves remainder 1.
last digit is either 0 or 5. Thus, 723 when divided by 6 will leave remainder 1×1×1… (23
times) = 1
(5) Divisibility by 6 : A number is exactly divisible by 6, if it is
divisible by 2 as well as 3. Ex. 4 : Evaluate (52)² + (48)² + 2 × 52 × 48
e.g. 54 is divisible by 2 as well as by 3. Hence, 54 is divisible by Sol. : (a² + b² + 2ab) = (a + b)²
6. (52 + 48)² = 100² = 10000
(6) Divisibility by 8 : A number is exactly divisible by 8, If the Ex. 5 : Find the unit digit in the product 269 × 541 × 366 × 345
number formed by its last three digits of that number are either × 38
000 divisible by 8. Sol. : Unit digit in the product = 9 × 1 × 6 × 5 × 8 = 0
e.g. 12520 . here, last three digits are 520. 520 is divisible by 8.
Hence, 12520 is divisible by 8. Practice Set 1:

For details on our programs: - Contact: +91 9930202329, +91, 8291924300 or visit www.campuscredentials.com (18)
1) (2222.5 – 1680 * 0.75)*1.2 = ?
A) 1015 B) 1255 C) 1115 15) A number is divided by 2, 3, 4, 5 or 6, reminder in each case
D) 1055 E) 1155 is one. But the number is exactly divisible by 7. The number lies
between 250 and 350, the sum of digits of the number will be
2) [18 * 15 – 20] / [(3.2 + 9.4) – 7.6] = ? A) 4 B) 7 C) 6 D) 10 E) Cannot be determined
A) 30 B) 40 C) 50
D) 70 E) 60 16) Sum of eight consecutive odd numbers is 656. Average of
four consecutive even numbers is 87. What is the sum of the
3) The sum of two number is 24.The greatest product of these largest even number and largest odd number?
two numbers can be A) 171 B) 191 C) 101
A) 124 B) 144 C) 164 D) 140 D) 181 E) 179

4) (a-b) = 1, (b-c) = 2 and (c-a) = 3 then the value of 17) What is the remainder when 7100 is divided by 4?
(a3+b3+c3-3abc)/(a+b+c) is A) 1 B) 2 C) 0 D) 3 E) None of these
A) 5 B) 6 C) 7 D) 8
18) Remainder when (41 × 42) is divided by 43?
5) If a = -2 and b = -5 then find the value of a3 + b3 – a2 + b2 + A) 1 B) 2 C) 3 D) 0 E) None of these
2a – 3b
A) -98 B) -100 C) 101 D) -101 19) Find the remainder when 51203 is divided by 7?
A) 4 B) 5 C) 6 D) 3 E) None of these
6) Which of the following number is divisible by 11 ?
A) 29435417 B) 29435477 20) Find the remainder when 21875 is divided by 17?
C) 57463822 D) 57648317 A) 14 B) 15 C) 16 D) 13 E) None of these

7) What is the no of zeroes at the end of the product of the no 21) How many factors of 1080 are perfect squares?
from1 to 100 ? A) 4 B) 6 C) 8 D) 5 E) None of these
A) 20 B) 22 C) 24 D) 25
22) How many factors of 25 * 36 * 52 are perfect squares?
8) x+(1/x) = 2, then find the value of x10 + (-1/x)10 A) 18 B) 24 C) 36 D) 8 E) None of these
A) 0 B) 1 C) 2 D) 3
23) How many factors of 24 * 53 * 74 are odd numbers?
9) The average of 7 consecutive number is n, if the next 2 A) 100 B) 99 C) 20
number also included then the new average will be increased by D) 24 E) None of these
?
A) 0 B) 1 C) 2 D) 3 24) How many factors of the number 28 * 36 * 54 * 105 are
multiples of 120?
10) The number of prime factors of 510510 A) 540 B) 660 C) 594
A) 4 B) 5 C) 6 D) 7 D) 792 E) None of these

11) How many numbers are there up to 1000 which are 25) Number N = 26 * 55 * 76 * 107; how many factors of N are
divisible by 4, 6 and 8 together? even numbers?
A) 39 B) 40 C) 41 A) 1183 B) 1200 C) 1050
D) 42 E) None of these D) 840 E) None of these

12) In the examination a candidate must get 3/8th marks to pass, 26) Numbers A, B, C and D have 16, 28, 30 and 27 factors.
out of total marks. Shyam appeared in the exam and got 300 Which of these could be a perfect cube?
marks and still failed by 36 marks. The maximum mark is A) A and B B) A and B C) A, B and C
A) 856 B) 866 C) 876 D) A and B E) None of these
D) 886 E) 896
27) Six bells commence tolling together and toll at intervals of
13) Two different numbers are divided by the same divisor and 2, 4, 6, 8, 10 and 12 seconds respectively. In 30 minutes, how
left remainder 11 and 17 respectively and when their sum was many times do they toll together?
divided by the same divisor, remainder was 4. What is the A) 8 B) 11 C) 13 D) 16 E) None of these
divisor?
A) 20 B) 24 C) 25 28) The least multiple of 7, which leaves a remainder of 4, when
D) 28 E) None of these divided by 6, 9, 15 and 18 is:
A) 68 B) 98 C) 180 D) 364 E) None of these
14) If the places of last two-digits of a three digit number are
interchanged, a new number greater than the original number 29) The least number, which when divided by 48, 60, 72, 108
by 36 is obtained. What is the difference between the last two and 140 leaves 38, 50, 62, 98 and 130 as remainders
digits of that number? respectively, is:
A) 2 B) 3 C) 4 D) 7 E) None of these A) 11115 B) 15110 C) 15130

For details on our programs: - Contact: +91 9930202329, +91, 8291924300 or visit www.campuscredentials.com (19)
D) 15310 E) None of these A) 5184 B) 5060 C) 5148
D) 5084 E) None of these
30) The L.C.M. of two numbers is 48. The numbers are in the
ratio 2 : 3. Then sum of the number is: 11) 72519 x 9999 = ?
A) 30 B) 22 C) 40 D) 60 E) None of these A) 725117481 B) 674217481 C) 685126481
D) 696217481 E) None of these
31) The least number, which when divided by 12, 15, 20 and 54
leaves in each case a remainder of 8 is: 12) The smallest 3 digit prime number is:
A) 534 B) 486 C) 544 A) 101 B) 103 C) 109 D) 113
D) 548 E) None of these
13) (?) - 19657 - 33994 = 9999
32) The product of two numbers is 2028 and their H.C.F. is 13. A) 63650 B) 53760 C) 59640 D) 61560
The number of such pairs is: E) None of these
A) 1 B) 2 C) 3 D) 5 E) None of these
14) The sum of first 45 natural numbers is:
33) The least number which should be added to 2497 so that the A) 1035 B) 1280 C) 2070 D) 2140
sum is exactly divisible by 5, 6, 4 and 3 is:
A) 10 B) 14 C) 23 D) 30 E) None of these 15) (?) + 3699 + 1985 - 2047 = 31111
A) 34748 B) 27474 C) 30154 D) 27574
34) HCF of 4/3, 8/6, 36/63 and 20/42 E) None of these
A) 4/126 B) 4/8 C) 4/36
D) 4/42 E) None of these 16) Which of the following numbers is divisible by 3, 7, 9 And
11?
35) A gardener had a number of shrubs to plant in rows. At first A) 639 B) 2079 C) 3791 D) 37911
he tried to plant 8,then 12 and then 16 in a row but he had E) None of these
always 3 shrubs left with him. On trying 7 he had none left.
Find the total number of shrubs. 17) 39798 + 3798 + 378 =?
A) 154 B) 147 C) 137 A) 43576 B) 43974 C) 43984 D) 49532
D) 150 E) None of these E) None of these

Practice Set 2: 18) Which of the following numbers is exactly divisible by 99?
1) 186 x 186 + 159 x 159 - 2 x 186 x 159 =? A) 114345 B) 913464 C) 135792
A) 329 B) 700 C) 729 D) 848 D) 3572404 E) None of these

2) If one-third of one-fourth of a number is 15, then three-tenth 19) 8756 x 99999 =?


of that number is: A) 796491244 B) 875591244 C) 815491244
A) 35 B) 36 C) 45 D) 54 D) 88324284 E) None of these

3) Which one of the following is not a prime number? 20) 469157 x 9999 = ?
A) 31 B) 61 C) 71 D) 91 A) 4586970843 B) 4686970743 C) 4691100843
D) 484649125 E) None of these
4) (112 x 54) =?
A) 67000 B) 70000 C) 76500 D) 77200 21) 935421 x 625 = ?
A) 575648125 B) 584638125 C) 584649125
5) 1397 x 1397 =? D) 575628125 E) None of these
A) 1951609 B) 1981709 C) 18362619
D) 2031719 E) None of these 22) Which of the following is always odd?
A) Sum of two odd numbers
6) (935421 x 625) = ? B) Product of two odd numbers
A) 575648125 B) 584638125 C) 584649125 C) Difference of two odd numbers
D) 585628125 D) Sum of two even numbers
E) None of these
7) Which of the following is a prime number?
A) 33 B) 81 C) 93 D) 97 23) Find the number which is nearest to 457 And is exactly
divisible by 11?
8) 5358 x 51 = ? A) 450 B) 451 C) 460
A) 273258 B) 273268 C) 273348 D) 273358 D) 462 E) None of these

9) The sum of first five prime numbers is: 24) The smallest three-digit prime number is:
A) 11 B) 18 C) 26 D) 28 A) 104 B) 103 C) 107
D) 100 E) None of these
10) (12)3 x 64 ÷ 432 = ?

For details on our programs: - Contact: +91 9930202329, +91, 8291924300 or visit www.campuscredentials.com (20)
25) 1399 x 1399 = ? Chapter 02: Percentages
A) 1687401 B) 1901541 C) 1943211
D) 1957201 E) None of these The term percent means “divided by hundred”. The word
percent come from “Per Centum” or “Per hundred”. The word
26) Find the units digit of the expression centum means hundred X percent means x / 100
111+122+133+144+155+166 X percent is written as x%
A) 1 B) 9 C) 7 D) 0 E) 8
CONVERTING A DECIMAL INTO PERCENTAGE :
27) The last digit of the number obtained by multiplying the To convert a decimal into percent, Move the decimal point two
number 91*92*93*94*95*96*97*98*99 will be places to the right and add the symbol %, to indicate percent.
A) 0 B) 9 C) 7 D) 2 E) 8 e.g. 0.45 = 45%
0.64 = 64%
28) What is the remainder, when 2256 is divided by 17? 0.586 = 58.6%
A) 1 B) 16 C) 14 D) 10 E) None of these 0.003 = 0.3%

29) Three times the first of three consecutive odd integers is 3 CONVERTING A FRACTION INTO PERCENTAGE :
more than twice the third. The third integer is: To convert a fraction into percent, Convert the given fraction
A) 9 B) 11 C) 13 D) 15 into its, decimal then convert the decimal into percent by
shifting the decimal point to two places on the right and adding
30) A two-digit number is such that the product of the digits is % symbol.
8. When 18 is added to the number, then the digits are e.g.
reversed. The number is: 𝟏
= 0.25 = 25%,
A) 18 B) 24 C) 42 D) 81 𝟒
𝟐
= 0.6666….. = 66.66%,
𝟑
𝟒
= 0.8 = 80%
𝟓

CONVERTING A PERCENTAGE INTO DECIMAL :


To convert a percent to a decimal, drop the % symbol. Then
move the decimal point two places to the left.
Add. if necessary,
e.g.
50% = 0.5, 16.5% = 0.165,
3% = 0.03, 47.53% = 0.4753

CONVERTING A PERCENTAGE INTO FRACTION :


To convert a percent to a fraction, drop the % symbol, write the
number over 100 to form a fraction. Reduce to get the simplest
fraction.

e.g.

PERCENT CHANGE (INCREASE AND DECREASE) :


Very often, we need to find the percent of increase or decrease
in a given quantity. To find this, calculate the change (increase
or decrease) and divide it by the original (initial)amount.
THIS CAN BE REPRESENTED AS.

SUCCESSIVE PERCENTAGE CHANGE :

For details on our programs: - Contact: +91 9930202329, +91, 8291924300 or visit www.campuscredentials.com (21)
If a no. is changed (increase / decrease) by x% and in the Practice Set 1
second step. This changed no. is again changed
(increase/decrease) by y% then, 1) A batsman scored 110 runs which included 3 boundaries and
Net % change = 8 sixes. What percent of his total score did he make by running
between the wickets?
𝟓 𝟔
A) 45% B) 45 % C) 54 % D) 55%
𝟏𝟏 𝟏𝟏
If x and y shows decrease then put a (-ve) sign before x and y,
otherwise (+ve) sign. 2) Find the missing figures: ?% of 25 = 2.125
A) 7.5 B) 8.5 C) 10.5 D) 11.2
SOME IMPORTANT POINTS : E) None of these
• If A is R%more than B, than B is less than A by =
3) 218% of 1674 = ? x 1800
A) 0.5 B) 4 C) 6 D) None of these

4) √𝟕𝟖𝟒 + ? = 78% of 500 :


• If A is R% less than B, then B is more than A by =
A) 342 B) 352 C) 362 D) 372

5) In a competitive examination in State A, 6% candidates got


SOLVED EXAMPLES selected from the total appeared candidates. State B had an
equal number of candidates appeared and 7% candidates got
𝟏 𝟏 selected with 80 more candidates got selected than A. What was
Ex. 1 : is what percent of ?
𝟓 𝟔 the number of candidates appeared from each State ?
A) 7600 B) 8000 C) 8400
Sol. : D) Data inadequate

Ex. 2 : 6) Raj saves 20% of his salary. His salary has increased by
x% of 35 = 2.275, Find the value of x = ? 20% and his expenditure has increased by 30%. What is the
percentage change in his savings?
A) 20% fall B) 4% fall C) 20% rise
Sol. : D) 4% rise

Ex. 3 : 7) In an examination, 40% marks are required to pass. A


Find the number which exceeds 15% of itself by 51% obtains 10% less than the number of marks required to pass. B
Sol. : x – 15% of x = 51 obtains 111/9 % less than A and C obtained 413/17% less than
the number of marks obtained by A and B together. What marks
did C get?
 100x – 15x = 5100 A) 50 B) 40 C) 35 D) 45
 85x = 5100
8) Sita’s salary in 2012 was Rs.1000/day and her salary in 2013
 x = 60
was Rs.1250/day. Again Sita’s salary in 2013 is Rs.1000/day.
what is the percent change in the salary of the year 2013?
Ex. 4 :
A) 12.5% B) 6.67% C) 25% D) 35 %
If the price of an item is increased by 30% and then a discount
of 5% in given on the increased price. What will be the effect on
9) A and B enter into a partnership. A puts in the whole capital
sale ?
of Rs.45000 on the condition that the profits will be equally
Sol. :
divided after which B will pay an interest on half the capital at
10% p.a. and receive 60 per month from A for carrying on the
concern. What is the yearly profit, if B's income is half of A's
income?
A) Rs.8190 B) Rs.9180 C) Rs.6180
D) Rs. 6900

10) In a certain school, 20% of students are below 8 years of


Ex. 5 : A’s income is 80% of B’s. B’s income is 60% of C’s. If age. The number of students above 8 years of age is 2/3 of the
C’s income is 80,000. What is A’s income ? number of students of 8 years of age which is 48. What is the
Sol. : B’s income = 60% of 80,000 = Rs. 48,000 total number of students in the school?
A’s income = 80% of 48,000 = Rs. 38,400 A) 72 B) 80 C) 120 D) 150
E) 100

For details on our programs: - Contact: +91 9930202329, +91, 8291924300 or visit www.campuscredentials.com (22)
11) If the sales tax reduced from 3 1/2 % to 3 1/3%, then what B) A is greater than B
difference does it make to a person who purchases an article C) Relationship between A and B cannot be determined.
with market price of Rs. 8400? D) If x is smaller than y, then A is greater than B.
A) 12 B) 13 C) 14 D) 15 E) None of these
E) None of these
21) Two numbers A and B are such that the sum of 5% of A and
12) 1100 boys and 700 girls are examined in a test; 42% of the 4% of B is two-third of the sum of 6% of A and 8% of B. Find
boys and 30% of the girls pass. The percentage of the total who the ratio of A:B.
failed is: A) 2 : 3 B) 1 : 1 C) 3 : 4 D) 4 : 3
A) 58% B) 62 2/3% C) 64% D) 78%
22) Rajeev buys good worth Rs. 6650. He gets a rebate of 6%
13) While purchasing one item costing Rs. 400, I had to pay the on it. After getting the rebate, he pays sales tax @ 10%. Find
sales tax at 7% and on another costing Rs. 6400, the sales tax the amount he will have to pay for the goods.
was 9%. What percent of the sales tax I had to pay, taking the A) Rs. 6876.10 B) Rs. 6999.20 C) Rs. 6654
two items together on an average? D) Rs. 7000
𝟏𝟑 𝟏𝟓 𝟏 23) Difference of two numbers is 1660. If 7.5% of the number is
A) 8% B) 8 % C) 8 % D) 8 %
𝟏𝟕 𝟏𝟕 𝟐
12.5% of the other number, find the number?
A) 2390 and 4050B) 2490 and 4150
14) Which one of the following shows the best percentage?
𝟑𝟖𝟒 𝟒𝟐𝟓 𝟓𝟕𝟎 𝟒𝟖𝟎 C) 2400 and 4060 D) 2490 and 4100
A) B) C) D) E) None of these
𝟓𝟒𝟎 𝟓𝟎𝟎 𝟕𝟎𝟎 𝟔𝟔𝟎

15) The price of a car is Rs. 3,25,000. It was insured to 85% of 24) What percent of 6.5 litres is 130 ml?
its price. The car was damaged completely in an accident and A) 1% B) 2% C) 3% D) 4%
the insurance company paid 90% of the insurance. What was E) None of these
the difference between the price of the car and the amount
received ? 25) In an election between two candidates, 75% of the voters
A) Rs. 32.500 B) Rs. 48,750 C) Rs. 76,375 cast thier votes, out of which 2% of the votes were declared
D) Rs. 81,250 invalid. A candidate got 9261 votes which were 75% of the total
valid votes. Find the total number of votes enrolled in that
16) T he price of mangoes has increased by 25%. By what election?
percent the consumption needs to be reduced so that the A) 16800 B) 17800 C) 26800
expense on mangoes remains the same? D) 306800 E) None of these
A) 25% B) 33.33% C) 20% D) None
26) If A is 150 percent of B, then B is what percent of (A + B) ?
17) Mohan spends 40% of his salary on food items, 50% of the 𝟏 𝟐
A) 33 % B) 40% C) 66 % D) 75%
𝟑 𝟑
remaining on transport, 30% of the remaining on clothes , after
spending on food and transport; and saves the balance. If he
27) If one number is 80% of the other and 4 times the sum of
saves Rs 630 every month, what is his monthly salary?
their squares is 656, then the numbers are:
A) Rs 1500 B) Rs 3000 C) Rs 5000
A) 4, 5 B) 8, 10 C) 16, 20
D) Rs 6500
D) None of these
18) The population of rats in a locality increases by 20% in one
28) The price of a car is Rs. 3,25,000. It was insured to 85% of
year. Observing this, the pest control committee decided to use
its price. The car was damaged completely in an accident and
a special kind of pesticide `xyz' which effectively kills 160 rats
the insurance company paid 90% of the insurance. What was
in 3 months. Just after 2 years, what is the net increase or
the difference between the price of the car and the amount
decrease in the population of rats if, initially the population of
received?
rats is 3200 and pesticide is used effectively?
A) Rs. 32,500 B) Rs. 48,750 C) Rs. 76,375
A) Increase of 128 rats
D) Rs. 81,250
B) Decrease of 128 rats.
C) Neither an increase nor a decrease in the population
29) In an examination, 5% of the applicants were found
D) None of these
ineligible and 85% of the eligible candidates belonged to the
general category. If 4275 eligible candidates belonged to other
19) One merchant correctly calculates his percentage profit on
categories, then how many candidates applied for the
the cost price; another wrongly calculates it on the selling
examination?
price. The difference in actual profits if both claim to make 17
A) 30,000 B) 35,000 C) 37,000
.5 % profit on goods sold at Rs.3760 is
D) None of these
A) Rs.98 B) Rs.56 C) Rs.65
D) Cannot be determined

20) If A = x% of y and B = y% of x, then which of the following


is true?
A) A is smaller than B.

For details on our programs: - Contact: +91 9930202329, +91, 8291924300 or visit www.campuscredentials.com (23)
Practice Set 2: 13) A scored 30% marks and failed by 18 marks. B scored 35%
marks and obtained 12 marks more than those required to pass.
1) A student has to obtain 35% of the total marks to pass. He The pass percentage is :
got 165 marks and failed by 45 marks. The maximum marks A) 33% B) 38% C) 43% D) 46%
are:
A) 300 B) 500 C) 800 D) 600 14) The price of a table is Rs. 350 more than that of a chair. If 7
tables and 7 chairs together cost Rs. 4550, by what percent is
2) A’s salary is first increased by 40% and then decreased by the price of the chair less than that of the table ?
50%. The result is the same as B’s salary increased by 50% and A) 33 1/3 % B) 70% C) 66 2/3 %
then decreased by 40%. Find the ratio of B’s salary to A’s D) None of these
salary?
A) 10 : 9 B) 6 : 7 C) 7 : 9 D) 9 : 10 15) A reduction of 23% in the price of Rice enables a person to
buy 11.5 kg. more for Rs. 100. what is the reduced price per Kg.
3) A% of B is B% of : ?
A) A B) B/100C) A/100 D) 100A A) Rs 1/Kg B) Rs 2/ Kg C) Rs 3/Kg
D) Rs 4/Kg
4) The length of a rectangle plot is increased by 10% to keep its
area unchanged, the width of the plot should be : 16) In a restaurant, 70% had vegetarian dinner while 20% had
A) Kept unchanged B) increased by 10% non-vegetarian dinner and 10% had both types of dinner. If 120
C) increased by 9 1/11% D) Reduced by 9 1/11% people were present, how many did not eat either type of
dinner?
5) A salesman averages Rs. 390 during a normal 30 h week. A) 20 B) 24 C) 26 D) 28
During a sale, his rates are increased by 50%. What is his
commission if he puts in 50 h during the sale? 17) The price of a radio includes the manufacturing cost, 10%
A) 650 B) 540 C) 600 D) 975 sales tax and 20% profit. What is the manufacturing cost, if the
price is Rs. 16,900? (Sales tax and profit are to be calculated on
6) A is 5 times as large as B. then percent by which B is less manufacturing cost.)
than A, is : A) 10.000 B) 12,000 C) 13,000 D) 9,000
A) 831/3% B) 161/3% C) 90% D) 80%
18) To a sugar solution of 4 liters containing 30% sugar, one
7) A secured 40%marks in Hindi, 50% in English and 60% in liter of water is added. The percentage of sugar in the new
Maths and 70% in Science. What were his total marks if the solution is:
maximum marks obtained in each of these 4 subjects was 60? A) 13 1/3 % B) 15% C) 30% D) 24%
A) 125 B) 120 C) 132 D) 150
19) Jaya’s Mathematics Test had 80 problems i.e., 40
8) Milk contains 6% water. What quantity of pure milk should arithmetic, 30 algebra and 10 geometry problems. Although she
be added to 15 liters of milk to reduce this to 4%? answered 80% of the arithmetic, 60% of the algebra and 50% of
A) 6.5 liters B) 7 liters C) 7.5 liters the geometry problems correctly, she did not pass the test
D) can’t be determined because she got less than 75% of the problems right. How many
more questions she must have attempted correctly to earn a
9) A man spends 25% of his income on food, 45% on children’s 75% passing grade?
education and 70% of the remaining on house rent. What A) 5 B) 10 C) 15 D) 20
percent of his income he is left with?
A) 8% B) 10% C) 9% D) 14% 20) The Salary of a person was reduced by 20%. By what
percent should his reduced salary be raised so as to bring it
10) A bag contains 800 coins of 25 p and 1600 coins of 50 p. If with his original salary?
16% of 25 p coins and 32% of 50 p coins are removed, the A) 11 1/9 % B) 20 % C) 25%
percentage of money removed from the bag is nearly: D) None of these
A) 15.6% B) 17.8% C) 21.6% D) 28.8%
21) During one year, the population of a town increased by 5%
11) The length of a rectangle is increased by 30% and breadth and during the next year, the population decreased by 5%. If the
is decreased by 15%. Calculate the percentage change in the total population is 10374 at the end of the second year, then
area. what was the population size in the beginning of the first year?
A) 8.5% B) 8% C) 10% D) 10.5% A) 8000 B) 10000 C) 10400 D) 10100

12) When the price of sugar was increased by 30%, a family 22) Due to a reduction of 3 1/8% in the price of sugar, a man is
reduced its consumption in such a way that the expenditure on able to buy 1 kg more for Rs. 208. Find the reduced rate of
sugar was only 30% more than before. If 25 kg were consumed sugar.
per month before, find the new monthly consumption. A) Rs. 5.50 per kg B) Rs. 6.50 per kg
A) 20 Kg B) 25 Kg C) 30 Kg D) 35 Kg C) Rs. 7.50 per kg D) None of these

For details on our programs: - Contact: +91 9930202329, +91, 8291924300 or visit www.campuscredentials.com (24)
23) The value of a machine depreciates at the rate of 20% per Chapter – 03: Profit & Loss, Partnership
annum. If its present value is Rs. 1,87,500, what will be its
worth after 2 years? A complete business transaction takes place when goods
A) Rs. 100000 B) Rs. 150000 C) 200000 products are bought for one price and sold for another. The
D) Rs 120000 selling price of a product is denoted by SP and the cost price is
denoted by CP.
24) In an examination, 75% of the students passed in Science, There is a profit if SP is greater than CP or there can be a loss
70% in Mathematics and 65% in both Science and if SP is lesser than CP. If SP is same as that of CP, then the
Mathematics. If 60 students failed in both the subjects, find the transaction is made at no profit and no loss basis.
total number of students.
A) 100 B) 200 C) 300 D) 400

25) In an election a candidate who gets 82% of the votes is


elected by a majority of 512 votes. What is the total number of
votes polled?
A) 672 B) 700 C) 749 D) 800

26) The population of a village is 6400. If the number of males


increases by 40% and that of females increased by 30%, the
population becomes 8380. Find the population of females in the
town.
A) 6400 B) 5800 C) 7000 D) 6000

27) An inspector rejects 0.06% of the meters as defective. How


many will he examine to reject 3?
A) 2000 B) 5000 C) 2500 D) 3000
If CP = SP , then the transaction is made at no profit and no
28) If 50% of the 2 : 3 solution of milk and water is replaced loss.
with water, then the concentration of the solution is reduced by Profit or Loss is generally represented as a percentage of the
A) 25% B) 33.33% C) 50% D) 75% cost price unless otherwise stated.

29) Population of a district is 4,76,000 out of which 1,56,000 Example 1 : A camera is purchased for Rs. 999 and sold for Rs.
are males. 50% of the population is literate. If 60% males are 777. Find the loss percent
literate, the number of women, who are literate, is: Solution :
A) 17,000 B) 1,53,000 C) 1,44,400
D) 1,64,300

30) 40kg. Solution of salt and water contains 5% salt. What


quantity of salt must be added to the solution to increase this to Example 2 :
8%? I bought an article for Rs. 61.25 and sold it for Rs. 73.50. What
A) 2 2/3kg. B) 1.80 kg. C) 1.30 kg. is the gain percentage?
D) None of these Solution : SP = Rs. 73.50, CP = 61.25, Profit = Rs. 73.50 – Rs.

61.25 = Rs. 12.25

Example 3 :
Selling an article for Rs. 390 means a loss of 12%. Find the cost
price of that article.
Solution : SP = Rs. 390 ; Loss % = 12% ; CP = ?
CP – SP = Loss  CP - 390 = Loss

If two items are sold at the same SP, one at a profit of P% and
the other at a loss of P%, then
𝟐𝑷𝟐 × 𝑺𝑷 𝑷𝟐
𝑳𝒐𝒔𝒔 = 𝑳𝒐𝒔𝒔% =
(𝟏𝟎𝟎)𝟐 − 𝑷𝟐 𝟏𝟎𝟎
For details on our programs: - Contact: +91 9930202329, +91, 8291924300 or visit www.campuscredentials.com (25)
Suppose SP of each item = Rs. x.
𝟏𝟎𝟎
CP of one item = ×𝒙
𝟏𝟎𝟎+𝑷
𝟏𝟎𝟎
CP of other item = ×𝒙
𝟏𝟎𝟎−𝑷
𝟏𝟎𝟎𝒙 𝟏𝟎𝟎𝒙 𝟐𝟎𝟎𝟎𝟎𝒙
Total CP = + = (𝟏𝟎𝟎)𝟐 +𝑷𝟐
𝟏𝟎𝟎+𝑷 𝟏𝟎𝟎−𝑷 Example 6 : Rohan bought a music system at 15% discount on
Total SP = 2x the marked price. Had he bought it at 18% discount, he would
𝟐𝟎𝟎𝟎𝟎𝒙 𝟐𝒙𝑷𝟐 𝟐𝑷𝟐 have saved Rs. 150. What is the price at which he bought the
Loss - − 𝟐𝒙 = = (𝟏𝟎𝟎)𝟐
𝟏𝟎𝟎𝟎𝟎−𝑷𝟐 (𝟏𝟎𝟎)𝟐 −𝑷𝟐 −𝑷𝟐
system ?
𝟐𝑷𝟐 ×𝑺𝑷 𝑷𝟐
Loss % = × 𝟏𝟎𝟎 = Solution : Let the price at which Rohan bought the system be
𝟐𝟎𝟎𝟎𝟎𝑺𝑷 𝟏𝟎𝟎
Rs. x.
Example 4 : Let the labeled price be Rs. 100.
A man sells two watches for Rs. 2900 each. On one watch he SP in the first case = Rs. 85, SP in the second case = Rs. 82
gains 10% and on the other he losses 10%. Find his gain/loss
percentage on the whole transaction.
Solution : There is always a loss given by loss % =

Practice Set 1:

1) A fruit seller sells mangoes at the rate of Rs.9 per kg and


Example 5 :
thereby loses 20%. At what price per kg, he should have sold
The CP of 11 articles is equal to the SP of 10 articles. Find the
them to make a profit of 5%?
percentage profit.
A) Rs.11.81 B) Rs.12 C) Rs.12.25
Solution : If CP of 1 article is Re. 1 then CP of 10 articles is Rs.
D) Rs.12.31
10 and CP of 11 articles is Rs. 11. Given SP of articles = CP of
11 articles = Rs. 11.
2) A shopkeeper expects a gain of 22½% on his cost price. If in
 Profit = Re. 1 a week, his sale was of Rs.392, what was his profit?
A) Rs. 18.20 B) Rs. 70 C) Rs. 72
D) Rs. 88.25
DISCOUNT :
Buy x get y free i.e., if x + y articles are sold at cost price of x 3) If a man reduces the selling price of a fan from Rs.400 to
articles, then the percentage discount Rs.380, his loss increases by 2%. The cost price of the fan is
A) Rs. 480 B) Rs. 500 C) Rs. 600
D) Rs. 1000
By using false weight, If a substance is sold at cost price, the
overall gain % is given by Gain % 4) If by selling 110 mangoes, the C.P. of 120 mangoes is
realised, the gain percentage is :
𝟏 𝟏 𝟏𝟎 𝟏
A) 9 % B) 9 % C) 10 % D) 11 %
𝟏𝟏 𝟗 𝟏𝟏 𝟗
If two items have the same CP and loss % and gain % on the
two are equal, the net loss or profit is zero. 5) A man buys eggs at 2 for Re. 1 and an equal number at 3 for
Rs. 2 and sells the whole at 5 for Rs. 3. His gain or loss percent
DIFFERENT TERMS : is:
Marked Price : The price marked on the article is called as 𝟐 𝟔 𝟐 𝟔
A) 2 % loss B) 3 % gain C) 3 % loss D) 2 % gain
Marked Price or List price 𝟕 𝟕 𝟕 𝟕

Trade Discount : The reduction made on the marked price of an


article is called as Trade Discount. If marked price = Rs. 80 6) If selling price is doubled, the profit triples. Find the profit
and Discount = 16% percent.
𝟐 𝟏
A) 66 B) 100 C) 105 D) 120
𝟑 𝟑

If discount is given, then selling price is different from marked 7) A man buys a cycle for Rs. 1400 and sells it at a loss of 15%.
price. It is also called as Net Price What is the selling price of the cycle?
Also, Selling price = Marked price – Discount A) Rs. 1090 B) Rs. 1160 C) Rs. 1190
If discount is not given selling price is same as marked price. D) Rs. 1202
In case of two successive discounts of a% and b%, effective
discount 8) A man buys a cycle for Rs.1400 and sells it at a loss of 15%.
𝒂𝒃 What is the selling price of the cycle?
(𝒂 + 𝒃 )% A) Rs. 1090 B) Rs. 1160 C) Rs. 1190
𝟏𝟎𝟎
D) Rs. 1202
e.g. Let the marked price of a TV be Rs. 14,500 ; two successive
discount of 10% and 20% are given, then effective discount

For details on our programs: - Contact: +91 9930202329, +91, 8291924300 or visit www.campuscredentials.com (26)
9) Peter purchased a machine for Rs. 80,000 and spent Rs.5000 21) A man buys an article for 10% less than its value and sells it
on repair and Rs.1000 on transport and sold it with 25% profit. for 10% more than its value. His gain or loss percent is :
At what price did he sell the machine? A) no profit, no loss B) 20% profit
A) Rs. 1,05,100 B) Rs. 1,06,250 C) less than 20% profit D) more than 20% profit
C) Rs. 1,07,500 D) Rs. 1,17,500
22) Jacob bought a scooter for a certain sum of money. He
10) I gain 70 paise on Rs.70. My gain percent is spent 10% of the cost of repairs and sold the scooter for a profit
A) 0.1% B) 1% C) 7% D) 10% of Rs. 1100. How much did he spend on repairs if he made a
profit of 20%?
11) The cost price of an article is 64% of the marked price. A) Rs. 400 B) Rs. 440 C) Rs. 500 D) Rs. 550
Calculate the gain percent after allowing a discount of 12%.
A) 37.5% B) 48% C) 50.5% D) 52% 23) By selling 45 lemons for Rs. 40, a man loses 20%.
How many should be sell for Rs. 24 to gain 20% in the
12) A trader mixes three varieties of groundnuts costing Rs. 50, transaction?
Rs. 20 and Rs. 30 per kg in the ratio 2 : 4 : 3 in terms of weight, A) 18 B) 19 C) 20
and sells the mixture at Rs. 33 per kg. What percentage of profit D) 23.8 E) None of these
does he make?
A) 8% B) 9% C) 10% 24) A man buys an article for 10% less than its value and sells it
D) 11% E) None of these for 10% more than its value. His gain or loss percent is:
A) no profit, no loss B) 20% profit
13) A vendor loses the selling price of 4 oranges on selling 36 C) less than 20% profit D) more than 20% profit
oranges, His loss percent is: E) None of these
A) 9% B) 10% C) 11%
D) 12% E) None of these 25) Alfred buys an old scooter for Rs. 4700 and spends Rs. 800
on its repairs. If he sells the scooter for Rs. 5800, his gain
14) The ratio of the cost price and the selling price is 4 : 5. The percent is:
profit percent is: 𝟒 𝟓
A) 4 % B) 5 % C) 10% D) 12%
𝟕 1𝟏
A) 10% B) 20% C) 25%
D) 30% E) None of these
26) The percentage profit earned by selling an article for Rs.
1920 is equal to the percentage loss incurred by selling the
15) The cost price of 20 articles is the same as the selling price
same article for Rs. 1280. At what price should the article be
of x articles. If the profit is 25%, then the value of x is:
sold to make 25% profit?
A) 15 B) 16 C) 18 D) 25
A) Rs. 2000 B) Rs. 2200 C) Rs. 2400
D) Data inadequate
16) On selling 17 balls at Rs. 720, there is a loss equal to the
cost price of 5 balls. The cost price of a ball is:
27) A man bought goods worth Rs. 6000 and sold half of them
A) Rs. 45 B) Rs. 50 C) Rs. 55 D) Rs. 60
at a gain of 10%. At what gain percent must he sell the
remainder so as to get a gain of 25% on the whole ?
17) Samant bought a microwave oven and paid 10 % less than
A) 25% B) 30% C) 35% D) 40%
the original price. He sold it with 30% profit on the price he
had paid.What percentage of profit did samant earn on the
28) A sells an article which costs him Rs. 400 to B at a profit of
original price?
20%. B then sells it to C, making a profit of 10% on the price he
A) 17% B) 20% C) 27% D) 32%
paid to A. How much does C pay B?
A) Rs. 472 B) Rs. 476 C) Rs. 528
18) A book was sold for Rs.27.50 with a profit of 10%.If it were
D) Rs. 532
sold for Rs.25.75, then what wuould have been the percentage
of profit or loss?
29) In a certain store, the profit is 320% of the cost. If the cost
A) 2% B) 3% C) 4% D) 5%
increases by 25% but the selling price remains constant,
approximately what percentage of the selling price is the profit?
19) At what percent above the cost price must a shopkeeper
A) 30% B) 70% C) 100% D) 250%
mark his goods so that he gains 20% even after giving a
discount of 10% on the marked price ?
𝟏 𝟏 30) Arun purchased 30 kg of wheat at the rate of Rs. 11.50 per
A) 25% B) 30% C) 33 % D) 37 % kg and 20 kg of wheat at the rate of Rs. 14.25 per kg. He mixed
𝟑 𝟐
the two and sold the mixture. Approximately what price per kg
20) A person purchases 90 clocks and sells 40 clocks at a gain should he sell the mixture to make 30% profit?
of 10% and 50 clocks a a gain of 20%. If he sold all of them at a A) 15.20 B) 16.30 C) 17.40
uniform profit of 15%, then he would have got Rs. 40 less. The D) 18.50 E) None of these
cost price of each clock is :
A) Rs. 50 B) Rs. 60 C) Rs. 80 D) Rs. 90 31) Akash, Balu and Gopi invested Rs. 6000 , Rs.4000 and
Rs.8000 in a business. Akash left a business after 6 months. If
after one year there was a gain of Rs.5200.Then what will be

For details on our programs: - Contact: +91 9930202329, +91, 8291924300 or visit www.campuscredentials.com (27)
the share of Akash ? Chapter 04: SI & CI
A)1000 B)1020 C)1040
D)1004 E)None of these SIMPLE INTEREST :

32) Kiran and Yokesh partners in a business . Kiran contributes When a sum of money is lent by A to B. A is called the lender
¼ of the capital for 15 months and Yokesh received 2/3 of the (creditor) and B is called the borrower (debtor). The amount
profit. Then how long Yokesh invest the money in the business ? borrowed from A is called the ‘principal’. B uses the money
A) 8 months B) 9 months C) 10 months borrowed from A and pays him some extra amount for the time
D) 1 year E) None of these that he has used the money. This sum of money is called
‘interest’ This interest is known as ‘Simple Interest’ and is
33) Three partners shared a profit in the ratio 5:7:8.They had calculated as:
partnered for 10 months,8 months and 7 months. What was the
ratio of their investments?
A) 28:46:60 B) 28:49:64 C) b24:45:63
D) 25:40:64 E) None of these where P = principal, T = time for which money is borrowed, R
= rate (p.c.p.a.)
34) X, Y and Z hire a Auto Rs.250 and used it for 4,3 and 2 The sum of the principal and interest is called the ‘amount’, and
hours respectively. Hire charge paid by Z is is denoted by A.
A) 55 B) 55.50 C) 55.55
D) 55.25 E) None of these

35) A, B, C started a business with their investments in the ratio COMPOUND INTEREST :
2:4:5. After 8 months, A invested the same amount as before
and both B and C withdrew half of their investments. The ratio
Money is said to be lent at compound interest when at the end of
of their profits at the end of the year is: a year or at a fixed period, the interest that has become due is
A) 10:12;15 B) 12:20:25 C) 9:12:25
not paid to the lender, but is added to the sum lent and the
D) 12:25:20 amount thus obtained, becomes the principal for the next year
or period. The difference between the final amount and the
36) A, B, C subscribe Rs. 60,000 for a business, A subscribes original principal is called the ‘Compound Interest’.
Rs.8000 more than B and B Rs. 5000 more thanC. Out of a total 𝒓 𝑵
profit of Rs. 40,000, A receives : Amount : 𝑨 = 𝑷 (𝟏 + )
𝟏𝟎𝟎
A) Rs.20000 B) Rs.18000 C) Rs.16000 Compound interest : CI = A – P
D) Rs.14000 When Compound Interest is Reckoned Half-yearly
If the annual rate is r % per annum and is to be calculated for n
37) A, B and C enter into a partnership in the ratio 5/2:4/3:7/5 years.
. After 4 months, A increases his share 23%. If the total profit at In this case, rate = (r/2)% and time = (2n)
the end of one year be Rs.31,800, then B’s share in the profit is:  From above
A) Rs.7548.49 B) Rs.7548.96 A = P (1 + r/2  100)2n
C) Rs.7548.66 D) None of these When Compound Interest is Reckoned Quarterly
In this case, rate = (r/4)% quarterly and time = (4n) quarter
38) A starts a business with Rs 20,000 and after 4 months B also years.
joins with some capital. After a year, the profit is divided  As before A = P (1 + r/4  100)4n
between them in the ratio 5 : 3. How much did B invested?
A) Rs 12,000 B) Rs 18,000 C) Rs 15,050 Note : The difference between the compound interest and the
D) Rs 12,750 E) Rs 15,800 simple interest over two years is given by Pr² /100²
e.g. : If P = 2500, N = 2 years, R = 6 p.c.p.a., find SI and C.I.
39) A and B started a business by investing Rs 8,000 and Rs Sol. :
9,000 respectively. After 4 months A withdrew Rs 400 and B
added Rs 400 more. What is the share of B from the profit of Rs
10,200?
A) Rs 5460 B) Rs 5480 C) Rs 5560
D) Rs 4460 E) Rs 4640

40) The profit received at the end of a year is divided among A,


C.I. = Amount – P = 2809 – 2500 = 309.
B and C in the ratio 5:4:7 respectively. The share of A is Rs
Population Formula :
2300 more than that of B. What is the amount that C got?
The original population of a town is P and the annual increase
A) Rs 26,100 B) Rs 16,800 C) Rs 14,100 𝒓 𝒏
D) Rs 16,100 E) Rs 19,140 is r%, then the population in n years is P(𝟏 + ) and if the
𝟏𝟎𝟎
annual decrease is r%, then the population in n years is given
𝒓 𝒏
by a change of sign in the formula, i.e. 𝑷 (𝟏 − )
𝟏𝟎𝟎

For details on our programs: - Contact: +91 9930202329, +91, 8291924300 or visit www.campuscredentials.com (28)
e.g. if the annual increase in the population of a town is 4% and
the present population is 15000, then the population after 2
years will be = 15000 (1.04)² = 16224.
Depreciation Formula :
Let P = present value of the machine, R = rate of depreciation
𝒓 𝑵
 Value of the machine after n years = 𝑷 (𝟏 + 𝟏𝟎𝟎)

SOLVED EXAMPLES
Ex. 1 :
What invested at 6% for 4 months will yield a simple interest of
Rs. 30 ?
Solution : Practice Set 1

1) The difference between simple and compound interests


compounded annually on a certain sum of money for 2 years at
4% per annum is Re. 1. The sum (in Rs.) is:
A) 625 B) 630 C) 640 D) 650
Ex. 2 :
2) The effective annual rate of interest corresponding to a
In what time will Rs. 1700 amount to Rs. 2924 at 8% per annum
nominal rate of 6% per annum payable half-yearly is:
at simple interest ?
A) 6.06% B) 6.07% C) 6.08% D) 6.09%
Solution :
P = Rs. 1700 ; A = Rs. 2924 ; S. I. = 2924 – 1700 = Rs. 1224
3) A sum fetched a total simple interest of Rs. 4016.25 at the
rate of 9 p.c.p.a. in 5 years. What is the sum?
A) Rs. 4462.50 B) Rs. 8032.50 C) Rs. 8900
D) Rs. 8925 E) None of these
Ex. 3 :
What is the compound interest on Rs. 15000 at 20% per annum 4) An amount of Rs. 1,00,000 is invested in two types of shares.
for 4 years ? The first yields an interest of 9% per annum and the second,
Solution : 11% per annum. If the total interest at the end of one year is
𝟑
9 %, then the amount invested in each share was:
𝟒
A) Rs 52,500 ; Rs 47,500 B) Rs 72,500 ; Rs 27,500
C) Rs 62,500 ; Rs 37,500 D) Rs 82,500 ; Rs 17,500
E) None of these

5) Two equal amount of money was deposited in two banks,


 Compound interest = Amount – Principal = Rs. (31104 - each at 15% per annum, 3.5 years and 5 years respectively. If
15000) = Rs. 16104 the difference between the interest is Rs 144, each sum is:
A) 620 B) 630 C) 640
Ex. 4 : D) 650 E) None of these
Find the compound interest on Rs. 3000 at 10% per annum for
1 years, compounded half yearly. 6) On a sum of money, the simple interest for 2 years is Rs. 660,
Solution : while the compound interest is Rs. 696.30, the rate of interest
Principal = Rs. 3000, Time = 1 year = 2 years being the same in both the cases. The rate of interest is:
Rate = 10% per annum = 5% yearly A) 10 B) 12 C) 15
D) 18 E) None of these

7) Divide Rs. 3903 between A and B, so that A's Share at the


 C.I. = Rs. (3307.5 - 3000) = Rs. 307.5 end of 7 years may equal to B's share at the end of 9 years,
compound interest being at 4 percent.
Ex. 5 : A) 2018 and 1885B) 2028 and 1875
The difference between the compound interest and the simple C) 2008 and 1895 D) 2038 and 1865
interest on a certain sum at 20% per annum for 2 years is Rs. E) None of these
200. Find the sum.
Solution : 8) If Rs. 1000 be invested at interest rate of 5% and the interest
Let the sum be Rs. x be added to the principal after 10 yr, then the number of years
in which it will amount to Rs 2000 is ?
A) 162/3 yr B) 161/4 yr C) 16 yr D) 11 yr

For details on our programs: - Contact: +91 9930202329, +91, 8291924300 or visit www.campuscredentials.com (29)
9) Sona invests an amount of Rs.9535 at the rate of 4% per 19) How much time will it take for an amount of Rs. 450 to yield
annum, for how many years did she invest the amount to obtain Rs. 81 as interest at 4.5% per annum of simple interest?
the double her sum? A) 3.5 years B) 4 years C) 4.5 years D) 5 years
A) 10 yr B) 25 yr C) 5 yr D) 4 yr
20) The Sum of money that will produce Rs. 1770, interest in 7
10) The Simple interest on a certain sum for 2 years at 10% per 1/2 years at 8% simple interest per annum is:
annum is Rs. 90. The corresponding compound interest is: A) Rs 2950 B) Rs 2800 C) Rs 3120
A) 99 B) 95.60 C) 94.50 D) Rs 3200 E) None of these
D) 108 E) None of these
21) Mr. Mitesh invested an amount of Rs. 12000 at the simple
11) The rate of interest on a sum of money is 4% per annum for interest rate of 10% per annum and another amount at the
the first 2 years, 6% per annum for the next 4 years and 8% per simple interest rate of 20% per annum. The total interest earned
annum for the period beyond 6 years. If the simple interest at the end of one year on the total amount invested became 14%
accrued by the sum for the total period of 9 years is Rs 1120. per annum. Find the total amount invested?
What is the sum? A) 20,000 B) 21,000 C) 20,800
A) 1800 B) 1900 C) 2000 D) 21,000 E) None of these
D) 2100 E) 2200
22) If the simple interest on Rs. 3000 is less than the Simple
12) A sum of Rs. 800 amounts to Rs. 920 in 3 years at simple Interest on Rs. 2000 at 5% by Rs. 50, find the time?
interest. If the interest rate is increased by 3%, it would amount A) 1 B) 2 C) 3 D) 4
to how much? E) None of these
A) 764 B) 992 C) 850
D) 770 E) None of these 23) A candidate who gets 20% marks fails by 10 marks but
another candidate who gets 42% marks gets 12% more than the
13) A sum of money at simple interest amounts to Rs. 815 in 3 passing marks. Find the maximum marks?
years and to Rs. 854 in 4 years. The sum is: A) 100 B) 80 C) 70 D) 60
A) Rs. 650 B) Rs. 690 C) Rs. 698 D) Rs. 700 E) None of these

14) There is 60% increase in an amount in 6 years at simple 24) A person invested some amount at the rate of 12% simple
interest. What will be the compound interest of Rs. 12,000 after interest and a certain amount at the rate of 10% simple interest.
3 years at the same rate? He received yearly interest of Rs130. But if he had interchanged
A) Rs. 2160 B) Rs. 3120 C) Rs. 3972 the amounts invested, he would have received f 4 more as
D) Rs. 6240 E) None of these interest. How much did he invest at 12% simple interest?
A) Rs 700 B) Rs. 500 C) Rs. 800 D) Rs. 400
15) Amit borrowed a certain sum of money for 2 yr at 8% per
annum on simple interest and immediately lent it to Ravi but at 25) A trader owes a merchant Rs. 10028 due in 1 yr; but the
compound interest and gained by Rs 16. What amount did Amit trader wants to settle the account after 3 months. If the rate of
borrow? interest is 12% per annum, how much cash should he pay?
A) Rs. 1600 B) Rs. 2500 C) Rs. 24000 D) Rs. 1800 A) Rs. 9025 B) Rs. 9200 C) Rs. 9600 D) Rs. 9200

16) Sumit lent some money to Mohit at 5% per annum simple 26) Two equal sums of money were invested, one at 4% and the
interest. Mohit lent the entire amount to Birju on the same day other at 41/2. At the end of 7 yr, the simple interest received from
at 81/2 % per annum. In this transaction after a year, Mohit the latter exceeded that received from the former by Rs. 31.50.
earned a profit of Rs. 350. Find the sum of money lent by Sumit Each sum was?
to Mohit. A) Rs 1000 B) Rs 500 C) Rs 750 D) Rs 900
A) Rs. 10000 B) Rs. 9000 C) Rs. 10200
D) None of these 27) The least number of complete years in which a sum of
money put out at 20% compound interest will be more than
17) Maninder puts equal amounts of money in two schemes: one doubled is:
at 10% per annum compound interest payable half yearly and A) 3 B) 4 C) 5 D) 6
the second at a certain per cent per annum compound interest
payable yearly. If he gets equal amounts after 3 yr, what is the 28) Mr. Ramu invested an amount of Rs. 13,900 divided in two
value of the second per cent? different schemes A and B at the simple interest rate of 14% p.a.
A) 10 1/4% B) 10% C) 9 1/2% D) 81/4% and 11% p.a. respectively. If the total amount of simple interest
earned in 2 years be Rs. 3508, what was the amount invested in
18) Albert invested an amount of Rs. 8000 in a fixed deposit Scheme B?
scheme for 2 years at compound interest rate 5 p.c.p.a. How A) Rs. 6400 B) Rs. 6500 C) Rs. 7200
much amount will Albert get on maturity of the fixed deposit? D) Rs. 7500 E) None of these
A) Rs. 8600 B) Rs. 8620 C) Rs. 8820
D) None of these 29) Mr. Jumbo borrowed a sum of Rs. 10000 from a finance
company for 6 years at 8% per annum. The amount returned by
Mr. Prakash to the finance company is:

For details on our programs: - Contact: +91 9930202329, +91, 8291924300 or visit www.campuscredentials.com (30)
A) Rs. 12600 B) Rs. 13300 C) Rs. 14800 A) Rs. 500 B) Rs. 600 C) Rs. 700 D) Rs. 800
D) Rs. 15200 E) None of these
12) A sum of money placed at compound interest doubles itself
30) A sum was put a simple interest at a certain rate for 2 years. in 4 years. In how many years will it amount to eight times itself
Had it been put at 3% higher rate, it would have fetched Rs. 72 ?
more. The sum is: A) 13 B) 14 C) 12 D) 15
A) 8% B) 9% C) 10% D) 11%
E) None of these 13) At what rate percent compounded yearly will Rs. 48,400
amount to Rs. 52,900 in 2 years?
Practice Set 2: A) 2% B) 3 ½ % C) 7 ¼ % D) 46/11 %

1) If a sum of money at compound interest amounts to 4 times 14) At what rate percent will be compound interest, does a sum
itself in 3 yr, then in how many years will it be 16 times itself ? of money become 16 times in 2 yr ?
A) 12 yr B) 6 yr C) 9 yr D) 15 yr A) 100% B) 150% C) 300% D) 200%

2) How much interest will Rs. 12,000 earn in 8 months at an 15) A sum of money placed at compound interest amounts to 5
annual rate of 8 percent ? times itself in 4 yr. In how many years will it amount to 25 times
A) 450 B) 500 C) 640 D) 600 itself?
A) 8 yr B) 15 yr C) 14 yr D) 10 yr
3) A sum of money invested at compound interest amounts in 4
yr to Rs. 4200 and in 5 yr to Rs. 4410. The interest rate per 16) The difference between the compound interest and simple
annum is interest on a certain sum at 10% per annum for 2 yr is Rs. 432.
A) 6% B) 5% C) 10% D) 12% Find the sum?
A) Rs. 43000 B) Rs. 43400 C) Rs. 43200 D) Rs. 64500
4) On what principle will the compound interest for 3 yr at 4
percent amount to Rs. 62.432? 17) Find the compound interest on Rs. 15000 at 24% per annum
A) Rs. 400 B) Rs. 500 C) Rs. 450 D) Rs. 550 for 6 months compounded quarterly.
A) Rs. 1800 B) Rs. 1854 C) Rs. 1984 D) Rs. 1900
5) The simple interest on a certain sum of money for 2 years at
5% per annum is Rs. 600. Find the compound interest at the 18) The compound interest on Rs. 400 for 1 yr at 6% per
same rate and for the same time? annum, the interest being payable half yearly, will be
A) Rs. 600 B) Rs. 615 C) Rs. 620 A) Rs. 424.36 B) Rs. 425.37 C) Rs. 14.14 D) Rs. 24.36
D) None of these
19) Rs. 6800 is borrowed at CI at the rate of 2% for the first
6) A sum of money doubles itself in 20 years . What is the rate of year, 4% for the second year and 5% for the third year. Find
interest? the amount to be paid after 3 years?
A) 10% B) 11% C) 5% A) Rs. 6834.24 B) Rs. 7324.10
D) None of these C) Rs. 7574.112 D) None of these

7) Vinod lent some money to Gaurang at 4% per annum simple 20) A man invested 1/3 of his capital at 9%, ¼ at 8% and the
interest. Gaurang lent the entire amount to Soham on the same remainder at 6%. If his annual income is Rs. 660, find the
day at 10½ % per annum. In this transaction, after a year capital?
Gaurang earned a profit of Rs. 650. Find the sum of money lent A) Rs. 7600 B) Rs. 8800 C) Rs. 6600
by Vinod to Gaurang. D) None of these
A) Rs.10,000 B) Rs. 9,000 C) Rs. 10,200
D) None of these 21) Two equal amounts of money are deposited in two banks
each at 8% per annum for 5½ years and 8 years respectively. If
8) On what sum will the difference between simple interest and the difference between their interests is Rs. 170, find the sum?
compound interest for 3 yr at 3 percent per annum is Rs. 8.181? A) Rs. 850 B) Rs. 815 C) Rs. 800
A) Rs. 2700 B) Rs. 3000 C) Rs. 3500 D) Rs. 2520 D) None of these

9) The ratio of the amount for two years under CI annually and 22) A certain sum of money amounts to Rs. 2584 in 3 years at
for one year under SI is 11:10. When the rate of interest is 12% per annum. In how many years will it amount to Rs. 3496
same, then the value of the rate of interest is at the same rate?
A) 12.5% B) 10% C) 20% D) 16.66% A) 5 Years B) 7 Years C) 15 Years D) 20Years

10) Find the difference between CI and SI on Rs. 6000 for 3 23) On what sum will the difference between SI and CI for 2 yrs.
years at 2% pa. at 8% per annum is Rs. 2.88?
A) Rs. 12.120 B) Rs. 15.125 C) Rs. 18.150 D) None A) 400 B) 350 C) 450 D) 500

11) The compound interest on a certain sum for 2 years is Rs.


61.80 and simple interest is Rs. 60. find the sum ?

For details on our programs: - Contact: +91 9930202329, +91, 8291924300 or visit www.campuscredentials.com (31)
24) What sum of money at compound interest will amount to Rs. Chapter 05: Averages
3439.17 in 3 years, if the rate of interest is 3% for the first year,
6% for the second year and 5% for the third year? The average (arithmetic mean) of a group or set of N numbers
A) Rs. 1500 B) Rs. 2000 C) Rs. 2500 is defined as the sum of those numbers divided by N.
D) Rs. 3000 Here N is the number of values or observations in a set.
𝑆𝑢𝑚 𝑜𝑓 𝑁 𝑜𝑏𝑠𝑒𝑟𝑣𝑎𝑡𝑖𝑜𝑛𝑠
25) A sum was put at SI at a certain rate for 3 years. Had it 𝐴𝑣𝑒𝑟𝑎𝑔𝑒 =
𝑇𝑜𝑡𝑎𝑙 𝑁𝑢𝑚𝑏𝑒𝑟𝑠 𝑜𝑓 𝑜𝑏𝑠𝑒𝑟𝑣𝑎𝑡𝑖𝑜𝑛𝑠
been put at 4% higher rate, it would have fetched Rs. 600 more. 𝑆𝑢𝑚
Find the sum ? 𝐴= 𝑜𝑟 𝑆𝑢𝑚 = 𝑁 × 𝐴𝑣𝑒𝑟𝑎𝑔𝑒
𝑁
A) Rs. 3500 B) Rs. 4500 C) Rs. 5000 D) Rs. 5500
Ex. 1: If weights of three Women are 60, 70 and 56 Kilogram,
26) The rate of interest for the first 3 years is 4% per annum, the average of the weights of Women can be calculated as :
for the next 2 years is 6% per annum and for the period beyond Sol.:
5 years 8% per annum. If a man gets Rs. 1200 as a simple 𝑆𝑢𝑚 𝑜𝑓 𝑤𝑒𝑖𝑔ℎ𝑡 𝑜𝑓 𝑊𝑜𝑚𝑒𝑛
interest for 7 years, how much money did he deposit? 𝐴𝑣𝑒𝑟𝑎𝑔𝑒 =
𝑁𝑢𝑚𝑏𝑒𝑟𝑠 𝑜𝑓 𝑊𝑜𝑚𝑒𝑛
A) Rs. 3500 B) Rs. 3000 C) Rs. 3200 D) Rs. 3800 60 + 70 + 56
=
3
27) A certain sum of money amounts to Rs. 5320 in 3 years and 186
to Rs. 5980 in 4½ years. Find the rate of interest ? =
3
A) 10% B) 11% C) 12% D) 13% = 62 Kilogram

28) At a certain rate of simple interest Rs. 600 amounts to Rs. Ex. 2:
744 in 4 years. If the rate of interest be decreased by 3%, what If average of 4, 6, 7 and x is 5, what is the value of x ?
will be the amount after 4 years? Sol.:
A) Rs. 600 B) Rs. 640 C) Rs. 660 D) Rs. 672 We have been given N = 4 observations and average of these
observations is 5.
29) At a certain rate of simple interest Rs. 500 amounts to Rs. So, Average =
665 in 3 years. If the rate of interest be increased by 4%, what 𝑆𝑢𝑚 4+6+7+𝑥
will be the amount after 3 years? 𝑆𝑜, 𝐴𝑣𝑒𝑟𝑎𝑔𝑒 = →5=
𝑁 4
A) Rs. 700 B) Rs. 792 C) Rs. 725 D) Rs. 780  20 = 17 + x
 x = 3., Hence, the missing observation is 3.
30) A sum of Rs. 2170 is lent out in two parts in such a way that
the interest on one part at 8% for 3 years is equal to that on Ex.3 :
another part at 12% for 5 years. Find the II part? If the average of five numbers is 20 and the sum of three
A) Rs. 600 B) Rs. 850 C) Rs. 700 D) Rs. 620 numbers is 26, then what is the average of other two numbers.
Sol.: Let the five numbers a, b, c, d and e.

Average in Different Cases :


1. If all the numbers in a set are the same, then that number is
the average of that set. i.e.
Average of set of values 5, 5 and 5 is 5.
Also be theoretically, Average =

2. If the numbers in a set are not all the same, then the average
must be greater than the smallest number and less than the
greatest number in the set.
i.e. Average of set of values 73,74,77,87,89

For details on our programs: - Contact: +91 9930202329, +91, 8291924300 or visit www.campuscredentials.com (32)
6) Find the average of all the numbers between 6 and 34 which
are divisible by 5
A) 18 B) 20 C) 24 D) 30
Here, 89 > 80 and 80 > 73.
Weighted Average To calculate the weighted average of a set of 7) The average age of a group of persons going for picnic is 16
numbers, multiply each number in the set by the number of years. Twenty new persons with an average age of 15 years join
times it appears, add all products and divide by the total the group on the spot due to which their average age becomes
number of observations in the set. 15.5 years. The number of persons initially going for picnic is:
i.e. On one day, 30 out of 36 students take test and their average A) 5 B) 10 C) 20 D) 30
was 80. On another day, the rest 6 students take test and their
average is 92. What was the average for the entire class ? 8) In a school with 600 students, the average age of the, boys is
The average of entire class = 12 years and that of the girls is 11 years.- If the average age of
the school is 11 years 9 months, then the number of girls in the
school is :
A) 150 B) 250 C) 350 D) 450

Ex.4: 9) The average price of three items of furniture is Rs. 15000. If


Find the average of first 60 natural Numbers? their prices are in the ratio 3:5:7, the price of the cheapest item
Sol.: Sum of first N natural Number in is :
A) Rs. 9000 B) Rs. 15000 C) Rs. 18000
D) Rs. 21000

Average of first N Natural No.: = 10) The average of 20 numbers is zero. Of them, at the most,
how many may be greater than zero?
A) 0 B) 1 C) 10 D) 19

11) The average age of boys in a class is 16 years and that of


Ex.5: the girls is 15 years. The average age for the whole class is
The average of 27 results is 16. The average of first 13 of them A) 15 years B) 15.5 years C) 16 years
is 15 and that of last 13 in 14. Find the fourteenth result. D) Cannot be computed
Sol.:14th result = Sum of 27 result – Sum of 26 Result
= 27 × 16 – [(13×15) + (13×14)]→ 432 – (195 + 182) = 55 12) The average of first five multiples of 3 is :
A) 9 B) 10 C) 8 D) 11
Practice Set 1
13) The average age of 30 boys of a class is equal to 14 years.
1) A student was asked to find the arithmetic mean of the When the age of the class teacher is included the average
numbers 3, 11, 7, 9, 15, 13, 8, 19, 17, 21, 14 and x. He found the becomes 15 years. Find the age of the class teacher?
mean to be 12. What should be the number in place of x? A) 40 B) 43 C) 45 D) 47
A) 3 B) 7 C) 17 D) 31 E) None of these

2) The average age of 36 students in a group is 14 years. when 14) In the first 10 overs of a cricket game, the run rate was only
teacher's age is included to it, the average increases by one. 3, 2. What should be the run rate in the remaining 40 overs to
What is the teacher's age in years? reach the target of 282 runs?
A) 31 B) 36 C) 51 A) 6.25 B) 6.5 C) 6.75 D) 7
D) None of these
15) The average age of husband, wife and their child 3 years
3) After replacing an old member by a new member, it was ago was 27 years and that of wife and the child 5 years ago was
found that the average age of five members of a club is the same 20 years. The present age of the husband is :
as it was 3 years ago. What is the difference between the ages of A) 35 years B) 40 years C) 50 years
the replaced and the new member? D) None of these
A) 2 years B) 4 years C) 8 years
D) 15 years 16) Distance between two stations A and B is 778 km. A train
covers the journey from A to B at 84 km per hour and returns
4) Find the average of all the prime number between 30 and back to A with a uniform speed of 56km per hour. Find the
50? average speed of the train during the whole journey?
A) 38 B) 38.8 C) 39 D) 39.8 A) 67.0 km /hr B) 67.2 km /hr
E) None of these C) 69.0 km /hr D) 69.2 km /hr

5) The average of 11 results is 50. If the average of first 6 17) The average of six numbers is X and the average of three of
results is 49 and that of last 6is 52, find the sixth result? these is Y.If the average of the remaining three is z, then
A) 50 B) 56 C) 60 D) 64 A) x = y + z B) 2x = y + z
E) None of these C) x = 2y + 2z D) None of these

For details on our programs: - Contact: +91 9930202329, +91, 8291924300 or visit www.campuscredentials.com (33)
18) Find the average of first 20 multiple of 7? 28) A motorist travel to a place 150 km away at an average
A) 71.5 B) 73.5 C) 75.5 D) 77.5 speed of 50 km/hr and returns ar 30 km/hr. His average speed
E) None of these for the whole journey in km/hr is
A) 35 B) 37 C) 37.5 D) 40
19) Of the three numbers, the average of the first and the
second is greater than the average of the second and the third 29) The Average of marks obtained by 120 candidates in a
by 15. What is the difference between the fire and the third of certain examination is 35. If the average marks of passed
the three numbers? candidates is 39 and that of failed candidates is 15, what is the
A) 15 B) 45 C) 60 number of candidates who passed the examination?
D) Data inadequate E) None of these A) 95 B) 100 C) 105 D) 110
E) None of these
20) There are two sections A and B of a class, consisting of 36
and 44 students’ respectively. If the average weight of section A 30) The average of all odd numbers upto 100 is:
is 40kg and that of section B is 35kg, find the average weight of A) 49 B) 49.5 C) 50 D) 51
the whole class.
A) 30 kg B) 35 kg C) 42.5 kg Practice Set 2:
D) 37.25 kg
1) The average salary of the entire staff in an office is Rs. 140
21) A batsman in his 17th innings makes a score of 85, and per month. The average salary of officers is Rs. 440 and that of
thereby increases his average by 3. What is his average after 17 Assistants is Rs. 130. If the number of officers is 16, then find
innings? the number of Assistance in the office.
A) 35 B) 37 C) 39 D) 40 A) 480 B) 505 C) 510 D) 435
E) None of these
2) There were 40 students in a hostel. If the number of students
22) A car owner buys petrol at Rs 7.50, Rs. 8 and Rs. 8.50 per increases by 8, the expenses of the mess increase by Rs. 48 per
litre for three successive years. What approximately is the day while the average expenditure per head diminishes by Rs 2.
average cost per litre of petrol if he spends Rs. 4000 each year? Find the original expenditure of the mess.
A) Rs. 7.98 B) Rs. 8 C) Rs. 8.50 D) Rs. 9 A) Rs. 700 B) Rs. 710 C) Rs. 680
D) Rs. 720
23) Of the three numbers, the first is twice the second and the
second is twice the three. The average of the reciprocal of the 3) Find the average of first 71 natural numbers.
numbers is 7/72. The numbers are : A) 30 B) 31 C) 36 D) 34
A) 16, 8, 4 B) 20, 10, 5 C) 24, 12, 6
D) 36, 18, 9 4) The average age of students of a class is 17.4 years. The
average age of boys in the class is 17.9 years and that of the
24) Distance between two stations A and B is 778km. A train girls is 16.8 years. The ratio of the number of boys to the
covers the journey from A to B at 84km per hour and returns number of girls in the class is :
back to A with a uniform speed of 56km per hour. Find the A) 6 : 5 B) 2 : 3 C) 6 : 3 D) 5 : 3
average speed of train during the whole journey.
A) 60 km/hr B) 30.5 km/hr C) 57 km/hr 5) If average of 7 consecutive even numbers is 21, what is the
D) 67.2 km/hr difference between the smallest and the largest numbers ?
A) 10 B) 11 C) 12 D) 13
25) 1/3 rd of certain journey is covered at the rate of 25 km/hr,
1/4 th at the rate of 30 km/hr and rest at 50 km/hr. Find the 6) The average weight of a class of 34 students is 45 kg. if the
average speed for the whole journey? weight of the teacher be included, the average rises by 400 g.
A) 32 1/3 km/hr B) 33 1/3 km/hr The weight of the teacher is :
C) 34 1/3 km/hr D) 35 1/3 km/hr A) 59 kg B) 50 kg C) 53 kg D) 55 kg
E) None of these
7) The average age of 7 men is increased by 2 years when two
26) Three years ago, the average age of A and B was 18 years. of them whose ages are 22 years and 26 years are replaced by
With C joining them, the average age becomes 22 years, How two new men. The average age of the two new men is :
old is C now? A) 22 years. B) 24 years C) 31 years
A) 24 years B) 27 years C) 28 years D) 30 years
D) 30 years
8) The average weight of 8 oranges increase by 22 gm if one of
27) In the first 10 overs of a cricket game, the run rate was only them weighing 124 gm is replaced by another. The weight of
3.2. What should be the run rate in the remaining 40 overs to new orange is
reach the target of 282 runs? A) 180 gm B) 200 gm C) 260 gm
A) 6.25 B) 6.5 C) 6.75 D) 7 D) 300 gm

For details on our programs: - Contact: +91 9930202329, +91, 8291924300 or visit www.campuscredentials.com (34)
9) A motorist travels to a place 170 km away at an average 20) The average age of the mother and her 4 children is 15
speed of 20 km/h and returns at 30 km/h. His average speed for years which is reduced by 6 years if the age of the mother is
the whole journey in km per hour is excluded. How old is the mother?
A) 35 B) 25.7 C) 37.5 D) 24 A) 40 years B) 42 years C) 48 years
D) 39 years
10) The average weight of 39 students is 38 kg. By the
admission of a new student, the average weight is reduced to 21) The average weight of 6 persons increases by 3.5 kg when a
37.5 kg. The weight of the new student is new person comes in place of one of them weighing 59 kg. What
A) 18 kg B) 37.5 kg C) 19 kg D) 21 kg. might be the weight of the new person?
A) 76 kg B) 80 kg C) 85 kg
11) The average temperature on Monday, Tuesday and D) Data inadequate
Wednesday was 51ºC and on Monday, Wednesday and
Thursday it was 50ºC. If on Thursday it was exactly 49ºC, then 22) A batsman had an average of 60 runs in 9 innings, but in
on Tuesday, temperature was the 10th inning, he was out on zero. What is the average after
A) 42ºC B) 46ºC C) 23ºC D) 52ºC 10th inning?
A) 51 B) 54 C) 53 D) 55
12) The average of the first and the second of three numbers is
20 more than the average of the second and the third of these 23) The average weight of 40 coins is 7 gm. If the weight of the
numbers. What is the difference between the first and the third bag be included the average weight increase by 0.07 gm. What
of these three numbers ? is the weight of the bag ?
A) 10 B) 20 C) 30 D) 40 A) 3.87 gm B) 5.87gm C) 9.87 gm
D) 7.87 gm
13) The average of 100 observations was calculated as 30. It
was found later on that one of the observations was misread as 24) Find the average of the consecutive even numbers given
76 instead of 46. the correct average is : below.
A) 40.3 B) 39.7 C) 29.7 D) 30.3 40, 42, 44, 46, 48, 50, 52, 54, 56, 58, 60, 62, 64
A) 50 B) 55 C) 52 D) 60
14) 2 years ago, the average age of a family of 5 members was
15 years. A baby having been born, the average age of the 25) The average age of 28 boys of a class is equal to 15 years.
family is the same today. The present age of the baby is : When the age of the class teacher is included the average
A) 3 year B) 1 ½ year C) 4 years becomes 16 years. Find the age of the class teacher.
D) 5 years. A) 45 years B) 42 Years C) 40 Years
D) 44 Years
15) The average weight of A, B and C is 35 kg. If the average
weight of A and B be 30 kg and that of B and C be 33 kg, then 26) The average weight of 5 men is increased by 4 kg when one
the weight of B is : of them who weighs 90 kg is replaced by another man. What is
A) 17 kg B) 20 kg C) 21 kg D) 31 kg the weight of the new man ?
A) 125 kg B) 110 kg C) 132 kg D) 142 kg
16) The average monthly income of L and M is Rs. 4040. The
average monthly income of M and N is Rs. 5240 and the 27) The average of marks obtained by 120 candidates in a
average monthly income of L and N is Rs. 4200. The monthly certain examination is 33. If the average marks of passed
income of L is : candidates is 37 and that of the failed candidates is 13, what is
A) Rs. 3500 B) Rs. 4000 C) Rs. 4050 the number of candidates who passed the examination?
D) Rs. 3000 A) 100 B) 105 C) 110 D) 115

17) The average of 40 numbers is 35. If two numbers, namely 45 28) A batsman in his 15th innings makes a score of 90 and
and 63 are discarded, the average of remaining numbers is thereby increases his average by 4. What is his average after 15
A) 36.5 B) 35 C) 34 D) 37.52 innings?
A) 27 B) 37 C) 30 D) 34
18. The average age of 28 students is 9 yr. If the age of their
teacher is included, it becomes 10 yr. The age of the teacher (in 29) A person divides his total route of journey into three equal
years) is : parts and decides to travel the three parts with speeds of 40, 30
A) 27 B) 31 C) 38 D) 40 and 15 km/hr respectively. Find his average speed during the
whole journey.
19) The average income of Mohan and Ram is Rs. 300. The A) 21 km/hr B) 22km/hr C) 23 kh/hr
average income of Raghu and Jaya is Rs. 350. The average D) 24 km/hr
income of all four persons is
A) Rs. 275 B) Rs. 225 C) Rs. 325 30) The average age of all the students of a class is 23 years.
D) Rs. 250 The average age of boys of the class is 25 years and that of the
girls is 20 years. If the number of girls in the class is 30, then
find the number of boys in the class.
A) 30 B) 45 C) 35 D) 40

For details on our programs: - Contact: +91 9930202329, +91, 8291924300 or visit www.campuscredentials.com (35)
Chapter 06 Ratio & Proportion CONTINUED PROPORTION :
When a, b, c are in continued proportion a : b = b : c. Here, ‘b’
RATIO : is called ‘geometric mean’ or ‘mean’ proportional between a
A ratio is a comparison of two numbers or quantities by and c.
division.
The ratio of ‘a’ to ‘b’ is written as a : b where ‘a’ is called the If …., then a, b, c, d, …. are said to be in continued
‘antecedent’ and ‘b’ is called the ‘consequent’ proportion.
While comparing two quantities in terms of ratio.
(i) the two quantities must be of the same kind.
(ii) the units of the two quantities must be the same
(iii) ratio does not have any unit of measurement. e.g., 6 is the geometric mean of 3 and x.
THE SIMPLEST FORM OF RATIO :  6² = 3  x  36 = 3x  x = 12.
The expression of a ratio, both of whose terms that do not have PROPERTIES OF PROPORTION
any common factor other than 1, is called the ratio in its
simplest form.

NOTE :
(i) As a/b is a real number, the operation of addition,
subtraction, multiplication and division can be performed just
like those on real numbers.
(ii) To compare the two ratios, apply the same method as in the
case of real numbers.

e.g. Compare
Consider 3  8 and 7  5

35 > 24 
DIVIDENDO :
(iii) The ratio b : a is the inverse ratio of a : b.
(iv) Property of equal ratio :

PROPORTION :
The equality between the two ratios is called proportion.

VARIATION :
 a : b : : c : d then the numbers a, b, c and d are
1. DIRECT VARIATION :
in proportion and they are called the first, the second, the third
If two quantities ‘a’ and ‘b’ are related such that any increase
and the fourth proportional, respectively . ‘a’ and ‘d’ are called
or decrease in ‘b’ produces a proportionate increase or
the extremes, while b and c are called the means.
decrease in ‘a’, or vice versa, then the two quantities are said to
e.g.,
be in direct proportion. the quantities ‘a’ and ‘b’ are directly
related and their relationship is expressed as a  b. In this case
 4, 12, 5 and 15 are in proportion ; where 4 and 15 a = kb, where ‘k’ is non – zero constant.
are extremes and 12 and 5 are means. e.g., If 14 oranges cost Rs,. 28, then the cost of 5 oranges can be
found as follows :

When,
i.e., Product of extremes = Product of means
e.g. the fourth proportional to 3, 4, 6 can be found as follows :
Let x be the fourth proportional.
i.e., 3, 4, 6 and x are in proportion
 3x = 4  6  x = 24/3  x = 8

If ….. , then a, b, c, d, e, f, …….. are said to be


in proportion. 2. INVERSE VARIATION :

For details on our programs: - Contact: +91 9930202329, +91, 8291924300 or visit www.campuscredentials.com (36)
The values of a’ and ‘b’ vary (change) in such a way that the So, we have, y  x.
product of corresponding values of ‘a’ and ‘b’ is constant. Such  y = kx ……. (k is a constant)
a relation between a and b is known as inverse variation and Now,. y = 160,. when x = 20

the relationship is expressed as a  i.e., ab =


k, where ‘k’ is a non zero constant.  y = 8x is the equation of variation
e.g., Ram drives a car at a speed of 30 kmph for 2 hours. If he  Putting x = 5 in the equation, we get y = 8  5 = 40.
has to cover the same distance in 1 hour, his speed can be found  The cost of 5 kg of sugar is Rs. 40.
as follows :
Example 5 :
16 workers take 3 days to pack 30 kgs of rice. How many days
will be taken by 20 workers to pack 36 kgs of rice ?
Solutions :
Let the number of days required be x. workers  work

SOLVED EXAMPLES

Example 1 :
If a : b = 3 : 7 and b : c = 2 : 5, find a : b : c. Practice set 1:
Solution : a : b = 3 : 7
1) The third proportional to 0.36 and 0.48 is :
A) 0.64 B) 0.1728 C) 0.42 D) 0.94

2) If a : b = 5 : 9 and b : c = 4: 7, find a : b : c?
A) 20:36:63 B) 20:63:36 C) 63:20:36
D) 36:63:20 E) None of these
Example 2 :
3) Which of the following ratios is greatest ?
A) 7 : 15 B) 15 : 23 C) 17 : 25 D) 21 : 29

Solution : 4) If 10% of x = 20% of y, then x : y is equal to:


A) 1 : 2 B) 2 : 1 C) 5 : 1 D) 10 : 1

Required expression can be written 5) If x : y = 3 : 4, find (4x + 5y) : (5x - 2y)?


A) 23/7 B) 32/7 C) 23/7 D) 21/6
E) None of these

6) The ratio of 43.5 : 25 is same as :


A) 2 : 1 B) 4 : 1 C) 7 : 5 D) 7 : 10

Example 3 : 7) If A : B = 2 : 3, B : C = 4 : 5 and C : D = 6 : 7, then A : B :


C : D is:
A) 16 : 22 : 30 : 35 B) 16 : 24 : 15 : 35
C) 16 : 24 : 30 : 35 D) 18 : 24 : 30 : 35
Solution :
8) A sum of money is to be distributed among A, B, C, D in the
proportion of 5:2:4:3. If C gets Rs. 1000 more than D, what is
B's share?
A) Rs. 500 B) Rs. 1500 C) Rs. 2000
D) None of these

9) Find the fourth proportional to 4, 9, 12?


Example 4 : A) 24 B) 21 C) 27
The cost of sugar is directly proportional to its weight. 20 kg of D) 36 E) None of these
sugar costs Rs. 160. Find the cost of 5 kg of sugar.
Solution : 10) If (4x2 - 3y2) : (2x2 + 5y2) = 12 : 19, then (x : y) is :
Let us denote the cost of sugar by y and its weight x. A) 2 : 3 B) 1 : 2 C) 3 : 2 D) 2 : 1

For details on our programs: - Contact: +91 9930202329, +91, 8291924300 or visit www.campuscredentials.com (37)
24) A sum of Rs.312 was divided among 100 boys and girls in
𝟏 𝟏 𝟏 such a way that the boy gets Rs.3.60 and each girl Rs.2.40 the
11) The sides of a triangle are in the ratio : : and its perimeter
𝟐 𝟑 𝟒
number of girls is
is 104 cm. The length of the longest side is :
A) 35 B) 40 C) 45 D) 50
A) 52 cm B) 48 cm C) 32 cm D) 26 cm
25) A bag contains 50 p, 25 p and 10 p coins in the ratio 5: 9: 4,
12) If 0.75:x::5:8, then x is equal to:
amounting to Rs. 206. Find the number of coins of 25 paisa
A) 1.12 B) 1.20 C) 1.25 D) 1.30
type?
A) 200 B) 360 C) 160 D) 320
13) Two number are in the ratio 3 : 5. If 9 is subtracted from
E) None of these
each, the new numbers are in the ratio 12 : 23. The smaller
number is:
26) The prices of a scooter and a T.V. are in the ratio 7 : 5. If
A) 27 B) 33 C) 49 D) 55
the scooter costs Rs. 8000 more than a T.V. set, then the price of
a T.V. set is :
14) Divide Rs. 672 in the ratio 5 : 3?
A) Rs. 20,000 B) Rs. 24,000
A) 420 and 252 B) 410 and 246 C) 400 and 240
C) Rs. 28,000 D) Rs. 32,000
D) 440 and 264 E) None of these
𝟒
𝑨 𝑩 𝑪 27) A and B together have Rs. 1210. If of A's amount is equal
15) If A : B : C = 2 : 3 : 4, then, : : is equal to: 𝟏𝟓
𝑩 𝑪 𝑨 𝟐
A) 4 : 9 : 16 B) 8 : 9 : 12 C) 8 : 9 : 16 D) 8 : 9 : to of B's amount, how much amount does B have?
𝟓
24 A) Rs. 460 B) Rs. 484 C) Rs. 550 D) Rs. 664

𝟏 𝟐 𝟑 28) A and B are two alloys of gold and copper prepared by


16) If Rs. 782 be divided into three parts, proportional to : : ,
𝟐 𝟑 𝟒
mixing metals in the ratio 7 : 2 and 7 : 11 respectively. If equal
then the first part is:
quantities of the alloys are melted to form a third alloy C, the
A) Rs. 182 B) Rs. 190 C) Rs. 196
ratio of gold and copper in C will be:
D) Rs. 204
A) 5 : 7 B) 5 : 9 C) 7 : 5 D) 9 : 5
17) Divide Rs. 1162 among A, B, C in the ratio 35 : 28 : 20?
29) The ratio of the number of boys and girls in a college is 7:8.
A) 480 , 382 and 300 B) 490, 280 and 382
If the percentage increase in the number of boys and girls be
C) 490 , 392 and 280 D) 392 , 490 and 280
20% and 10% respectively, what will be the new ratio?
E) None of these
A) 8:9 B) 17:18 C) 21:22
D) Cannot be determined
18) Zinc and copper are melted together in the ratio 9 : 11.
What is the weight of melted mixture, if 28.8 kg of zinc has been
30) Three containers have their volumes in the ratio 3 : 4 : 5,
consumed in it?
They are full of mixtures of milk and water. The mixtures
A) 58 kg B) 60 kg C) 64 kg D) 70 kg
contain milk and water in the ratio of (4 : 1), (3 : 1) and (5 : 2)
respectively. The contents of all these three containers are
19) The ages of A and B are in the ratio of 3 : 1. Fifteen years
poured into a fourth container. The ratio of milk and water in
hence, the ratio will be 2 : 1. Their present ages are :
the fourth container is :
A) 30 years, 10 years B) 45 years, 15 years
A) 4 : 1 B) 151 : 48 C) 157 : 53 D) 5 : 2
C) 21 years, 7 years D) 60 years, 20 years
Practice Set 2:
20) The fourth proportional to 5, 8, 15 is:
A) 18 B) 24 C) 19 D) 20
1) The ratio compounded of 3:2, 4:9, 6:5, 10:8 is :
A) 1 : 1 B) 2 : 1 C) 1 : 2
21) A mixture contains alcohol and water in the ratio 4 : 3. If 5
D) None of these
litres of water is added to the mixture, the ratio becomes 4: 5.
Find the quantity of alcohol in the given mixture?
A) 10 B) 12 C) 15 D) 18 2) Divide Rs. 858 in the ratio of Find first share:
E) None of these A) Rs. 296 B) Rs. 288 C) Rs. 196
D) None of these
22) Gold is 19 times as heavy as water and copper is 9 times as
heavy as water. In what ratio should these be mixed to get an 3) If A : B = 7 : 3, B : C = 6 : 10 & C : D = 15 : 17 then find A
alloy 15 times as heavy as water? :B:C:D
A) 1 : 1 B) 2 : 3 C) 1 : 2 D) 3 : 2 A) 6 : 7 : 9 : 12 B) 9 : 12 : 15 : 17
C) 21 : 4 : 8 : 15 D) 21 : 9 : 15 : 17
23) If a carton containing a dozen mirrors is dropped, which of
the following cannot be the ratio of broken mirrors to unbroken 4) If 5A:6B is the duplicate ratio of 5A – C : 6B-C then :
mirrors? A) c² = 6AB B) c2= 30AB C) 2C = 3AB
A) 2 : 1 B) 3 : 1 C) 3 : 2 D) 7 : 5 D) None of these

For details on our programs: - Contact: +91 9930202329, +91, 8291924300 or visit www.campuscredentials.com (38)
5) Ratio of boys to the girls in a class is 6 : 7. which of the share of Shyam and Mohan is four times that of Ram. What is
following cannot be the number of students in the class ? Ram’s share?
A) 26 B) 52 C) 104 D) 67 A) Rs. 3000 B) Rs. 5000 C) Can’t be determined
D) None of these
6) Find the third proportional to the numbers 4 and 16.
A) 64 B) 8 C) 25 D) 30 18) The ratio between two numbers is 4:5, their LCM is 260.
Determine the numbers?
7) A dog chase a cat and takes 6 leaps for every 9 leaps of the A) 60,45 B) 52,60 C) 52,65 D) 48, 64
cat, but 3 leaps of the dog are equal to 5 leaps of the cat.
Compare the speeds of the dog and the cat. 19) Two numbers are in the ratio 3 : 5. if each number is
A) 9 : 10 B) 10 : 9 C) 24 : 25 D) 25 : 24 increased by 7, the ratio becomes 2 : 3. The numbers are
A) 5, 3 B) 18, 21 C) 21, 35 D) 20, 34
8) Divide 150 into three parts such that the first part is 15 more
than the second and the ratio between 1 st and 3rd is 2 : 7, then 20) A mixture of 680 ml contains milk and water in the ratio 10
find the ratio between three parts. : 7. How much more water is added to get a new mixture
A) 38 : 23 : 113 B) 30 : 15 : 105 containing milk and water in the ratio 5 : 4 ?
C) 40 : 30 : 100 D) 34 : 30 : 119 A) 30 ml B) 20 ml C) 80 ml D) 40 ml

9) What should be subtracted from each of the numbers 58, 76, 21) The sum of three no. is 126. If the ratio b/w 1 st & 2nd be 3 : 4
73 and 96, so that the remainders are in continued proportion? & that between 2nd & 3rd be 5 : 7. Find the first no.
A) 9 B) 7 C) 4 D) None of these A) 20 B) 25 C) 30 D) 35

10) x is inversely proportional to the square root of the cube of 22) Two Books cost as much as 6 Pens, 8 pens as much as 10
y. When x=2, y= 16. What is the value y when x=1024? Pencils, 15 Pencils as much as 20 rubbers,10 rubbers as much
A) ½ B) ¼ C) 4 D) 2 as 50 Sharpeners. If the price of one Sharpener is Rs. 10. How
much will one Book cost.
11) If p/q = 2/5, then find the value of the expression (7p – 2q) / A) Rs. 270 B) Rs. 250 C) Rs. 350 D) Rs. 370
(7p + 3q) :
A) 3/29 B) 5/29 C) 4/29 D) 5/58 23) In a Stationery Shop, the ratio of the no. of Books, Pen &
Pencil is 2 : 7 : 9. If the difference between the numbers of Pen
12) A melon is cut into two pieces in the ratio of 5 : 6 by weight. & Pencil is a multiple of 3 as well as 7, what is the minimum no.
The bigger of the two is further cut in the ratio of 6 : 7 by of Book, Pen and Pencil in the Shop:
weight. Find the ratio of each of the three pieces A) 415 B) 289 C) 378
A) 42 : 36 : 65 B) 65 : 36 : 42 D) Can’t be determined
C) 36 : 65 : 42 D) 30 : 60 : 40
24) If A : B = 8 : 3, B : C = 6 : 7, find A : B : C
13) If A : B = 2 : 3, B : C = 5 : 14, C : D = 7 : 10 & D : E = 8 : A) 12 : 20 : 35 B) 16 : 20 : 28
7 then find A:E C) 16 : 6 : 7 D) None of these
A) 1 : 4 B) 4 : 21 C) 3 : 16
D) None of these 25) The ratio of the prices of two horses was 18 : 25. Three year
later when the price of the first has increased by 10% & that of
14) A bag contains Rs. 6500 in the form of one rupee, 50 paise the second by Rs. 392, the ratio of the prices becomes 3 : 5.
& 25 paise coins in the ratio of 5 : 6 : 8. find the total number of Find the original prices of the two horses:
25 paise coins in the bag. A) 848, 1219 B) 882, 1225
A) 650 B) 6000 C) 5200 D) 1300 C) Can’t be determined D) None of these

15) Sunil’s Salary is 7/6 of Dhiraj’s salary, and Dhiraj’s salary 26) The monthly income of two persons are in the ratio 6 : 8 &
is 6/5 of Vijay’s salary, while Sneha’s salary is equal to the sum their expenditure are in the ratio 14 : 19. If each saves Rs. 80
of salaries of Sunil, Dhiraj and Vijay. If salary of Vijay is Rs per month, find sum of their monthly incomes
1500, What is the salary of Sneha? A) 900 B) 1400 C) 2800 D) 2600
A) 5500 B) 6500 C) 5400 D) 5800
27) Divide Rs. 616 among A,B & C such that for 3 times A’s
16) The sum of the ages of the 4 members of Jain family is 143 share equal to 6 times B’s share is equal to 9 times of C share.
years. 6 years ago the ages of the 4 members Amit, Aman, Mrs. Find the share of A.
Jain and Mr. Jain were in the ratio of 2 : 3 : 5 : 7. after how A) Rs.168 B) Rs. 112 C) Rs. 369 D) Rs. 336
many years would Aman be as old as the present age of his
Father?
28) If then the value of 3x – 7y + 6z/ 3y is :
A) 10 Yrs B) 17 Yrs C) 30 Yrs D) 28 Yrs
A) 17/6 B) 15/2 C) 16/15 D) 18/19
17) An amount of money is to be distributed among Ram, Shyam
and Mohan in the ratio 6 : 13 : 17 respectively. If the total 29)

For details on our programs: - Contact: +91 9930202329, +91, 8291924300 or visit www.campuscredentials.com (39)
A) 2 B) 3 C) C/2 D) None of these Chapter 07: Mixtures & Allegation
30) A vessel contains liquids A & B in ratio 6 : 7, if 26 lit. of MIXTURE :
mixture is removed & the same quantity of liquid B is added
then the ratio becomes 9 : 17. What quantity does the vessel When two or more different ingredients are mixed together, the
hold : resulting formation is known as mixture.
A) 123 lit B) 98 lit C) 40 lit D) 104 lit
ALLEGATION:

Allegation is a rule which states that, “when different quantities


of the same or different ingredients, of different prices are
mixed together to produce a mixture of a mean price, the ratio
of their quantities is inversely proportional to the difference in
their price from the mean price”. We can say that,

We can diagrammatically represent it as :

• Consider a vessel contains only ‘a’ unit of ingredient A. From


this b is taken out and replaced by in equal amount of
ingredient B, and the process is repeated ‘n’ times in all then,

SOLVED EXAMPLES
Ex. 1 : In what proportion should one variety of oil at Rs. 3.30
per litre be mixed with another at Rs. 4.00 per litre to get a
mixture worth Rs. 3.70 per litre ?
Solution :

They must be mixed in the ratio of 3:4.

For details on our programs: - Contact: +91 9930202329, +91, 8291924300 or visit www.campuscredentials.com (40)
Ex. 2 : There are two types of rice. Type A costs Rs. 14 per kg
and Type B costs Rs. 18 per kg. If A and B are mixed in the ratio Practice Set 1:
5 : 3, what is the cost price per kg of the resultant mixture ? 1) In what ratio must water be mixed with milk costing Rs.12
Solution : per litre to obtain a mixture worth of Rs.8 per litre?
A) 1 : 2 B) 2 : 1 C) 4 : 3 D) 5 : 3

2) In what ratio must be a grocer mix two verities of tea worth


Rs. 60 a kg and Rs. 65 a Kg so that by selling the mixture at Rs.
68.20 a Kg he may gain 10% ?
A) 3 : 4 B) 3 : 2 C) 3 : 5 D) 4 : 5

3) In what ratio must a grocer mix two varieties of pulses


costing Rs. 15 and Rs. 20 per kg respectively so as to get a
mixture worth Rs. 16.50 kg?
A) 3 : 7 B) 5 : 7 C) 7 : 3 D) 7 : 5

4) In what ratio must rice at Rs.9.30 per Kg be mixed with rice


at Rs. 10.80 per Kg so that the mixture be worth Rs.10 per Kg ?
A) 8 : 7 B) 7 : 8 C) 8 : 9 D) 9 : 8

5) Tea worth Rs. 126 per kg are mixed with a third variety in
the ratio 1: 1 : 2. If the mixture is worth Rs. 153 per kg, the
price of the third variety per kg will be
Ex. 3 : A) Rs. 169.50 B) Rs.1700 C) Rs. 175.50
A vessel contains 66 litres of a mixture of alcohol and water in D) Rs. 180
the ratio 7 : 4. How much water should be mixed with it so that
the alcohol to water ratio becomes 21 : 20? 6) A container contains 40 litres of milk. From this container 4
Solution : Let x litres of water be added to the mixture. litres of milk was taken out and replaced by water. This process
was repeated further two times. How much milk is now
contained by the container?
A) 26.34 litres B) 27.36 litres
C) 28 litres D) 29.l6 litres
 16 litres of water should be added.
7) Tea worth Rs. 126 per kg and kg and Rs. 135 per kg are
Ex. 4 : In what ratio must 45% alcohol be mixed with 60% mixed with a third variety in the ratio 1 : 1 : 2. If the mixture is
alcohol to get a mixture of 55% alcohol strength ? worth Rs. 153 per kg, the price of the third variety per kg will be
Solution : :
A) Rs. 169.50 B) Rs. 170
C) Rs. 175.50 D) Rs. 180

8) Two vessels A and B contain spirit and water in the ratio 5 :


2 and 7 : 6 respectively. Find the ratio in which these mixture
be mixed to obtain a new mixture in vessel C containing spirit
and water in the ration 8 : 5 ?
A) 4 : 3 B) 3 : 4 C) 5 : 6 D) 7 : 9

9) A merchant has 1000 kg of sugar, part of which he sells at


8% profit and the rest at 18% profit. He gains 14% on the
whole. The quantity sold at 18% profit is:
A) 400 kg B) 560 kg C) 600 kg
Ex. 5 : D) 640 kg
A container contained 100 kg of milk. From this container 10 kg
of milk was taken out and replaced by water. This process was 10) A can contains a mixture of two liquids A and B in the ratio
further repeated two times. How much milk does the container 7 : 5. When 9 litres of mixture are drawn off and the can is filled
have now ? with B, the ratio of A and B becomes 7 : 9. How many litres of
Solution : Amount of liquid left after n operations, when the liquid A was contained by the can initially?
container originally contains a units of liquid from which b A) 10 B) 20 C) 21 D) 25
units are taken out each time are
11) A can contains a mixture of two liquids A and B in the ratio
7 : 5. When 9 litres of mixture are drawn off and the can is filled
with B, the ratio of A and B becomes 7 : 9. How many litres of
liquid A was contained by the can initially?

For details on our programs: - Contact: +91 9930202329, +91, 8291924300 or visit www.campuscredentials.com (41)
A) 10 B) 20 C) 21 D) 25 24) In what ratio must tea Rs. 62 per kg be mixed with tea at Rs.
72 per kg so that the mixture must be worth Rs. 64.50 per kg?
12) In what ratio must rice at Rs 9.30 per Kg be mixed with rice A) 3 : 1 B) 3 : 2 C) 4 : 3 D) 5 : 3
at Rs 10.80 per Kg so that the mixture be worth Rs 10 per Kg?
A) 6:5 B) 8:7 C) 3:7 D) 6:1 25) In what ratio must tea at Rs.62 per Kg be mixed with tea at
Rs. 72 per Kg so that the mixture must be worth Rs. 64.50 per
13) Find the ratio in which rice at Rs. 7.20 a kg be mixed with Kg?
rice at Rs. 5.70 a kg to produce a mixture worth Rs. 6.30 a kg. A) 3 : 1 B) 3 : 2 C) 4 : 3 D) 5 : 3
A) 1 : 3 B) 2 : 3 C) 3 : 4 D) 4 : 5
26) In what ratio must water be mixed with milk to gain 16 %
14) In what ratio must water be mixed with milk costing Rs. 12 on selling the mixture at cost price?
per litre to obtain a mixture worth of Rs. 8 per litre? A) 1 : 6 B) 6 : 1 C) 2 : 3 D) 4 : 3
A) 1 : 2 B) 2 : 1 C) 2 : 3 D) 3 : 2
27) 8 litres are drawn from a cask full of wine and is then filled
15) A milk vendor has 2 cans of milk. The first contains 25% with water. This operation is performed three more times. The
water and the rest milk. The second contains 50% water. How ratio of the quantity of wine now left in cask to that of the water
much milk should he mix from each of the containers so as to is 16 : 65. How much wine the cask hold originally ?
get 12 litres of milk such that the ratio of water to milk is 3 : 5 ? A) 18 litres B) 24 litres C) 32 litres
A) 4litres, 8 litres B) 6litres, 6 litres D) 42 litres
C) 5litres, 7 litres D) 7litres, 4 litres
28) In what ratio must wheat at Rs.3.20 per kg be mixed with
16) In what proportion must rice at Rs. 3.10 per kg be mixed wheat at Rs.2.90 per kg so that the mixture be worth Rs.3.08 per
with rice at Rs. 3.60 per kg so that the mixture be worth Rs. 3.25 kg?
per kg? A) 3 : 4 B) 2 : 3 C) 3 : 2 D) 4 : 3
A) 3 : 7 B) 5 : 3 C) 3 : 5 D) 7 : 3
29) How many kilogram of sugar costing Rs. 9 per kg must be
17) In what ratio must a grocer mix two varieties of tea worth mixed with 27 kg of sugar costing Rs. 7 per kg so that there may
Rs. 60 a kg and Rs. 65 a kg so that by selling the mixture at Rs. be a gain of 10% by selling the mixture at Rs. 9.24 per kg?
68.20 a kg he may gain 10%? A) 36 kg B) 42 kg C) 54 kg D) 63 kg
A) 3 : 2 B) 3 : 4 C) 3 : 5 D) 4 : 5
30) Two vessels A and B contain milk and water mixed in the
18) Tea worth Rs. 126 per kg and Rs. 135 per kg are mixed with ratio 8 : 5 and 5 : 2 respectively. The ratio in which these two
a third variety in the ratio 1 : 1 : 2. If the mixture is worth Rs. 𝟑
mixtures be mixed to get a new mixture containing 69 %
𝟏𝟑
153 per kg, the price of the third variety per kg will be:
milk, is:
A) Rs. 169.50 B) Rs. 170 C) Rs. 175.50
A) 2 : 7 B) 3 : 5 C) 5 : 2 D) 5 : 7
D) Rs. 180

19) Two vessels A and B contain spirit and water mixed in the
ratio 5 : 2 and 7 : 6 respectively. Find the ratio in which these
mixture be mixed to obtain a new mixture in vessel C containing
spirit and water in the ratio 8 : 5 ?
A) 4 : 3 B) 3 : 4 C) 5 : 6 D) 7 : 9

21) A merchant has 1000 kg of sugar part of which he sells at


8% profit and the rest at 18% profit. He gains 14% on the
whole. The Quantity sold at 18% profit is
A) 400 kg B) 560 kg C) 600 kg
D) 640 kg

22) The cost of Type 1 rice is Rs. 15 per kg and Type 2 rice is
Rs. 20 per kg. If both Type 1 and Type 2 are mixed in the ratio
of 2 : 3, then the price per kg of the mixed variety of rice is:
A) Rs. 18 B) Rs. 18.50 C) Rs. 19
D) Rs. 19.50

23) A can contains a mixture of two liquids A and B is the ratio


7 : 5. When 9 litres of mixture are drawn off and the can is filled
with B, the ratio of A and B becomes 7 : 9. How many litres of
liquid A was contained by the can initially?
A) 10 B) 20 C) 21 D) 25

For details on our programs: - Contact: +91 9930202329, +91, 8291924300 or visit www.campuscredentials.com (42)
Chapter 8: Time & Work = 2700 / 600 = 4½ days
Two persons A & B, working together, can complete a piece of
Time= Work done/Efficiency work in x days. If A, working alone, can complete the work in y
days, then B, working alone, will complete the work in
When work is same. ⇒xy / (y - x)
Time∝1/Efficiency
Explanation
If A can do a piece of work in n days.
Then, per day working efficiency of A = 1/n ⇒ A + B’s 1 day work = 1/x
A’s 1 day work = 1/y
If working efficiency of A & B is → x : y. B’s 1 day work = 1/x-1/y
Then, the time taken by A & B to finish the work is in the ratio = (y - x) / yx
→y:x B will complete the work = yx / (y - x)

e.g. If A does three times faster work than ‘B’, then ratio of Q. A and B working together take 15 days to complete a piece of
work done by A and B is 3 : 1. Then work. If A alone can do this work in 20 days, how long would B
Ratio of time taken by A & B = 1 : 3 take to complete the same work?

If A can do a piece of work is x days and B can do a piece of Sol. x = 15, y = 20


work in 4 days, then both of them working together will do the B will complete the work in = (15 × 20) / 5
same work in = 60 days

xy/(x+y) days If A & B working together, can finish a piece of work is x days,
B & C in 4 days, C & A in z days. Then, A + B + C working
Explanation together will finish the job is

⇒ A’s 1 day’s work = 1/x ⇒2xyz / (xy + yz + zx)


B’s 1 day’s work = 1/y
(A + B)’s 1 day work = 1/x+1/y =(x + y)/xy Explanation
A + B will complete the work in = xy/(x + y)
⇒ A + B’s 1 day work = 1/x
Q. A can finish a piece of work by working alone in 6 days and B + C’s 1 day work = 1/y
B, while working alone, can finish the same work in 12 days. If C + A’s 1 day work = 1/z
both of them work together, then in how many days, the work [(A + B) + (B + C) + (C + A)]’s 1 day’s work
will be finished? =1/x+1/y+1/z
= (yz + xz + xy) / xyz
Sol. x = 6, y = 12 2 (A + B + C)’s 1 day work = (xy + yz + xz)/xyz
Working together A + B will complete the work in = xy/(x + y) A + B + C’s 1 day work = (xy + yz + xz)/2xyz
= (6 × 8)/18 A + B + C working together will complete the work in
= 4 days =2xyz / (xy + yz + xz)

If A, B & C will working alone, can complete a work in x, y and Q. A and B can do a piece of work in 12 days, B and C in 15
z days, respectively, then they will together complete the work in days, C and A in 20 days. How long would they take to complete
xyz / (xy + yz + zx) the full work together?
Explanation
Sol. x = 12 days, y = 15 days, z = 20 days
⇒ A’s 1 day work = 1/x A + B + C = (2 × 12 × 15 × 20) / (180 + 300 + 240)
B’s 1 day work = 1/y =7200 / 720 = 10 days
C’s 1 day work = 1/z
(A + B + C)’s 1 day work = 1/x + 1/y + 1/z = (yz + xz + xy) / If A can finish a work in x days and B is k times efficient than A,
xyz then the time taken by both A and B, working together to
(A + B + C) will complete the work in complete the work is
=xyz / (yz + xz + xy)
x/(1 + k)
Q. A, B and C can complete a piece of work in 10, 15 and 18
days, respectively. In how many days would all of them Explanation
complete the same work working together?
⇒ Ratio of working efficiency, A & B = 1 : k
Sol. x = 10 days, y = 15 days & z = 18 days Ratio of Time taken = k : 1
The work will be completed in k → x days
= (10 × 15 × 18) / (10 × 15 + 15 × 18 + 18 × 10) 1r → x/k days
A → x days
For details on our programs: - Contact: +91 9930202329, +91, 8291924300 or visit www.campuscredentials.com (43)
B → x/k days hrs more to complete the job than A and B worked together.
1 day work of A = 1/x What time would they take if both A and B worked together?
1 day work of B = k/x days
(A + B)’s 1 day work = 1/x + k/x = (k + 1)/x Sol. a = 8, b = 9/2
(A + B) will complete the work is = x/(k + 1) A + B will take = √(8 × 9 / 2)
=√36
Q. Harbans Lal can do a piece of work in 24 days. If Bansi Lal = 6 days
works twice as fast as Harbans Lal, how long would they take to
finish the work working together? Points to remember:
1) If a man can do a piece of work in n days, work done by him
Sol. x = 24, k = 2
Working together they will complete the work in = 24/(1 + 2) in one day = part of total work or he will finish work in
=24/3=8 days one day.

If A & B working together can finish a work in x days & B is k 2) If a man completes work in one day, he will complete
times efficient than A, then the time taken by, the entire work in n days.
3) If A can complete a piece of work in X days and B can
A working Alone will take ⇒ (k + 1) x complete the same work in Y days, both A and B working
B working Alone will take ⇒ ((k + 1) / k) x
together can finish the same work in days.
Explanation 4) If A is thrice as good as workman as B or A can work three
times faster than B, the ratio of work done by A and B for the
⇒ Efficiency Ratio → 1 : k same duration of time will be = 3 : 1. And the ratio of time
Time Ratio → k : 1 taken by A and B to finish the same amount of work will be = 1:
A’s 1 day work = 1/k 3.
B’s 1 day work = 1 5) A, B and C can do a work in D1, D2 and D3 days
(A + B)’s 1 day work = 1/x respectively. If they work for X1, X2 and X3 days respectively;
1/k + 1 = 1/x
(k + 1)/k = 1/x o Work done by A in X1 days =
k = (k + 1) x
A alone working together will take ⇒ (k + 1) x days o Work done by B in X2 days =
1 ratio = ((k + 1) x)/k
B Alone working Alone will take o Work done by C in X3 days =
⇒((k + 1) x)/k 6) If A, B, C can do a piece of work in X, Y, and Z days
respectively, if they work together they can do the same work in
Q. A and B together can do a piece of work in 3 days. If A does
thrice as much work as B in a given time, find how long A alone days.
would take to do the work? 7) A and B working together can do a work in X days. If A alone
can do the same work in Y days, B alone can do the same work
Sol. x = 3, k = 3
Time taken by A, working Alone to complete the work = ((3 + in days.
1)/3) × 3 = 4 days 8) A and B working together can finish a piece of work in X
days, B and C working together can finish the same work in Y
If A working Alone takes a days more than A & B, & B working days and C and A in Z days. Then;
Alone takes b days more than A & B. Then , o A, B and C working together will finish the work
Number of days, taken by A & B working together to finish a job
is = √ab in days.

Explanation : o A alone will finish the work in = days.

⇒ Let A + B takes x days o B alone will finish the work in = days.


A → x + a days
B → x + b days o C alone will finish the work in = days.
1/(x + a) +1/ (x + b) = 1/x 9) A can do a work in X days. If B is P times efficient than A, A
(2x + a + b) / (x² + xa + xb + ab) = 1/x
2x² + xa + xb = x² + xa + xb + ab and B working together can do the work in days.
x² = ab 10) A and B working together can complete a work in X days. If
x = √ab days B is P times efficient than A;
o A alone can complete the work in (P + 1) X days.
Q. A alone would take 8 hrs more to complete the job than if
both A and B worked together. If B worked alone, he took 41/2 o B alone can complete the work in X days.

For details on our programs: - Contact: +91 9930202329, +91, 8291924300 or visit www.campuscredentials.com (44)
11) P working alone takes X days more to do a work than P and 3) A alone can do a piece of work in 6 days and B alone in 8
Q working together takes to do the same work. And, Q working days. A and B undertook to do it for Rs. 3200. With the help of
alone takes Y days more to do the same work than P and Q C, they completed the work in 3 days. How much is to be paid to
working together takes to finish the same work. Then P and Q C?
working together can finish the same work in √XY days. A) Rs. 375 B) Rs. 400 C) Rs. 600
12) A is P times more efficient than B and able to finish the D) Rs. 800
work in X days less than B, then
o A and B working together can finish the work in 4) A and B together can complete a work in 12 days. A alone
can complete it in 20 days. If B does the work only for half a
days. day daily, then in how many days A and B together will
complete the work?
o A alone can finish the work in days. A) 10 days B) 11 days C) 15 days
D) 20 days
o B alone can finish the work in days.
5) A and B can do a job together in 7 days. A is 1 3/4 times as
13) If A completes part of a work in X days, part of the efficient as B. The same job can be done by A alone in :
A) 9 1/3 days B) 11 days C) 12 1/4 days
D) 16 1/3 days E) None of these
same work he can complete in days.
14) There are two groups of workers with same efficiency. In
6) A and B together can complete a piece of work in 4 days. If A
one group M1 workers can do W1 work in D1 days or time. In
alone can complete the same work in 12 days. in how many
the second group M2 workers can do W2 work in D2 days or
days can b alone complete that work?
time.Then;
A) 1/3 days B) 1/6 days C) 1/9 days
M1 D1 W2 = M2 D2 W1
D) 1/12 days
15) There are two groups of workers with the same efficiency.
In one group M1 workers can do W1 work in D1 time or days
7) 10 women can complete a work in 7 days and 10 children
working T1 hours a day. In the second group, M2 workers can
take 14 days to complete the work. How many days will 5
do W2 work in D2 time or days working T2 hours in a day.
women and 10 children take to complete the work?
Then;
A) 3 B) 5 C) 7
M1 D1 T1 W2 = M2 D2 T2 W1
D) Cannot be determined E) None of these
o If the efficiency of the workers is different in these
groups i.e. E1 and E2,; M1 D1 T1 E1W2 = M2 D2
8) A can finish a work in 18 days and B can do the same work in
T2E2W1
half the time taken by A. Then, working together, what part of
o Also remember that efficient person takes less time to
the same work they can finish in a day?
complete a given work. So, we can say that efficiency 𝟏 𝟐 𝟏 𝟐
(E) is indirectly proportional to the number of days (D) A) B) C) D)
𝟔 𝟓 𝟗 𝟕
taken to complete the given work.
Therefore, ED = constant 9) A, B and C can complete a piece of work in 24, 6 and 12 days
E1D1 = E2D2 respectively. Working together, they will complete the same
More men will take fewer days and fewer men will take more work in:
days to complete a given work. A) 1/24 days B) 3 3/7 days C) 3 days D) 4 days
16) Wages are directly proportional to the work done by the E) None of these
individual and inversely proportional to the time taken by the
individual. 10) Worker A takes 8 hours to do a job. Worker B takes 10
Total wages = One person's one day's wage * number of hours to do the same job. How long should it take both A and B,
persons* number of days working together but independently, to do the same job?
A's share : B's share : C's share = B's time*C time : A's A) 2x4/9 B) 4x4/9 C) 5x4/9 D) 4x2/9
time*C's time : A's time * B's time.
11) A, B and C can do a piece of work in 20, 30 and 60 days
Practice set 1: respectively. In how many days can A do the work if he is
1) A can finish a work in 18 days and B can do the same in half assisted by B and C on every third day?
the time taken by A. Then, working together , what part of the A) 12 days B) 15 days C) 16 days D)18 days
same work they can finish in a day?
A) 1/6 B) 1/9 C) 2/5 D) 2/7 12) A, B and C can do a piece of work in 11 days, 20 days and
55 days respectively, working alone. How soon can the work be
2) Two men, A and B, run a 4 km race on a circular course of done if A is assisted by B and C on alternate days?
1/4 km. If their speeds are in the ratio of 5:4, how often doe's A) 7 days B) 8 days C) 9 days
the winner pass the other? D) 10 days
A) Once B) Twice C) Thrice
D) Four times 13) A works twice as fast as B. If B can complete a work in 12
days independently, the number of days in which A and B can
together finish the work in :

For details on our programs: - Contact: +91 9930202329, +91, 8291924300 or visit www.campuscredentials.com (45)
A) 4 days B) 6 days C) 8 days 24) X can do a piece of work in 40 days. He works at it for 8
D) 18 days E) None of these days and then Y finished it in 16 days. How long will they
together take to complete the work?
14) A,B, and C can complete a piece of work in 24,6 and 12 A)13 1/3 days B) 15 days C) 20 days
days respectively. Working together , they will complete the D) 26 days E) None of these
same work in
A) 1/24 day B) 7/24 day C) 3x3/7 days 25) 4 men and 6 women can complete a work in 8 days, while 3
D) 4 days men and 7 women can complete it in 10 days. In how many days
will 10 women complete it?
15) A can lay railway track between two given stations in 16 A) 35 B) 40 C) 45 D) 50
days and B can do the same job in 12 days. With help of C, they
did the job in 4 days only. Then, C alone can do the job in: 26) A and B can do a piece of work in 30 days, while B and C
𝟏 𝟐 𝟑 can do the same work in 24 days and C and A in 20 days. They
A) 9 B) 9 C) 9 D) 10
𝟓 𝟓 𝟓
all work together for 10 days when B and C leave. How many
days more will A take to finish the work?
16) A can finish a work in 18 days and B can do the same work
A) 18 days B) 24 days C) 30 days
in 15 days. B worked for 10 days and left the job. In how many
D) 36 days
days. A lone can finish the remaining work?
A) 5 B) 7 C) 6 D) 8
27) A can do a work in 15 days and B in 20 days. If they work
on it together for 4 days, then the fraction of the work that is left
17) A can finish a work in 24 days, B in 9 days and C in 12
is
days. B and C start the work but are forced to leave after 3
A) 1/4 B) 1/10 C) 7/15 D) 8/15
days. The remaining work was done by A in:
A) 5 days B) 6 days C) 8 days
28) A and B together can complete a piece of work in 35 days
D) 10 days E) None of these
while A alone can complete the same work in 60 days. B alone
will be able to complete the same working in:
18) Ronald and Elan are working on an assignment. Ronald
A) 74 Days B) 80 Days C) 84 Days
takes 6 hours to type 32 pages on a computer, while Elan takes
D) 90 Days
5 hours to type 40 pages. How much time will they take,
working together on two different computers to type an
29) P can complete a work in 12 days working 8 hours a day. Q
assignment of 110 pages?
can complete the same work in 8 days working 10 hours a day.
A) 7 hours 30 minutes B) 8 hours
If both P and Q work together, working 8 hours a day, in how
C) 8 hours 15 minutes D) 8 hours 25 minutes
many days can they complete the work?
𝟓 𝟔 𝟓 𝟔
19) A and B can together finish a work 30 days. They worked A) 5 B) 5 C) 6 D) 6
𝟏𝟏 𝟏𝟏 𝟏𝟏 𝟏𝟏
together for 20 days and then B left. After another 20 days, A
finished the remaining work. In how many days A alone can 30) If A can do 1/4 of a work in 3 days and B can do 1/6 of the
finish the work? same work in 4 days, how much will A get if both work together
A) 40 B) 50 C) 54 D) 60 and are paid Rs. 180 in all?
A) Rs. 36 B) Rs. 60 C) Rs. 108
20) A and B can do a work in 12 days, B and C in 15 days, C D) Rs. 120
and A in 20 days. If A, B and C work together, they will
complete the work in: Practice set 2:
𝟓 𝟐
A) 5 days B) 7 C) 10 days D) 15
𝟔 𝟑
1) Worker A completes a task in 8 days, and worker B
completes the same task in 10 days. If both A and B work
21) Twenty women can do a work in sixteen days. Sixteen men
together, in how many days they will complete the task?
can complete the same work in fifteen days. What is the ratio
A) 40/10 days. B) 40/9 days. C) 40/8 days.
between the capacity of a man and a woman?
D) 40/7 days.
A) 3 : 4 B) 4 : 3 C) 5 : 3
D) Data inadequate E) None of these
2) Vikas and Mohan working together can complete a work in 6
days. If Vikas alone completes the same work in 10 days, in how
22) If 6 men and 8 boys can do a piece of work in 10 days while
many days Mohan alone can complete the same work?
26 men and 48 boys can do the same in 2 days, the time taken
A) 13 days B) 14 days C) 15 days
by 15 men and 20 boys in doing the same type of work will be:
D) 16 days
A) 4 days B) 5 days C) 6 days D) 7 days
3) A can do a work in 10 days and B can do the same work in
23) A takes twice as much time as B or thrice as much time to
15 days. If they start working together but stop working after
finish a piece of work. Working together, they can finish the
four days, find the fraction of the work that is left.
work in 2 days. B can do the work alone in:
A) 1/a B) 2/3 C) 4/7 D) ½
A) 4 days B) 6 days C) 8 days
D) 12 days

For details on our programs: - Contact: +91 9930202329, +91, 8291924300 or visit www.campuscredentials.com (46)
4) Peter is twice as good as workman as Tom. When they work 15) A group of workers undertakes a task. They can complete
together they can finish a task in 16 days. If Tom works alone, the task in 30 days. If 5 of them did not turn for the work and
in many days he will complete the task? the remaining workers complete the task in 40 days, find the
A) 46 days B) 48 days C) 50 days original number of workers.
D) 52 days A) 25 days B) 23 days C) 21 days
D) 20 days
5) A can do a job in 12 days and B can do the same job in 10
days. With the help of C they can do the same job in 4 days. In 16) 40 men can paint a building in 15 days working 4 hours a
how many days C alone can do this job? day. In how many days 30 men working 5 hours a day will
A) 15 days B) 14 days C) 13 days complete the work?
D) 12 days A) 14 days B) 15 days C) 16 days
D) 17 days
6) A, B, C can do a job in 10, 20 and 40 days respectively. In
how many days A can complete the job if he is assisted by B and 17) A certain number of men can do a piece of work in 50 days.
C on every third day? If there were 6 men more the same work could be done in 10
A) 8 days B) 7 days C) 9 days days less. Find the original number of men.
D) 6 days A) 21 men B) 22 men C) 23 men
7) If 5 men can colour 50-meter long cloth in 5 days, in many D) 24 men
days 4 men can color a 40-meter long cloth?
A) 5 days B) 6 days C) 4 days D) 3 days 18) A is twice as good as workman as B so he can do a work in
40 days less than B. If they work together, in how many days
8) If 4 men can finish 4 times of a work in 4 days, in how many they can do the work?
days 6 men can finish the 6 times of same work ? A) 80/4 days B) 80/5 days C) 80/3 days
A) 3 days B) 4 days C) 5 days D) 6 days D) 80/6 days

9) A can do a piece of work in 10 days. B is 50% more efficient 19) A is twice as good as workman as B and working together
than A. In how many days B alone can do the same job? they can do a piece of work in 20 days. If A alone works, in how
A) 6.2 days B) 6.6 days C) 7 days many days he will do the work?
D) 7.2 days A) 30 days B) 33 days C) 37 days
D) 40 days
10) A can do a job in 30 days. B alone can do the same job in
20 days. If A starts the work and joined by B after 10 days, in 20) 3 workers can do a piece of work in 20 days. If one of them
how many days the job will be done? works only for half time in a day, in how many days the work
A) 15 days B) 16 days C) 17 days will be done?
D) 18 days A) 24 days B) 20 days C) 22 days
D) 23 days
11) A can do a piece of work in 6 days working 8 hours a day. B
can do the same work in 4 days working 6 hours a day. If they 21) A can finish the job at the same time in which B and C
work together 8 hours a day, in how many days they will do this together do it. If A and B together can finish the work in 10 days
work? and C alone can do the work in 50 days, how many days B will
A) 3 days B) 3.5 days C) 2 days take to complete the same job?
D) 2.5 days A) 20 days B) 22 days C) 22(1/2) days
D) 25 days
12) A can finish a work in 6 days and B can finish the same
work in 8 days. A and B charge Rs. 2800 for the work. If with 22) A and B working together can finish a work in 12 days, B
the help of C they complete the work in 3 days, how much they and C working together can finish the work in 16 days. If A
will pay to C? works for 5 days, B works for 7 days, and C completes the
A) Rs. 350 B) Rs. 345 C) Rs. 340 remaining work in 13 days, C alone can complete the work in
D) Rs. 320 how many days?
A) 22 days B) 24 days C) 26 days
13) 5 men undertook a piece of work and finished half the work D) 28 days
in 18 days if two men drop out, in how many days the remaining
work will be completed? 23) A can finish a work in 10 days and B can finish the same
A) 30 days B) 32 days C) 34 days work in 15 days. If they work alternatively, find the time taken
D) 36 days to finish the job.
A) 14 days B) 15 days C) 12 days
14) If 5 workers can paint a house in 9 days, in how many days D) 14.5 days
3 workers can complete the same task?
A) 13 days B) 14 days C) 15 days 24) A, B, and C individually can finish a job in 10, 15, and 30
D) 16 days days respectively. If A starts the work and continues until the
end, B and C work alternatively, in how many days work will be
done?

For details on our programs: - Contact: +91 9930202329, +91, 8291924300 or visit www.campuscredentials.com (47)
A) 6 days B) 4 days C) 4[1/3] days Chapter 09: Time Speed & Distance
D) 6[3/5] days
Distance = Time × Speed
25) A can finish a work in 10 days, B can finish the same work • When Distance is constant
in 15 days, and C can finish it in 30 days. All three start Time ∝ 1/speed
working together, but after some days A leaves the job, then
• When Time is constant
after one day B also left the job. C completes the remaining job
Distance ∝ speed
in 3 days. Find the number of working days of B.
• When speed in constant
A) 4 days B) 5 days C) 6 days D) 7 days
Distance ∝ Time
• Average speed = (Total Distance )/(Total Time Taken)
When Distance is equal
Average speed = 2xy/(x + y)
x, y → speeds

Q1. A car takes half of the time taken by truck to go from Pune
to Mumbai. A truck takes 20 hours to go for the same journey.
What is the speed of truck, if the speed of car be 120 km/hr?

Q2. Maitry when increases her speed from 24 km/hr to 30 km/hr


she takes one hour less than the usual time to cover a certain
distance. What is the distance usually covered by Maitry?

Q3. Kirti goes to school at 20 km/hr and reaches the school 4


minutes late. Next time, she goes at 25 km/hr and reaches the
school 2 minutes earlier than the scheduled time. What is the
distance of her school?

For details on our programs: - Contact: +91 9930202329, +91, 8291924300 or visit www.campuscredentials.com (48)
Q4. Ramesh travels half of his journey by train at the speed of
120 km/hr and rest half by car at 80 km/hr. What is the average Q2. The distance between two cities A and B is 330 km. A train
speed? starts from A at 8 a.m. and travels towards B at 60 km/hr.
Another train starts from B at 9 a.m. and travels towards A at
75 km/hr. At what time do they meet?

Sol. Distance covered by train started from A in 1 hour = 60


km/hr
Remaining distance = 330 – 60 = 270
Time = 270/135 = 2 hours
They will meet at 9 + 2 = 11:00 am
Q5. Walking at 4/5 of his normal speed, Ravi is 15 minutes late
Q3. A thief Bhagu Ram is spotted by the policeman Pakad Singh
in reaching his club. What is the usual time taken by him to
from a distance of 200m. Once they see each other they start
cover the distance?
Running. What is the distance Bhagu Ram, who is running at 5
km/hr would have covered before being caught by Pakad Singh
running at 7 km/hr?

Relative Speed→

a) When two bodies move in the same direction, Let speed of Q4. A train starts from A to B at 9:00 am and takes 6 hours to
two bodies be S₁ & S₂. travel to B. Another train starts from B to A at 10:00 am and
Relative speed = S₁ – S₂ takes 8 hours to travel to A. At what time both train will meet?
(b) When two bodies are moving in the opposite direction. Let
the speed of two bodies be S₁ & S₂.
Relative speed = S₁ + S₂.
(c) When two bodies moving towards each other than time taken
by them to meet.
D → Distance between two bodies.
S₁, S₂ → Speed of two bodies.
T, time taken to meet other = D/(S₁ + S₂ )
(d) When two bodies are moving in opposite direction, time
taken to meet.
D→ Distance between the two bodies.
S₁, S₂ → Speed of two bodies.
T, time taken = D/(S₁ – S₂ )
(e) If two persons A & B, start at the same time from P and Q
towards each other and after crossing they take T₁ & T₂ hrs in
reaching Q & P
S₁ /S₂ =√(T₁/T₂ )
Q1. The distance between 2 places R and S is 42 km. Anita
starts from R with a uniform speed of 4 km/hr towards S and at
the same time Bipasha starts from S towards R also with some
uniform speed. They meet each other after 6 hours. The speed of
Bipasha is

For details on our programs: - Contact: +91 9930202329, +91, 8291924300 or visit www.campuscredentials.com (49)
Q5. Abhinav leaves Mumbai at 6 am and reaches Bangalore at Q1. Two trains of length 200 m and 175 m run or parallel
10 am Praveen leaves Bangalore at 8 am and reaches Mumbai tracks. When running is the same direction the faster train
at 11:30 am. At what time do they cross each other? crosses the slower train in 37½ sec. While running is opposite
directions they passes each other is 7½ s. Find the speed of
each train.

Q2. A train crosses a man coming from the opposite direction in


7.5 seconds. If the speed of man be 10 m/s and speed of train is
20 m/s, find the length of the train.

Trains
Q3. Two trains coming from the opposite sides crosses each
other in 10 seconds if the lengths of first train and second train
(a) If a train of length l meters passes a platform or bridge of
be 125 m and 175 m respectively, also the speed of first train be
length m metres, then distance travelled is
36 km/hr, find the speed of second train.
Distance = l + m

(b) If a train of length l meters passes a pole, man, tree etc, then
Distance travelled is
Distance = l meters

(c) If two trains of lengths L₁ & L₂ are travelling in the same


direction with speeds S₁ & S₂ then. Time taken by faster train to
cross slower train is given by
T=(L₁+L₂)/(S₁-S₂)

(d) If two trains of length L₁ & L₂ are travelling is opposite


direction with speeds S₁ & S₂, then time taken by trains to cross
each other is
T=(L₁+L₂)/(S₁-S₂)

(e) Two trains of length L₁ & L₂ run on parallel tracks. When


Q4. A train overtakes two girls who are walking in the opposite
running is same direction, the faster train passes slower train in
direction in which the train is going at the rate of 3 km/hr and 6
T₁ secs, but when they are running in opposite direction with
km/hr and passes them completely in 36 seconds and 30 seconds
same speeds, they passes each other in T₂ sec. Then,
respectively. The length of the train (in metres) is:

For details on our programs: - Contact: +91 9930202329, +91, 8291924300 or visit www.campuscredentials.com (50)
Q3. A man rows 12 km in 5 hours against the stream and the
speed of current being 4 kmph. What time will be taken by him
Boat & Stream
to row 15 km with the stream?
(a) Downstream→ When boat & stream moves in the same
direction.
Downstream Speed = u + v
u → speed of boat
v → speed of stream
(b) Upstream→ When boat & stream moves in the opposite
direction.
Upstream speed = u – v
u → speed of boat
v → speed of stream
(c) If D → is downstream speed , U→ is upstream speed.Then,
Speed of boat = (D + U)/2
Speed of stream = (D - U)/2
(d) When the distance traveled by boat is downstream is same
Practice set 1:
as the distance covered by boat is upstream, then,
1) A train travels at an average of 50 miles per hour for 2x1/2
hours and then travels at a speed of 70 miles per hour for 1x1/2
Q1. A man can row 9 km/hr in still water. If takes him twice as hours. How far did the train travels in the entire 4 hours?
long as to row up as to row down. Find the rate of stream of A) 120 miles B) 150 miles
river. C) 200 miles D) 230 miles

2) Two trains starting at the same time from two stations 200
km apart and going in opposite directions cross each other at a
distance of 110 km from one of the stations. What is the ratio of
their speeds?
A) 9 : 20 B) 11 : 9
C) 11 : 20 D) 9 : 20 E) None
of these

𝟏
3) One of the two buses completes a journey of 300 km in 7
𝟐
Hours and the other a journey of 450 km in 9 hours. The ratio
of their average speeds is:
A) 2:3 B) 3:4 C) 4:5 D) 8:9
Q2. The speed of a boat downstream is 15 km/hr and the speed
of current is 3 km/hr. Find the total time taken by the boat to 4) A person crosses a 600 m long street in 5 minutes. What is
cover 15 km upstream and 15 km downstream. his speed in km per hour?
A) 3.6 B) 7.2 C) 8.4 D) 10

5) Excluding stoppages, the speed of a bus is 54 kmph and


including stoppages, it is 45 kmph. For how many minutes does
the bus stop per hour?
A) 9 B) 10 C) 12 D) 20

For details on our programs: - Contact: +91 9930202329, +91, 8291924300 or visit www.campuscredentials.com (51)
6) Walking at the rate of 4kmph a man cover certain distance in becomes 35 kmph, then it reaches its destination 15 minutes
2hr 45 min. Running at a speed of 16.5 kmph the man will cover late. Find the length of journey.
the same distance in. A) 30 km B) 40 km C) 70 km D) 80 km
A) 12 min B) 25 min C) 40 min D) 60 min
17) A man completes a journey in 10 hours. He travels first half
7) 2 trains starting at the same time from 2 stations 200km of the journey at the rate of 21 km/hr and second half at the rate
apart and going in opposite direction cross each other at a of 24 km/hr. Find the total journey in km.
distance of 110km from one of the stations. What is the ratio of A) 220 km B) 224 km C) 230 km D) 234 km
their speeds?
A) 11 : 9 B) 7 : 3 C) 18 : 4 18) A passenger train takes two hours less for a journey of
D) None of these 300km if its speed is increased by 5km/hr from its normal speed.
The normal speed is:
8) An express train travelled at an average speed of 100 km/hr, A) 35 km/hr B) 50 km/hr C) 25 km/hr
stopping for 3 minutes after every 75 km. How long did it take D) 30 km/hr
to reach its destination 600 km from the starting point?
A) 6 hrs 21 min B) 6 hrs 24 min 19) Sound is said to travel in air at about 1100 feet per second.
C) 6 hrs 27 min D) 6 hrs 30 min A man hears the axe striking the tree, 11/5 seconds after he sees
it strike the tree. How far is the man from the wood chopper?
9) The distance between two cities A and B is 330 km. A train A) 2197 ft B) 2420 ft C) 2500 ft D) 2629 ft
starts from A at 8 a.m. and travels towards B at 60 km/hr.
Another train starts from B at 9 a.m. and travels towards A at 20) A man in a train notices that he can count 21 telephone
75 km/hr. At what time do they meet? posts in one minute. If they are known to be 50 meters apart,
A) 10 am B) 10:30 am C) 11 am then at what speed is the train travelling?
D) 11:30 am E) None of these A) 55 kmph B) 57 kmph C) 60 kmph
D) 63 kmph E) None of these
10) If a train runs at 40 kmph, it reaches its destination late by
11 minutes but if it runs at 50 kmph, it is late by 5 minutes only. 21) A boy rides his bicycle 10 km at an average speed of 12
The correct time for the train to complete its journey is: km/hr and again travels 12 km at an average speed of 10 km/hr.
A) 13 min B) 15 min C) 19 min D) 21 min His average speed for the entire trip is approximately
A) 10.4 km/hr B) 10.8 km/hr C) 11 km/hr
11) If a person walks at 14 km/hr instead of 10 km/hr, he would D) 12.2 km/hr
have walked 20 km more. The actual distance travelled by him
is: 22) A train can travel 50% faster than a car. Both start from
A) 50 km B) 56 km C) 70 km D) 80 km point A at the same time and reach point B 75 kms away from A
at the same time. On the way, however, the train lost about 12.5
12) Excluding stoppages, the speed of a bus is 54 kmph and minutes while stopping at the stations. The speed of the car is:
including stoppages, it is 45 kmph. For how many minutes does A) 100 kmph B) 110 kmph C) 120 kmph
the bus stop per hour? D) 130 kmph
A) 4 B) 6 C) 8 D) None of these
𝟐
23) Vikas can cover a distance in 1 hr 24 min by covering of
𝟑𝒓𝒅
13) Two stations A and B are 110 km apart on a straight line.
the distance at 4 kmph and the rest at 5kmph. The total distance
One train starts from A at 7 am and travel towards B at 20
is?
km/hr speed. Another train starts from B at 8 am and travel
A) 4 km B) 6 km C) 8 km D) 10 km
towards A at 25 km/hr speed. At what time will they meet?
A) 09 am B) 10 am C) 11 am
24) A person crosses a 600 m long street in 5 minutes. What is
D) None of these
his speed in km per hour?
A) 3.6 B) 7.2 C) 8.4 D) 10
14) The speed of a car increases by 2 kms after every one hour.
If the distance travelling in the first one hour was 35 kms. what
25) A farmer travelled a distance of 61 km in 9 hours. He
was the total distance travelled in 12 hours?
travelled partly of foot @ 4km/hr and partly on bicycle @ 9
A) 456 kms B) 482 kms C) 552 kms D) 556 kms
km/hr. The distance travelled on foot is:
A) 14 km B) 15 km C) 16 km D) 17 km
15) A and B walk around a circular track. They start at 8 a.m.
from the same point in the opposite directions. A and B walk at
26) The ratio between the speeds of two trains is 7 : 8. If the
a speed of 2 rounds per hour and 3 rounds per hour
second train runs 400 km in 4 hours, then the speed of the first
respectively. How many times shall they cross each other before
train is:
9.30 a.m.?
A) 70 km/hr B) 75 km/hr C) 84 km/hr
A) 5 B) 6 C) 7 D) 8
D) 87.5 km/hr
E) None of these

16) A train when moves at an average speed of 40 kmph,


reaches its destination on time. When its average speed

For details on our programs: - Contact: +91 9930202329, +91, 8291924300 or visit www.campuscredentials.com (52)
27) A train covers a distance in 50 min, if it runs at a speed of C) 200, 80 Sec D) None Of These
48kmph on an average. The speed at which the train must run to
reduce the time of journey to 40min will be. 37) In a circular race of 1200 m length, A and B start with
A) 45 kmph B) 60 kmph C) 75 kmph speeds of 18kmph and 27 kmph starting at the same time from
D) None of these the same point. When will they meet for the first time at the
starting point when running in the same direction and opposite
28) A man can row 4.5 km/hr in still water and he finds that it direction?
takes him twice as long to row up as to row down the river. A) 240, 160 Secs B) 480, 480 Secs
Find the rate of the stream. C) 240, 240 Secs D) 160, 480 Secs
A) 2 km/hr B) 2.5 km/hr C) 1.5 km/hr
D) 1.75 km/hr 38) A, B and C run around a circular track of length 1200 m
with respective speeds 9, 18, 27 kmph. If they started at the
29) Two guns were fired from the same place at an interval of same time from the same point and run in the same direction
10 minutes and 30 seconds, but a person in the train when will they meet for the first time?
approaching the place hears the second shot 10 minutes after A) 360 Sec B) 480 Sec C) 240 Sec
the first. The speed of the train (in km/hr), supposing that speed D) None Of These
travels at 330 metres per second, is:
A) 19.8 B) 58.6 C) 59.4 D) 111.80 39) P, Q, R run around a circular track 1200 m long with speed
of 9, 18, 27 kmph. If they start at the same point and at the
30) An airplane covers a certain distance at a speed of 240 same time in the same direction, when will they meet again at
𝟐 the starting point?
kmph in 5 hours. To cover the same distance in 1 hours, it must
𝟑
A) 360 Sec B) 480 Sec C) 240 Sec
travel at a speed of:
D) None Of These
A) 300 kmph B) 360 kmph C) 600 kmph
D) 720 kmph
40) A can run one full round of a circular track in 6 min and B
in 15 min. If both A and B start simultaneously from the same
31) In a race of 1000 meters, A can beat B by 100 meters, in a
starting point then How many times would they met in the time
race of 800 meters, B can beat C by 100 meters. By how many
B has completed 10 rounds when running in same direction,
meters will A beat C in a race of 600 meters?
and In opposite direction?
A) 125.5 Meters B) 126.5 Meters
A) 15, 10 B) 25, 30 C) 25, 35
C) 127.5 Meters D) 128.5 Meters
D) None Of These
E) None Of These

𝟐
32) A runs 1 times as fast as B. If A gives B a start of 80 m,
𝟑
how far must the winning post be so that A and B might reach it
at the same time?
A) 200 m B) 300m C) 270m
D) 160m E) None Of These

33) A can give B 100 meters start and C 200 meters start in a
kilometre race. How much start can B give C in a kilometre
race?
A) 110.12 Meters B) 111.12 Meters
C) 112.12 Meters D) 113.12 Meters
E) None Of These

34) In a 200 metres race A beats B by 35 m or 7 seconds. A's


time over the course is:
A) 40 Sec B) 47sec C) 33sec
D) 35 Sec E) None Of These

35) In a 500 m race, the ratio of the speeds of two contestants A


and B is 3 : 4. A has a start of 140 m. Then, A wins by:
A) 60 M B) 40 M C) 20 M
D) 10 M E) None Of These

36) In a circular race of 1200m, A and B start from the same


point and at the same time with speeds of 27kmph and 45
kmph. Find when will they meet again for the first time on the
trace when they are running in the same direction and Opposite
direction?
A) 240, 60 Secs B) 200, 60 Secs

For details on our programs: - Contact: +91 9930202329, +91, 8291924300 or visit www.campuscredentials.com (53)
Chapter 10 Pipe & Cistern Solution: A fills 4 buckets in 24 minutes. Thus, A fills 1 bucket
in 24/4 = 6 minutes
A pipe is connected to a tank or cistern. It is used to fill or
empty the tank; accordingly, it is called an inlet or an outlet. Similarly, B fills 8 buckets in 1 hour. Thus B fills 1 bucket in
Inlet: A pipe which is connected to fill a tank is known as an 60/8 minutes
inlet. Similarly, C fills one bucket in 20/2 = 10 minutes
Outlet: A pipe which is connected to empty a tank is known as In 2 hours,
an outlet.
Problems on pipes and cisterns are similar to problems on time Number of buckets filled by A will be = 120/6 = 20 buckets
and work. In pipes and cistern problems, the amount of work Number of buckets filled by B will be = 120/ (60/8) = (120 * 8) /
done is the part of the tank of filled or emptied. And, the time 60 = 16 buckets
taken to do a piece of work is the time take to fill or empty a
tank completely or to a desired level. Number of buckets filled by C will be = 120 / 10 = 12 buckets
Pipes and Cisterns Points to remember: Total number of buckets filled = (20 + 16 + 12) = 48 buckets
Total amount of water coming out of the tank = capacity of the
1) If an inlet connected to a tank fills it in X hours, part of the tank = 48 * 5 liters = 240 liters
tank filled in one hour is = 1/X Answer Choice: B

2) If an outlet connected to a tank empties it in Y hours, part of Example 2: There is a leak in the bottom of the tank. This leak
the tank emptied in one hour is = 1/Y can empty a full tank in 8 hours. When the tank is full, a tap is
opened into the tank which admit 6 liters per hour and the tank
3) An inlet can fill a tank in X hours and an outlet can empty the is now emptied in 12 hours. What is the capacity of the tank?
same tank in Y hours. If both the pipes are opened at the same A) 8.8 Liters B) 36 Liters C) 144 Liters
time and Y > X, the net part of the tank filled in one hour is D) Cannot Be Determined
given by;
= (1/X – 1/Y) Solution: Since the leak can empty the tank in 8 hours,
In one hour, part of the tank emptied by the leak = 1/8
Therefore, when both the pipes are open the time taken to fill Also, after opening the tap, in one hour, part of the tank emptied
the whole tank is given by; = 1/12
= (XY/Y-X) Hours
Let the tap can fill the tank in x hours. Therefore, In one hour,
If X is greater than Y, more water is flowing out of the tank than part of the tank filled by the tap = 1/x
flowing into the tank. And, the net part of the tank emptied in As per question, (1/x) – (1/8) = 1/12
one hour is given by; Or x = 24
= (1/Y – 1/X)
Since the tap admits 6 liters of water per hour, it will admit
Therefore, when both the pipes are open the time taken to empty (6*24) =144 liters of water in 24 hours, which should be the
the full tank is given by; capacity of the tank.
= (YX/X-Y) Hours Correct Answer: 24 minutes

4) An inlet can fill a tank in X hours and another inlet can fill Example 3: Three small pumps and one large pump are filling
the same tank in Y hours. If both the inlets are opened at the a tank. Each of the three small pump works at 2/3 of the rate of
same time, the net part of the tank filled in one hour is given by; the large pump. If all four pumps work at the same time, they
= (1/X + 1/Y) should fill the tank in what fraction of the time that it would
have taken the large pump alone?
Therefore, the time taken to fill the whole tank is given by; A) 4/7 B) 1/3 C) 2/3 D) ¾
= (XY/Y+X) Hours
In a similar way, If an outlet can empty a tank in X hours and Solution: As per the question,
another outlet can empty the same tank in Y hours, the part of Capacity of three small pumps = Capacity of two large pumps
the tank emptied in one hour when both the pipes start working Also, if we want to express the capacity of three small pumps +
together is given by; one large pump in terms of large pump, we should add one
= (1/X + 1/Y) large pump on both sides of the above equation
Adding one large pump on both sides of the above equation, we
Example 1: A water tank has three taps A, B and C. A fills four get
buckets in 24 minutes, B fills 8 buckets in 1 hour and C fills 2
buckets in 20 minutes. If all the taps are opened together, a full Three small pumps + one large pump = Three large pumps.
tank is emptied in 2 hours. If a bucket can hold 5 liters of water, Thus, if all the four pumps are open together, they would fill the
what is the capacity of the tank? tank in 1/3 rd of the time large pump would have taken alone.
A) 120 Liters B) 240 Liters C) 180 Liters Answer Choice B
D) 60 Liters
Example 4: A tank is fitted with 8 pipes, some of which that fill
the tank and others that empty the tank. Each of the pipes that

For details on our programs: - Contact: +91 9930202329, +91, 8291924300 or visit www.campuscredentials.com (54)
fills the tank fills it in 8 hours, while each of those that empty took 30 minutes more to fill the tank. If the tank is full, how long
the tank empties it in 6 hours. If all the pipes are kept open will it take for the leakage alone to empty the tank?
when the tank is full, it will take 6 hours to drain the tank. How 𝟏
A) 4 hrs B) 9 hrs C) 18 hrs D) 36 hrs
𝟐
many of these are fill pipes?
A) 2 Fill Pipes B) 4 Fill Pipes
2) Bucket P has thrice the capacity as bucket Q. It takes 60
C) 6 Fill Pipes D) 5 Fill Pipes
turns for bucket P to fill the empty drum. How many turns it will
take for both the buckets P and Q, having each turn together to
Solution: Let the number of fill pipes be ‘n’
fill the empty drum?
Therefore, there will be (8 – n) waste pipes.
A) 30 B) 40 C) 45 D) 90
Each of the fill pipes can fill the tank in 8 hours.
Therefore, each of the fill pipes will fill 1/8th of the tank in an
3) A booster pump can be used for filling as well as for
hour.
emptying a tank. The capacity of the tank is 2400 m3. The
emptying of the tank is 10 m3 per minute higher than its filling
Hence, n fill pipes will fill n/8th of the tank in an hour.
capacity and the pump needs 8 minutes lesser to empty the tank
Similarly, each of the waste pipes will drain the full tank in 6
than it needs to fill it. What is the filling capacity of the pump?
hours.
A) 50 m3/min B) 60 m3/min C) 72 m3/min
∴ each of the waste pipes will drain 1/6th of the tank in an hour.
D) None of these
(8 – n) waste pipes will drain (8-n)/6th of the tank in an hour.
Between the fill pipes and the waste pipes, they drain the tank in
4) Two pipes A and B can separately fill a cistern in 60 minutes
6 hours.
and 75 minutes respectively. Three is a third pipe in the bottom
of the cistern to empty it. If all the three pipes are
That is, when all 8 of them are opened, 1/6th of the tank gets
simultaneously opened, then the cistern is full in 50 minutes. In
drained in an hour.
how much time, the third pipe alone can empty the cistern?
(Amount of water filled by fill pipes in 1 hour – Amount of water
A) 90 min B) 100 min C) 110 min
drained by waste pipes 1 hour) = (1/6th ) of the tank
D) 120 min
Therefore,
(n/8) – ((8−n)/)6 = -1/6
5) Three taps A, B and C can fill a tank in 12, 15 and 20 hours
Note: The right hand side has a negative sign because the tank
respectively. If A is open all the time B and C are open for one
gets drained.
hour each alternately, the tank will be full in:
Cross multiplying and solving the equations, 14n – 64 = -8 𝟐 𝟏
or 14n = 56 or n = 4 A) 6 hrs B) 6 hrs C) 7 hrs D) 7
𝟑 𝟐
The correct answer is Choice (B). hrs

Example 5: Pipe A usually fills a tank in 2 hours. On account 6) Pipe A can fill a tank in 5 hours, pipe B in 10 hours and pipe
of a leak at the bottom of the tank, it takes pipe A 30 more C in 30 hours. If all the pipes are open in how many hours will
minutes to fill the tank. How long will the leak take to empty a the tank be filled ?
full tank if pipe A is shut? A) 2 B) 2.5 C) 3 D) 3.5
A) 2 Hours 30 Minutes B) 5 Hours
C) 4 Hours D) 10 Hours 7) Two pipes can fill a tank in 10 hours and 12 hours
respectively while a third pipe empties the full tank in 20 hours.
Pipe A fills the tank normally in 2 hours. If all the three pipes operate simultaneously, in how much time
Therefore, it will fill 1/2 of the tank in an hour. will the tank be filled?
Let the leak take x hours to empty a full tank when pipe A is A) 7 hrs 30 min B) 7 hrs 45 min
shut. C) 8 hrs 30 min D) 8 hrs 45 min
Therefore, the leak will empty 1/x of the tank in an hour.
The net amount of water that gets filled in the tank in an hour 8) A leak in the bottom of a tank can empty the full tank in 8
when pipe A is open and when there is a leak = (1/2 – 1/x) of hours. An inlet pipe fills water at the rate of 6 litres a minute.
the tank. —– (1) When the tank is full, the inlet is opened and due to the leak, the
tank is empty in 12 hours. How many litres does the cistern hold
Now, when there is a leak, the problem states that it takes two ?
and a half hours to fill the tank. i.e. 5/2hours. A) 7580 B) 7960 C) 8290 D) 8640
Therefore, in an hour, 2/5th of the tank gets filled. —– (2)
Equating (1) and (2), we get 1/2 – 1/x = 2/5 9) 12 buckets of water fill a tank when the capacity of each tank
=> 1/x = 1/2 – 2/5 = 1/10 is 13.5 litres. How many buckets will be needed to fill the same
=> x = 10 hours. tank, if the capacity of each bucket is 9 litres?
The correct answer is Choice (D). A) 8 B) 15 C) 16 D) 18

Practice Set 1: 10) A cistern can be filled by a tap in 4 hours while it can be
emptied by another tap in 9 hours. If both the taps are opened
1. Two taps A and B can fill a tank in 5 hours and 20 hours simultaneously, then after how much time will the cistern get
respectively. If both the taps are open then due to a leakage, it filled ?
A) 4.5 hrs B) 5 hrs C) 6.5 hrs D) 7.2 hrs

For details on our programs: - Contact: +91 9930202329, +91, 8291924300 or visit www.campuscredentials.com (55)
the proportion of the solution R in the liquid in the tank after 3
11) One pipe can fill a tank three times as fast as another pipe. minutes?
If together the two pipes can fill the tank in 86 minutes, then the 𝟓 𝟔 𝟕 𝟖
A) B) C) D)
𝟏𝟏 𝟏𝟏 𝟏𝟏 𝟏𝟏
slower pipe alone will be able to fill the tank in
A) 81 min B) 108 min C) 144 min
14) Pipes A and B can fill a tank in 5 and 6 hours respectively.
D) 192 min
Pipe C can empty it in 12 hours. If all the three pipes are
opened together, then the tank will be filled in:
12) Two pipes A and B can fill a tank in 6 hours and 4 hours 𝟏𝟑 𝟖 𝟗 𝟏
respectively. If they are opened on alternate hours and if pipe A A) 1 hours B) 2 hours C) 3 hours D) 4
𝟏𝟕 𝟏𝟏 𝟏𝟕 𝟐
is opened first, in how many hours, the tank shall be full ? hours
A) 4 hrs B) 5 hrs C) 7 hrs D) 9 hrs
15) A pump can fill a tank with water in 2 hours. Because of a
𝟏
13) Three pipes A, B and C can fill a tank from empty to full in leak, it took 2 hours to fill the tank. The leak can drain all the
𝟑
30 minutes, 20 minutes, and 10 minutes respectively. When the water of the tank in:
tank is empty, all the three pipes are opened. A, B and C 𝟏
A) 4 ℎ𝑜𝑢𝑟𝑠 B) 7 hours C) 8 hours
discharge chemical solutions P,Q and R respectively. What is 𝟑
D) 14 hours

For details on our programs: - Contact: +91 9930202329, +91, 8291924300 or visit www.campuscredentials.com (56)
Chapter 11 - Mensuration
Mensuration is the evaluation or calculation using suitable formulae without actual measurements of the geometrical concepts
such as:

length (of a straight line or any of the curve)


area (of the plane figure or bounded by curves)
Volume of a solid
Surface of a solid
Some basic Mensuration Formulas:

For details on our programs: - Contact: +91 9930202329, +91, 8291924300 or visit www.campuscredentials.com (57)
For details on our programs: - Contact: +91 9930202329, +91, 8291924300 or visit www.campuscredentials.com (58)
Practice Set 1: 11) The radius of a wheel of car is 70 cm. How many
1) The length and the breadth of a rectangular door are revolutions per minute the wheel will make in order to keep a
increased by 1 m each and due to this the area of the door speed of 66 km/hr?
increased by 21 sq. m. But if the length is increased by 1 m and A) 234 B) 272 C) 225 D) 300
breadth decreased by 1 m, area is decreased by 5 sq. m. Find E) 250
the perimeter of the door.
A) 25 M B) 20 M C) 40 M 12) The length and breadth of a rectangle are in the ratio 3 : 2.
D) 60 M E) 24 M If the length is increased by 5 m keeping the breadth same, the
new area of rectangle is 2600 m2. What is the breadth of the
2) The perimeter of a rectangular plot is 340 m. Find the cost of rectangle?
gardening 1 m broad boundary around it at the rate of Rs 10 A) 30 M B) 20 M C) 32 M
per sq. m. D) Cannot Be Determined E) None Of These
A) Rs 3450 B) Rs 3400 C) Rs 3400
D) Rs 3480 E) Rs 3440 13) What will be the percentage increase in the surface area of
the cube whose side is increased by 50%
3) The sides of a triangle are in the ratio 3 : 4 : 5 whose area is A) 75% B) 125% C) 150%
216 sq. cm. What will be the perimeter of this triangle? D) 100% E) 92%
A) 58 Cm B) 64 Cm C) 28 Cm
D) 36 Cm E) 72 Cm 14) The radius of base and height of a cylinder are in the ratio
2 : 3. Find the total surface area of the cylinder if its volume is
4) If the base of a triangle is increased by 50% and its height is 12936 cm3.
decreased by 50%, then what will be the effect on its area? A) 3080 Cm2 B) 2680 Cm2 C) 4940 Cm2
2
A) 50% Decrease B) 75% Increase D) 3280 Cm E) None Of These
C) No Effect D) 25% Decrease
E) 25% Increase 15) A rectangular box has dimensions 1.6 m × 1 m × 0.6 m.
How many cubical boxes each of side 20 cm can be fit inside
5) A rectangle whose sides are in the ratio 6 : 5 is formed by the rectangular box?
bending a circular wire of radius 42 cm. Find the largest side A) 220 B) 205 C) 120 D) 165
of the rectangle. E) 124
A) 60 Cm B) 72 Cm C) 66 Cm
D) 78 Cm E) 84 Cm 16) If the volume of wire remains the same but its radius
decreases to 1/3rd of previous, then the new length of the wire
6) A rectangular sheet of 0.5 cm thickness is made from an iron will be how many times of the previous length?
cube of side 10 cm by hammering it down. The sides of the A) 3 Times B) 4 Times C) 6 Times
sheet are in the ratio 1 : 5. Find the largest side of the sheet. D) 10 Times E) 9 Times
A) 100 Cm B) 72 Cm C) 20 Cm
D) 70 Cm E) 88 Cm 17) Four circular cardboard pieces, each of diameter 28 cm
are placed in such a way that each piece touches two other
7) The area of the inner part of a cylinder is 616 sq. cms and its pieces. Find the area of the space enclosed by the four pieces.
radius is half its height. Find the inner volume of the cylinder. A) 195 Cm2 B) 134 Cm2 C) 168 Cm2
3 3 3 2
A) 1577.5 Cm B) 1768.2 Cm C) 1538.5 Cm D) 162 Cm E) None Of These
D) 1435.8 Cm3 E) 1238.5 Cm3
18) Perimeter of a square and an equilateral triangle is equal.
8) A cylinder and a cone have equal base and equal height. The If the diagonal of the square is 10√2 cm, then find the area of
ratio of the radius of base to height is 5 : 12. Find the ratio of equilateral triangle?
the total surface area of the cylinder to that of the cone. A) (400√3)/9 B) (400√3/7) C) (200√3/7)
A) 7:15 B)16:9 C) 17:9 D) (200√3)/9 E) None Of These
D) 9:17 E) 15:7
19) Length of a rectangular field is increased by 10 meters and
9) A cone of radius 12 cm and height 5 cm is mounted on a breadth is decreased by 4 meters, area of the field remains
cylinder of radius 12 cm and height 19 cm. Find the total unchanged. If the length decreased by 5 meters and breadth is
surface area of the figure thus formed. increased by 7 meters, again the area remains unchanged. Find
A) 2498 Cm2 B) 2400 Cm2 C) 2476 Cm2 the length and breadth of the rectangular field.
D) 2376 Cm2 E) 2546 Cm2 A) 12, 11 B) 13, 12 C) 13,11
D) 14,12 E) 13,15
10) How many spherical balls whose radius is half that of
cylinder can be formed by melting a cylindrical iron rod whose 20) If the length of the rectangle is increased by 20%, by what
height is eight times its radius? percent should the width be reduced to maintain the same
A) 44 B) 48 C) 60 D) 56 area?
E) Cannot Be Determined A) 13.37 B) 16.67 C) 21.33
D) 33.33 E) None Of These

For details on our programs: - Contact: +91 9930202329, +91, 8291924300 or visit www.campuscredentials.com (59)
21) A cone of radius 12 cm and height 5 cm is mounted on a
cylinder of radius 12 cm and height 19 cm. Find the total
surface area of the figure thus formed?
A) 2276 B) 2376 C) 2476 D) 2576

22) The sum of the radius and height of a cylinder is 19m. The
total surface area of the cylinder is 1672 m², what is the volume
of the cylinder?(in m³)
A) 3080 B) 2940 C) 3220 D) 2660
E) 2800

23) If the area of a square is equal to the area of that rectangle


whose width is double of the one side of the square then the
ratio of the length to the breadth of the rectangle will be?
A) 1 : 2 B) 1 : 4 C) 1 : 6 D) 1 : 8
E) None Of The Above

24) What is the volume of a right cone whose cross section is


isosceles triangle with a base 10cm and slant height 13cm ?
A) 312.4 Sq Cm B) 314.2 Sq Cm
C) 224.2 Sq Cm D) 354.2 Sq Cm
E) None Of These

25) In a swimming pool measuring 80 cm x 30 cm, 120 men


take a dip. If the average displacement of water by a man is 5
cm cube, What will be the rise in water level ?
A) 1/5 Cm B) 1/3 Cm C) 1/2 Cm
D) 1/4 Cm E) None Of These

26) If the radius of the cone is doubled, keeping the height


constant, what is the ratio of the volume of the smaller cone to
larger cone?
A) 2:9 B) 5:7 C) 1:4 D) 1:7
E) None Of These

27) After measuring 100m of a rope, it was discovered that the


metre rod was 2cm longer. The true length of the rod is
A) 95m B) 98m C) 96m D) 93m

28) The number of rounds that a wheel of diameter14m will


make in going 2 km is
A) 62 Rounds B) 50 Rounds C) 45 Rounds
D) 56 Rounds E) None Of These

29) Meena wishes to start a 78sq m triangular flower garden.


Since she has only 20m barbed wire, she fences three sides of
the garden letting her house compound wall act as the fourth
side fencing. The dimension of the garden is
A) 12m*3m B) 26m*13m C) 16m*13m
D) 20m*11m E) None Of These

30) The perimeter of a rhombus is 300cm. If one of its


diagonals is 16cm then the area of the rhombus is
A) 1129 Sq Cm B) 1092 Sq Cm
C) 1022 Sq Cm D) 1192 Sq Cm

For details on our programs: - Contact: +91 9930202329, +91, 8291924300 or visit www.campuscredentials.com (60)
Chapter 12 - Permutation and Combination Whenever we have to choose certain things from a group and
no arrangement is done. In that case combination comes into
Factorial picture. So let us see concept of combination.

Before going deep into Permutation & Combination, let us Combination


figure out a term known as ‘factorial’. The product of the
numbers starting from 1 up to a number ‘n’ is known as In combination, we select the things at random & check out the
factorial of number ‘n’. different possible ways of selection. So this is a one step
It means n! = 1 x 2 x 3 x 4 x 5 x 6……..x(n - 2) x (n - 1) xn process. Combination is also known as collection. The formula
1! = 1 used for combination is nCr.
2! = 1x2 = 2
n
3! = 1x2x3 = 6 Cr = n! / [r! x (n-r)!]
4! = 1x2x3x4 = 24
n
Cr = [n x (n-1) x (n-2) x (n-3) x…...x(n-r+1) x (n-r) X…....x 1] /
5! = 1x2x3x4x5 = 120 [1x 2 x 3….. x r] X [(n- r) x……..3 x 2 x 1]
n
Cr = [n x (n-1) x (n-2) x (n-3)….. x (n-r+1)] / [1x 2 x 3….. x r]
Key points related to Factorial: For example: 12C2 = 12!/ [2! X (12-2)!] = 12!/ (2! X 10!) = [12
x 11] / [1 x 2] = 66
5
0! & 1! are equal to 1. C2 = [5 x 4] / [1 x 2] = 10
n
We can’t find factorial of a negative number. Cr = nC(n-r)

Application of factorial: For example: 5C3 = [5 x 4 x 3] / [1 x 2 x 3] = [5 x 4]/[1 x 2]


= 5C2 = 10
10
Suppose we have 5 persons and we want to arrange them on 5 C7 = 10C3 = [10 x 9 x 8]/[1 x 2 x 3] = 120.
vacant places. Then we will start with the first place. We can
choose 1 person out of 5 for the first place. We can do that in 5 Q.3) In a class there are 4 boys and 5 girls. In how many
ways. different ways a class monitor can be chosen?
Solution: As we can cl earl y see that we have to choose a
Now only 4 seats are vacant and 4 persons are left. We will student from total 9. So
choose 1 person out of 4 for the second place now. We can do we will use combination concept here which will give us the
that in 4 ways. answer as 9C1 = 9/1 = 9.

In the same manner, for the third place, 3 ways, for the four Q.4) In a class there are 4 boys and 5 girls. In how many
th place, 2 ways and for the last vacant place only 1 way of different ways a boy and a girl can be selected for group leader
selection is possible. As we know that we have to do all of these of two groups?
activities, so we will multiply all these ways to get the final Solutions: We have to choose a boy from 4 boys and a girl
answer for getting the different ways of arrangement. from 5 girls for two groups.
So total ways = 5x4x3x2x1 which is 5! = 120. So total ways of selection = 4C1x5C1 = 4x5 = 20
Or we can say that whenever we have to arrange ‘n’ things at
‘n’ places then total arrangements that can be made will Q.5) In how many different ways a cricket team can be
always be equal to n! selected from total 16 players?
Solution: We need to select 11 players from total 16 players.
Q.1) In how many ways can the letters of the word PATNA be So the answer will be 16C11 = 16!/5!*{16-5)! = 16!/5!*11! =
rearranged? {16*15*14*13*12)/{1*2*3*4*5) = 4368
Solution: PATNA has total 5 words. So we will arrange 5
letters at 5 places in 5! = 120 ways. Q.6) An urn contains 5 red and 3 blue balls. In how many
But in this question, A is coming twice. Whenever any letter is different ways, 2 red and 1 blue balls can be drawn?
more than once in the word, then we have to divide by the Solution: The urn contains 5 red and we want 2 red balls. So
number of repetition of the word. So we have to divide the total ways of selecting red balls =5C2 = 10
120 ways by 2!= 2. Similarly ways of selecting 1 blue ball from 3 blue balls = 3C1
So total different words that can be made will be 120/2 = 60. =3
Direct answer: 5!/2! = 60. So total ways to select 2 red and 1 blue ball will be = 10*3 =
30
Q.2) How many different words can be made using letters of
PATNA star ting with P? Q.7) In how many different ways a team of 11 can be selected
Solution: PATNA has total 5 words. According to the question, from 15 players if 2 particular players are never selected?
P is fixed at 1st place, so we will arrange remaining 4 letters at Solution: It is given that 2 particular players are never
4 places in 4! = 24 ways. But in this question, A is coming selected. So we will do selection from rest of the players which
twice, so we have to divide the total 24 ways by ! = 2. So the means we will select 11 players out of 13 players.
different words starting from P will be 24/2 = 12. So total ways of selection = {15-2)C11 = 13C11 = 13C2 =
Direct answer: 4!/2! = 4*3 = 12 {13*12)/{1*2) = 78

For details on our programs: - Contact: +91 9930202329, +91, 8291924300 or visit www.campuscredentials.com (61)
Q.8) In how many different ways a team of 11 can be selected Q.12) Twenty students are participating in a race. In how
from 15 players if 2 particular players are always selected? many ways can the first three prizes be won?
Solution: It is given that we have to select two particular Solution: First of all, we will select 3 candidates from total 20
players always which means that we have choice of selection candidates. The ways of selection are,
20
only for remaining 9 players and the possible options are only C3 = (20*19*18)/(1*2*3) = 1140
13.
So total possible ways of selection = (15-2)C(11-2) = 13C9 = After doing selection, we can arrange 3 candidates on 3
13C4 = (13*12*11*10) / (1*2*3*4) = 715. positions in 3! = 6 ways. So total possible number of ways in
Whenever we have to choose certain things from a group and which the first three prizes can be won = 1140*6 = 6840.
arrangement of those chosen things is to be done. In that case Direct answer = 20P3 = (20*19*18) = 6840 ways.
permutation comes into picture. So let us see concept of
permutation. Key points related to Permutation & Combination:

Permutation Whenever we want to arrange n things at n places, we have


total n! ways of arrangement.
In permutation, we select the things and then arrange them to Whenever we have to select r things out of n, we have total
check out different possible ways of arrangement. So basically nCr ways of
permutation is a two-step process. The formula used for selection.
permutation is nPr = n!/(n-r)!
Suppose we have 5 persons and we have to arrange them on Whenever we have to select r things from n and then arrange
3 vacant places. Then first of all, we will choose 3 persons from those r things at r places, we have total nPr ways.
5. We can do that in 5C3 different ways. After choosing 3 nCr = n! I [r! x (n-r)!]
persons, we will have to arrange them on the 3 vacant places, nCr = nC(n-r)
for that we will use factorial concept. The total ways to nPr = n!I(n-r)!
arrange 3 persons on 3 places are 3!
So total ways to arrange 3 persons from total 5 on 3 vacant Practice Set 1:
places will be:
5C3*3! = 5C2*3! = 5!/(2!*3!) * 3! = 5!/2! = 60 ways. 1) How many arrangement can be made from the word
COMMERCE, such that all the vowels do not come together?
Q.9) A wicket-keeper and a bowler are to be chosen out of a A) 6800 B) 5600 C) 1080
team having 11 players. In how many different ways we can D) 3600 E) None of these
do this?
Solution: First of all, we will select 2 players from total 11 2) 12 students participated in the competition and each get
players. The ways of selection are 11C2 = (11*10)/(1*2) = 55. different score. In how many ways can three different prizes
After doing selection, we can arrange 2 players on 2 different given?
positions in 2! = 2 ways. So total ways of selecting a wicket- A) 1320 B) 1240 C) 1650
keeper and a bowler = 11C2 *2! = 55*2 = 110 D) 1870 E) None of these
Direct answer: 11P2 = 110
3) How many four digit number can be formed with the digits
Q.10) In how many ways can the letters of the word 5,9,1 and 3 only ?
EQUATION be arranged so that all the vowels come together? A) 64 B) 216 C) 256
Solution: In word EQUATION, we have 5 vowels (E, U, A, I, O) D) 324 E) None of these
and 3 consonants (Q, T, N). According the question, all five
vowels to take one place and other 3 consonants will be 4) In a exam paper 1st section contains 10 questions each with
arranged on 3 places, So total 4 places. 5 choices and second section contains 5 questions each with 4
So ways to arrange these on 4 places will be 4! = 24 choices. In how many different ways can the paper be answered
One important thing is that we can arrange the vowels order if all the questions are attempted?
as well and we can do that in 5!= 120 ways. A) 105 × 45 B) 510 × 45 C) 510 × 54
5 4
So total ways = 24*120 = 2880 D) 10 × 5 E) None of these
Direct answer: 4!*5! = 24*120 = 2880
5) Three brothers have 5 shirts, 8 pants and 6 ties. In how many
Q.11) There are 7 candidates for 4 different posts. In how ways can they wear them?
many ways we can fill the posts? A) 2400200 B) 2419000 C) 2419200
Solution: First of all, we will select 4 candidates out of total 7 D) 2419100 E) None of these
candidates. The ways of selection are,
7C4 = 7C3 = (7*6*5)/(1*2*3) = 35 6) 5 men and 3 women are to be seated such that no 2 women
After doing selection, we can arrange 4 candidates for 4 sit together. Find the number of ways in which this can be
different posts in 4! ways arranged?
= 24 ways. A) 10000 B) 10800 C) 12525
So total possible number of ways to fill the posts = 35*24 = D) 12144 E) None of these
840
Direct answer: 7P4 = (7*6*5*4) = 840

For details on our programs: - Contact: +91 9930202329, +91, 8291924300 or visit www.campuscredentials.com (62)
7) A group consists of 3 couples in which each of the 3 men 18) In how many ways can 7 girls and 4 boys stand in a row so
have one wife each. In how many ways could they arranged in a that no 2 boys are together?
straight line so that the men and women occupy alternate A) 8467200 B) 9062700 C) 7407000
position? D) 8407200 E) None of these
A) 216 B) 125 C) 256
D) 72 E) None of these 19) In how many ways the letters of the word PERMUTATION
be arranged?
8) In how many ways can 5 different balls be distributed to 4 A) 10!/2! B) 10! C) 11! D) 11!/2!
different boxes, when each of the can hold any number of balls? E) None of these
A) 1024 B) 1200 C) 1234 D) 1600
E) None of these 20) In how many ways all the letters of the word ‘MINIMUM’
be arranged such that all vowels are together?
9) In how many ways the letters of the word ‘AUTHOR’ be A) 60 B) 30 C) 90 D) 70 E) 120
arranged taking all the letters?
A) 120 B) 720 C) 360 D) 60 21) How many 3 digit numbers are divisible by 4?
E) None of these A) 256 B) 225 C) 198 D) 252
E) 120
10) How many words of 4 letters with or without meaning be
made from the letters of the word ‘LEADING’, when repetition 22) How many 3 digits numbers have exactly one digit 2 in the
of letters is allowed? number?
A) 4808 B) 57600 C) 2401 D) 57624 A) 225 B) 240 C) 120 D) 160
E) None of these E) 185

11) In how many ways letters of word ‘INVISIBLE’ be 23) In how many ways 5 African and five Indian can be seated
arranged such that all vowels are together? along a circular table, so that they occupy alternate position.
A) 2560 B) 2880 C) 5040 D) 2520 A) 5! 5! B) 4! 5! C) 5! 4! D) 4! 4!
E) 720
24) There are 15 points in a plane out of which 6 are collinear.
12) In how many ways a group of 2 men and 4 women be made Find the number of lines that can be formed from 15 points.
out of a total of 4 men and 7 women? A) 105 B) 90 C) 91 D) 95
A) 720 B) 210 C) 420 D) 360 E) None Of These
E) 120
25) In party there is a total of 120 handshakes. If all the
13) There are 8 men and 7 women. In how many ways a group persons shakes hand with every other person. Then find the
of 5 people can be made such that at least 3 men are there in number of person present in the party.
the group? A) 15 B) 16 C) 17 D) 18
A) 1545 B) 1626 C) 1722 D) 1768 E) None Of These
E) 1844

14) There are 6 men and 7 women. In how many ways a


committee of 4 members can be made such that a particular
woman is always included.
A) 180 B) 120 C) 240 D) 220
E) 260

15) There are 5 men and 3 women. In how many ways a


committee of 3 members can be made such that 2 particular
men are always to be excluded.
A) 50 B) 20 C) 24 D) 48
E) None of these

16) A bag contains 4 red balls and 5 black balls. In how many
ways can I make a selection so as to take at least 1 red ball and
1 black ball?
A) 564 B) 345 C) 465
D) 240 E) None of these

17) In how many ways can 7 beads be strung into necklace?


A) 2520 B) 5040 C) 720 D) 360
E) None of these

For details on our programs: - Contact: +91 9930202329, +91, 8291924300 or visit www.campuscredentials.com (63)
Chapter 13 - Probability Compound probability is when the problem statement asks for
the likelihood of the occurrence of more than one outcome.
Meaning and definition of Probability
Formula for compound probability
As the Oxford dictionary states it, Probability means ‘The • P(A or B) = P(A) + P(B) – P(A and B)
extent to which something is probable; the likelihood of where A and B are any two events.
something happening or being the case’. P(A or B) is the probability of the occurrence of atleast one of
In mathematics too, probability indicates the same – the the events.
likelihood of the occurrence of an event. P(A and B) is the probability of the occurrence of both A and B
at the same time.
Examples of events can be :
• Tossing a coin with the head up Mutually exclusive events:
• Drawing a red pen from a pack of different colored Mutually exclusive events are those where the occurrence of
pens one indicates the non-occurrence of the other
OR
• Drawing a card from a deck of 52 cards etc.
When two events cannot occur at the same time, they are
considered mutually exclusive.
Either an event will occur for sure, or not occur at all. Or there
Note: For a mutually exclusive event, P(A and B) = 0.
are possibilities to different degrees the event may occur.
Example 1: What is the probability of getting a 2 or a 5 when a
An event that occurs for sure is called a Certain event and its
die is rolled?
probability is 1.
Solution:
Taking the individual probabilities of each number, getting a 2
An event that doesn’t occur at all is called an impossible event
is 1/6 and so is getting a 5.
and its probability is 0.
Applying the formula of compound probability,
This means that all other possibilities of an event occurrence lie
Probability of getting a 2 or a 5,
between 0 and 1.
P(2 or 5) = P(2) + P(5) – P(2 and 5)
This is depicted as follows:
==> 1/6 + 1/6 – 0
0 <= P(A) <= 1
==> 2/6 = 1/3.
where A is an event and P(A) is the probability of the
occurrence of the event.
Example 2: Consider the example of finding the probability of
selecting a black card or a 6 from a deck of 52 cards.
This also means that a probability value can never be negative.
Solution:
Every event will have a set of possible outcomes. It is called the
We need to find out P(B or 6)
‘sample space’.
Probability of selecting a black card = 26/52
Probability of selecting a 6 = 4/52
Consider the example of tossing a coin.
Probability of selecting both a black card and a 6 = 2/52
When a coin is tossed, the possible outcomes are Head and
P(B or 6) = P(B) + P(6) – P(B and 6)
Tail. So, the sample space is represented as {H, T}.
= 26/52 + 4/52 – 2/52
Similarly when two coins are tossed, the sample space is
= 28/52
{(H,H), (H,T), (T,H), (T,T)}.
= 7/13.
The probability of head each time you toss the coin is 1/2. So is
Independent and Dependent Events
the probability of tail.
Independent Event
When multiple events occur, if the outcome of one event DOES
Basic formula of probability
NOT affect the outcome of the other events, they are called
independent events.
The Probability of the occurrence of an event A is defined as:
Say, a die is rolled twice. The outcome of the first roll doesn’t
P(A) = (No. of ways A can occur)/(Total no. of possible
affect the second outcome. These two are independent events.
outcomes)
Example 1: Say, a coin is tossed twice. What is the probability
Another example is the rolling of dice. When a single die is
of getting two consecutive tails ?
rolled, the sample space is {1,2,3,4,5,6}.
Probability of getting a tail in one toss = 1/2
What is the probability of rolling a 5 when a die is rolled?
The coin is tossed twice. So 1/2 * 1/2 = 1/4 is the answer.
No. of ways it can occur = 1
Here’s the verification of the above answer with the help of
Total no. of possible outcomes = 6
sample space.
So the probability of rolling a particular number when a die is
When a coin is tossed twice, the sample space is {(H,H), (H,T),
rolled = 1/6.
(T,H), (T,T)}.
Our desired event is (T,T) whose occurrence is only once out of
Compound probability
four possible outcomes and hence, our answer is 1/4.

For details on our programs: - Contact: +91 9930202329, +91, 8291924300 or visit www.campuscredentials.com (64)
Example 2: Consider another example where a pack contains 4 if A is the event of getting a head in coin toss, A c is not getting a
blue, 2 red and 3 black pens. If a pen is drawn at random from head i.e., getting a tail.
the pack, replaced and the process repeated 2 more times, What if A is the event of getting an even number in a die roll, A c is the
is the probability of drawing 2 blue pens and 1 black pen? event of NOT getting an even number i.e., getting an odd
Solution number.
Here, total number of pens = 9 if A is the event of randomly choosing a number in the range of
Probability of drawing 1 blue pen = 4/9 -3 to 3, Ac is the event of choosing every number that is NOT
Probability of drawing another blue pen = 4/9 negative i.e., 0,1,2 & 3 (0 is neither positive or negative).
Probability of drawing 1 black pen = 3/9
Probability of drawing 2 blue pens and 1 black pen = 4/9 * 4/9 Consider the following example:
* 3/9 = 48/729 = 16/243 Example: A single coin is tossed 5 times. What is the
probability of getting at least one head?
Dependent Events Solution:
When two events occur, if the outcome of one event affects the Consider solving this using complement.
outcome of the other, they are called dependent events. Probability of getting no head = P(all tails) = 1/32
Consider the aforementioned example of drawing a pen from a P(at least one head) = 1 – P(all tails) = 1 – 1/32 = 31/32.
pack, with a slight difference.
Example 1: A pack contains 4 blue, 2 red and 3 black pens. If 2 Sample Probability questions with solutions
pens are drawn at random from the pack, NOT replaced and Probability Example 1
then another pen is drawn. What is the probability of drawing 2 What is the probability of the occurrence of a number that is
blue pens and 1 black pen? odd or less than 5 when a fair die is rolled.
Solution: Solution
Probability of drawing 1 blue pen = 4/9 Let the event of the occurrence of a number that is odd be ‘A’
Probability of drawing another blue pen = 3/8 and the event of the occurrence of a number that is less than 5
Probability of drawing 1 black pen = 3/7 be ‘B’. We need to find P(A or B).
Probability of drawing 2 blue pens and 1 black pen = 4/9 * 3/8 P(A) = 3/6 (odd numbers = 1,3 and 5)
* 3/7 = 1/14 P(B) = 4/6 (numbers less than 5 = 1,2,3 and 4)
P(A and B) = 2/6 (numbers that are both odd and less than 5 =
Let’s consider another example: 1 and 3)
Example 2: What is the probability of drawing a king and a Now, P(A or B) = P(A) + P(B) – P(A or B)
queen consecutively from a deck of 52 cards, without = 3/6 + 4/6 – 2/6
replacement. P(A or B) = 5/6.
Probability of drawing a king = 4/52 = 1/13
After drawing one card, the number of cards are 51. Probability Example 2
Probability of drawing a queen = 4/51. A box contains 4 chocobars and 4 ice creams. Tom eats 3 of
Now, the probability of drawing a king and queen consecutively them, by randomly choosing. What is the probability of
is 1/13 * 4/51 = 4/663 choosing 2 chocobars and 1 icecream?
Solution
Conditional probability Probability of choosing 1 chocobar = 4/8 = 1/2
Conditional probability is calculating the probability of an After taking out 1 chocobar, the total number is 7.
event given that another event has already occurred . Probability of choosing 2nd chocobar = 3/7
The formula for conditional probability P(A|B), read as P(A Probability of choosing 1 icecream out of a total of 6 = 4/6 =
given B) is 2/3
P(A|B) = P (A and B) / P(B) So the final probability of choosing 2 chocobars and 1
icecream = 1/2 * 3/7 * 2/3 = 1/7
Consider the following example:
Example: In a class, 40% of the students study math and Probability Example 3
science. 60% of the students study math. What is the probability When two dice are rolled, find the probability of getting a
of a student studying science given he/she is already studying greater number on the first die than the one on the second,
math? given that the sum should equal 8.
Solution Solution
P(M and S) = 0.40 Let the event of getting a greater number on the first die be G.
P(M) = 0.60 There are 5 ways to get a sum of 8 when two dice are rolled =
P(S|M) = P(M and S)/P(S) = 0.40/0.60 = 2/3 = 0.67 {(2,6),(3,5),(4,4), (5,3),(6,2)}.
And there are two ways where the number on the first die is
Complement of an event greater than the one on the second given that the sum should
A complement of an event A can be stated as that which does equal 8, G = {(5,3), (6,2)}.
NOT contain the occurrence of A. Therefore, P(Sum equals 8) = 5/36 and P(G) = 2/36.
A complement of an event is denoted as P(Ac) or P(A’). Now, P(G|sum equals 8) = P(G and sum equals 8)/P(sum
P(Ac) = 1 – P(A) equals 8)
or it can be stated, P(A)+P(Ac) = 1 = (2/36)/(5/36)
For example, = 2/5

For details on our programs: - Contact: +91 9930202329, +91, 8291924300 or visit www.campuscredentials.com (65)
Practice set 1: 13) If you toss a coin and roll a die. What is the probability of
1) A bag contain 6 white balls and 4 red balls .Three balls are getting a head and a 3 on the die?
drawn randomly. What is the probability that one ball is red A) 1/10 B) 1/8 C) 1/12 D) 2/3
and other 2 are white
A) 1/2 B) 2/1 C) 2/3 D) 1/3 14) The probability of two persons of passing the interview are
1/3 and 3/5. Then calculate the probability that only one of
2) A five digit number is formed with the digits 0,1,2,3 and 4 them pass the interview?
without repetition. Find the chance that the number is divisible A) 7/15 B) 8/15 C) 11/15 D) 13/15
by 5
A) 3/5 B) 1/5 C) 2/5 D) 4/5 15) The probability that the problem will be solved by three
persons are 1/2, 1/3 and 1/6. Find the probability that the
3) A dice is thrown twice. What is the probability that at least problem is solved?
one of them comes up with number 4 A) 11/18 B) 13/18 C) 15/18 D) 17/18
A) 12/36 B) 11/36 C) 1/6 D) 11/6
16) An elevator starts with 4 passenger and stops at 7 floors of
4) From a group of 5 men and 2 women, 2 persons are selected an apartment. Find the probability that all 4 passengers travel
at random. Find the probability that at least one woman is to different floor?
selected? A) 110/343 B) 120/343 C) 18/49
A) 11/7 B) 21/11 C) 11/21 D) 1/21 D) 139/343

5) A bag contains 11 red and 5 green balls. Find the probability 17) From a deck of 52 cards, a card is selected and without
that 4 balls are red and 3 balls are green when 7 balls are replacing the card a new card is selected. Find out the
drawn at random. probability that the first card is an ace and second card is a
A) 16/52 B) 15/52 C) 11/52 D) 10/52 king?
A) 4/52*3/51 B) 4/51*3/52
6) A box contain 8 red balls, 6 green balls and 8 blue balls. A C) 4/52*4/51 D) 3/52*3/51
ball is drawn at random. What is the probability that the ball
drawn is neither red nor green? 18) In a class 30% of the students opt for Math, 20% opt for
A) 10/29 B) 2/11 C) 11/4 D) 4/11 Computers and 10% opt for both. A student is selected at
random, find the probability that he has opted either Math or
7) P and Q throw a dice. Find the probability that P’s throw is Computers.
not greater than Q’s through A) 3/5 B) 2/5 C) 4/9 D) 6/11
A) 7/5 B) 12/7 C) 7/12 D) 7/11 E) None of these
E) None of these
19) From a pack of 52 cards, 2 cards are drawn at random.
8) In a college 10% of the students speaking English, What is the probability that either both are red or both are
70% speaking Hindi and 10% speaking both. If a student is kings?
selected at random, what is the probability of the students who A) 55/221 B) 52/225 C) 44/221 D) 48/221
speak English or Hindi? E) None of these
A) 9/5 B) 10/7 C) 7/10 D) 5/7
E) None of these 20) A bag contains 8 blue and 7 green balls. A ball is drawn out
of it and put back in the bag. Then a ball is drawn again. What
9) From a pack of 52 cards, 3 cards are drawn together at is the probability that both the balls are green?
random, what is the probability of both the cards are king? A) 36/225 B) 48/221 C) 49/225
A) 1/5225 B) 1/5525 C) 5525 D) 40/221 E) None of these
D) 1/525 E) None of these
21) The probability of selecting a red ball at random from a jar
10) Out of a total 10 bulbs, 6 bulbs are working in order. One that contains only red, blue and orange balls is 1/4. The
has to choose 2 bulbs out of 6. What is the probability that all probability of selecting a blue ball at random from the same jar
the 2 bulbs will be glowing? 1/3. If the jar contains 10 orange balls, find the total number of
A) 1/3 B) 1/2 C) 1/5 D) 1/6 balls in the jar.
A) 24 B) 36 C) 30 D) 28
11) From a group of 4 men, 3 women, 2 persons are selected at E) None of these
random, Find the probability at least one man is selected?
A) 6/7 B) 7/6 C) 5/6 D) 11/21 22) A box contains tickets numbered from 1 to 24. 3 tickets are
to be chosen to give 3 prizes. What is the probability that at
12) There are 8 positive numbers and 6 negative numbers. 4 least 2 tickets contain a number which is multiple of 3?
numbers are chosen at random and multiplied. Find the A) 35/256 B) 33/220 C) 63/253 D) 43/190
probability that the product is a positive number? E) 59/253
A) 1001/505 B) 101/505
C) 505/1001 D) 450/965 Direction for Questions 23 to 25:

For details on our programs: - Contact: +91 9930202329, +91, 8291924300 or visit www.campuscredentials.com (66)
A Bag contain 6 orange, 4 green,3 pink and 4 black balls
23) If 3 balls are picked at random, what is the probability that
all are pink?
A) 2/650 B) 1/680 C) 4/630 D) 3/680

24) If 5 balls are picked at random, what is the probability that


none are orange?
A) 33/442 B) 442/33 C) 15/167 D) 23/235

25) If 3 balls are drawn at random, what is the probability that


1 is green and the other 2 are black?
A) 2/85 B) 4/87 C) 1/85 D) 3/85

For details on our programs: - Contact: +91 9930202329, +91, 8291924300 or visit www.campuscredentials.com (67)
12) Answer – e) 896
Quantitative Aptitude Solutions Explanation :
Let total marks = M
Chapter 01: Number System (3/8)*M = 300 + 36 = 336
Answer Set 1: M = 112*8 = 896
1) Answer – E. 1155
Explanation : 13)Answer – b) 24
2222.5 – 1260 = 962.5; 962.5*1.2 = 1155 Explanation :
Let the divisor = D
2)Answer – C. 50 so, first number = D*a + 11 and second number = D*b +17
Explanation : so sum of numbers = D*(a + b) + 28
250 / 5 = 50 given that remainder is 4 so, the number is 24

3)Answer -B) 144 14) Answer – c) 4


Explanation : Explanation :
X + Y = 24 let the number be 100a + 10b + c
Greatest Product then x and y must be equal (100a + 10b +c) – (100a + 10c +b) = 36
Possibility of getting 24 b–c=4
12+12= 24
12×12 = 144 15) Answer – A. 4
Explanation :
4) Answer - C)7 LCM of 2, 3, 4, 5 and 6 is 60
Explanation : 60 * 5 + 1 = 7x
(a3+b3+c3-3abc)/(a+b+c) =1/2[(a-b)2+(b-c)2+(c-a)2] = 7x = 301
½(1+4+9) = 14/2 = 7 Number – 301 ; The sum of digits of the number = 4

5)Answer - D)101 16)Answer – E. 179


Explanation : Explanation :
a3 + b3 – a2 + b2 + 2a – 3b odd numbers — x-8, x-6, x-4, x-2, x, x+2, x+4, x+6
(-2)3 + (-5) 3 – (-2)2 + (-5)2 + 2(-2) – 3(-5) = -8 -125-4+25- x-8 + x-6 + x-4 + x-2 + x + x+2 + x+4 + x+6 = 656
4+15 = -101 8x – 8 =656
x = 83
6)Answer - A)29435417 Even numbers — y-2, y, y+2, y+4
Explanation :(9+3+4+7) – (2+4+5+1) = 11 4y + 4 = 87 * 4
y = 86
7)Answer - C)24 sum of the largest even number and odd number = 89 + 90 =
Explanation :100/5 + 100/25 = 20+4 = 24 179

8)Answer - C)2 17)Answer – a) 1


Explanation : Explanation :
X2+1 = 2x Remainder[ 71 / 4 ] = 3, Remainder[ 72 / 4 ] = 1, Remainder[
X2– 2x+1 = 0 73 / 4 ] = 3, Remainder[ 74 / 4 ] = 1 and so on...
X= 1 Pattern repeats in cycles of 2. Remainder [ 7n / 4 ] is 3 when n
x10 + (-1/x)10 = 1+1 = 2 is odd and is 1 when n is even.
7100 when divided by 4 gives a remainder of 1.
9)Answer - B)1
Explanation : 18)Answer – b) 2
Avg of 7 no= (7+1)/2 = 8/2 = 4 Explanation :
Avg of 9 no = (9+1)/2 = 5 Use negative remainder concept,
5- 4 = 1 Remainder [ 41 * 42 / 13 ] = Remainder[(-2) * (-1) / 43 ]( as
Hence increased by 1 41 = 43 * 1 – 2 and 42 = 43 * 1 – 1)
= Remainder [ 2 /43 ] = 2
10)Answer - D)7
Explanation :2*3*5*7*11*13*17 = 510510 19)Answer – a) 4
Explanation:
11)Answer – c) 41
51 / 7 2 So
Explanation :
LCM of 4,6 and 8 is 24
Divide 1000 by 24, we get quotient = 41 and 16 as remainder
so 41 numbers are there which are divisible by 4,6 and 8
together.

For details on our programs: - Contact: +91 9930202329, +91, 8291924300 or visit www.campuscredentials.com (68)
The prime-factorization of 26 * 55 * 76 * 107 is 213 * 512 * 76
20) Answer – d) 13 The total number of factors of N = 14 * 13 * 7
Explanation: We need to find the total number of even factors. For this, let us
find the total number of odd factors and then subtract this from
the total number of factors. Any odd factor will have to be a
combination of powers of only 5 and 7.
Here our aim is obtained number as 24 = 16 ( 16 = 17 – 1) Total number of odd factors of 213 * 512 * 76 = (12 + 1) * (6 +
so according to that rewrite the equation as follow as 1) = 13 * 7
Total number of factors = (13 + 1) * (12 + 1) * (6 + 1)
Total number of even factors = 14 * 13 * 7 - 13 * 7
Number of even factors = 13 * 13 * 7 = 1183

26)Answer: a) A and B
Final remainder is 17 – 4 = 13 Explanation:
Any number of the form paqbrc will have (a + 1) (b + 1) (c + 1)
21) Answer - a) 4 factors, where p, q, r are prime. In order for the number to be a
Explanation: perfect cube a, b, c will have to be multiples of 3.
1080 = 23 * 33 * 5. For any perfect square, all the powers of We can assume that a = 3m, b = 3n, c = 3l.
the primes have to be even numbers. So, if the factor is of the This tells us that the number of factors will have to be of the
form 2a * 3b * 5c. form (3n + 1) * (3m + 1) * (3l + 1). In other words (a + 1), (b
The values 'a' can take are 0 and 2, b can take are 0 and 2, and + 1) and (c + 1) all leave a remainder of 1 on division by 3. So,
c can take the value 0. the product of these three numbers should also leave a
Totally there are 4 possibilities. 1, 4, 9, and 36. remainder of 1 on division by 3. Of the four numbers provided,
16 and 28 can be written in this form, the other two cannot..
22)Answer -b)24 So, a perfect cube can have 16 or 28 factors. Now, let us think
Explanation: about what kind of numbers will have 16 factors..
Any factor of this number should be of the form 2 a * 3b * 5c. A number of the form p15 or q3r3 will have exactly 16 factors.
For the factor to be a perfect square a,b,c have to be even. Both are perfect cubes. Note that there are other prime
a can take values 0, 2, 4... factorizations possible that can have exactly 16 factors. But
b can take values 0, 2, 4, 6... these two forms are perfect cubes, which is what we are
and c can take values 0, 2... interested in.
Total number of perfect squares = 3 * 4 * 2 = 24. Similarly, a number of the form p27 or q3r6 will have 28 factors.
Both are perfect cubes.
23)Answer: -c)20
Explanation: 27) Answer – D) 16
Any factor of this number should be of the form 2 a * 3b * 5c. Explanation – L.C.M. of 2, 4, 6, 8, 10, 12 is 120.So, the bells
For the factor to be an odd number, will toll together after every 120 seconds, i.e, 2 minutes. In 30
a should be 0. minutes, they will toll together 30/2 + 1 = 16
b can take values 0, 1, 2, 3.
and c can take values 0, 1, 2, 3, 4. 28)Answer – D. 364
Total number of odd factors = 4 * 5 = 20. Explanation – L.C.M. of 6, 9, 15 and 18 is 90.
Let required number be 90k + 4, which is multiple of 7.
24)Answer: c) 594 Least value of k for which (90k + 4) is divisible by 7 is k = 4.
Explanation: Required number = (90 x 4) + 4 = 364.
The prime factorization of 28 * 36 * 54 * 105 is 213 * 36 * 59.
For any of these factors questions, start with the prime 29)Answer – B. 15110
factorization. Remember that the formulae for number of Explanation – Here (48 – 38) = 10, (60 – 50) = 10, (72 – 62) =
factors, sum of factors, are all linked to prime factorization. 10, (108 – 98) = 10 & (140 – 130) = 10.
120 can be prime-factorized as 23 * 3 * 5. Required number = (L.C.M. of 48, 60, 72, 108, 140) – 10
All factors of 213 * 36 * 59 that can be written as multiples of = 15120 – 10 = 15110
120 will be of the form 23 * 3 * 5 * K.
213 * 36 * 59 = 23 * 3 * 5 * K 30)Answer – C. 40
=> K = 210 * 35 * 58. Explanation – Let the numbers be 2x and 3x.
The number of factors of N that are multiples of 120 is identical Then, their L.C.M. = 6x.
to the number of factors of K. So, 6x = 48 or x = 8.
Number of factors of K = (10 + 1) (5 + 1) * (8 + 1) = 11 * 6 * The numbers are 16 and 24.
9 = 594 Hence, required sum = (16 + 24) = 40.

25)Answer: a) 1183 31)Answer – D. 548


Explanation: Explanation – Required number = (L.C.M. of 12, 15, 20, 54) +
8

For details on our programs: - Contact: +91 9930202329, +91, 8291924300 or visit www.campuscredentials.com (69)
= 540 + 8 𝟏𝟎 𝟒
935421 x 625 = 935421 x 54 = 935421 x ( )
= 548. 𝟐
𝟗𝟑𝟓𝟒𝟐𝟏 × 𝟏𝟎𝟒 𝟗𝟑𝟓𝟒𝟐𝟏𝟎𝟎𝟎𝟎
=
32)Answer – B. 2 𝟐𝟒 𝟏𝟔
Explanation – Let the numbers 13a and 13b. = 584638125
Then, 13a x 13b = 2028
ab = 12. 7) Answer : Option D
Now, the co-primes with product 12 are (1, 12) and (3, 4). Explanation:
Clearly, 97 is a prime number.
33) Answer – C. 23
Explanation – L.C.M. of 5, 6, 4 and 3 = 60. On dividing 2497 8) Answer : Option A
by 60, the remainder is 37. Number to be added = (60 – 37) = Explanation:
23 5358 x 51 = 5358 x (50 + 1)
= 5358 x 50 + 5358 x 1
34)Answer : A) 4/126 = 267900 + 5358
Explanation: = 273258.
HCF of numerator(4,8,36,20) = 4
LCM of denominator(3,6,63,42) = 126 9) Answer : Option D
Explanation:
35)Answer – B) 147 Required sum = (2 + 3 + 5 + 7 + 11) = 28.
Explanation : Note: 1 is not a prime number.
Definition: A prime number (or a prime) is a natural number
that has exactly two distinct natural number divisors: 1 and
itself.

10) Answer : Option A


Answer Set 2: Explanation:
(𝟏𝟐)𝟑 ×𝟔𝟒 (𝟏𝟐)𝟑 ×𝟔𝟒
1) Answer - C (729) Given Exp. = = = (12)2 x 62 = (72)2 = 5184
𝟒𝟑𝟐 𝟏𝟐×𝟔𝟐
Explanation - Given Exp.
a2 + b2 - 2ab, where a = 186 and b = 159
11) Answer : Option A
= (a - b)2 = (186 - 159)2 = (27)2
Explanation:
= (20 + 7)2 = (20)2 + 7 + 2 x 20 x 7 = 400 + 49 + 280 = 729
72519 x 9999 = 72519 x (10000 - 1)
= 72519 x 10000 - 72519 x 1
2) Answer : Option D
= 725190000 – 72519
Explanation:
= 725117481.
Let the number be x.
𝟏 𝟏
Then, of of x = 15 x = 15 x 12 = 180 12) Answer : Option A
𝟑 𝟒
𝟑 Explanation:
So, Required number = ( × 𝟏𝟖𝟎) = 𝟓𝟒
𝟏𝟎
The smallest 3-digit number is 100, which is divisible by 2.
100 is not a prime number.
3) Answer : Option D
Explanation: √𝟏𝟎𝟏< 11 and 101 is not divisible by any of the prime numbers
91 is divisible by 7. So, it is not a prime number. 2, 3, 5, 7, 11.
101 is a prime number.
Hence 101 is the smallest 3-digit prime number.
4) Answer : Option B
Explanation:
13) Answer : Option A
𝟏𝟎 𝟒 𝟏𝟏𝟐 × 𝟏𝟎𝟒 𝟏𝟏𝟐𝟎𝟎𝟎𝟎
(𝟏𝟏𝟐 𝒙 𝟓𝟒 ) = 𝟏𝟏𝟐 × ( ) = = Explanation:
𝟐 𝟐𝟒 𝟏𝟔 19657 Let x - 53651 = 9999
= 𝟕𝟎𝟎𝟎𝟎
33994 Then, x = 9999 + 53651 = 63650
-----
5) Answer : Option A
53651
Explanation:
-----
1397 x 1397 = (1397)2
= (1400 - 3)2
14) Answer : Option A
= (1400)2 + (3)2 - (2 x 1400 x 3)
Explanation:
= 1960000 + 9 – 8400
Let Sn =(1 + 2 + 3 + ... + 45). This is an A.P. in which a =1, d
= 1960009 – 8400
=1, n = 45.
= 1951609 𝒏
Sn = 2a + (n - 1)d)
𝟐
𝟒𝟓
6) Answer : Option B = x 2 x 1 + (45 - 1) x 1)
𝟐
Explanation: 𝟒𝟓
= x 46
𝟐

For details on our programs: - Contact: +91 9930202329, +91, 8291924300 or visit www.campuscredentials.com (70)
= = (45 x 23) Explanation - (1399 x 1399) = (1399)2
= 45 x (20 + 3) = (1400 - 1)2 = (1400)2 + 12 - 2 x 1400 x 1
= 45 x 20 + 45 x 3 = 1960000 + 1 - 2800 = 1960001 - 2800 = 1957201
= 900 + 135
= 1035. 26) Answer: B.
Shorcut Method:
𝒏(𝒏+𝟏) 𝟒𝟓(𝟒𝟓+𝟏) 27) Answer: A
Sn = = = 𝟏𝟎𝟑𝟓
𝟐 𝟐

28) Answer: A
15) Answer : Option B
Explanation:
Explanation:
We can write 17 as 24+ 1 and 2256 as (24)64.
x + 3699 + 1985 - 2047 = 31111
If f(x) is divided by (x – a), the remainder is f(a))
x + 3699 + 1985 = 31111 + 2047
∴ The remainder is (– 1)64 = 1.
x + 5684 = 33158
x = 33158 - 5684 = 27474.
29) Answer : D
Explanation:
Let the three integers be x, x + 2 and x + 4.
16) Answer & Explanation
Then, 3x = 2(x + 4) + 3 x = 11.
Answer - B (2079)
Third integer = x + 4 = 15
Explanation - (a) 639 is not divisible by 7
(b) 2079 is divisible by 3, 7, 9 and 11
30) Answer : B
(c) 3791 is not divisible by 3
Explanation:
(d) 37911 is not divisible by 9 𝟖
Let the ten's and unit digit be x and respectively.
𝒙
17) Answer - B (43974) 𝟖 𝟖
Then, (𝟏𝟎𝒙 + ) + 𝟏𝟖 = 𝟏𝟎𝒙 + 𝒙
𝒙 𝒙
Explanation - 39798 + 3798 + 378 = 43974
10x2 + 8 + 18x = 80 + x2
9x2 + 18x - 72 = 0
18) Answer - A (114345)
x2 + 2x - 8 = 0
Explanation - The required number should be divisible by both
(x + 4)(x - 2) = 0
9 and 11.
x = 2.
Clearly, 114345 is divisible by both 9 and 11. So, it is divisible
by 99

19) Answer - B (875591244)


Explanation - 8756 x 99999 = 8756 x (100000 - 1) = (8756 x
100000) - (8756 x 1)
= (875600000 - 8756) = 875591244
20) Answer - C (4691100843)
Explanation - 469157 x 9999 = 469157 x (10000 -1) =
(469157 x 10000) - (469157 x 1)
= (4691570000 - 469157) = 4691100843

21) Answer - B (584638125)


Explanation - 935421 x 625 = 935421 x 54
= 9354210000 / 24
= 9354210000 / 16
= 584638125

22) Answer - B (Product of two odd numbers)


Explanation - Product of two odd numbers is always odd.

23) Answer - D (462)


Explanation - On dividing 457 by 11, remainder is 6.
Required number is either 451 or 462. Nearest to 456 is 462

24) Answer - E (None of these)


Explanation - 100 is divisible by 2, so it is not prime.
101 is not divisible by any of the numbers 2, 3, 5, 7. So, it is
prime.
Hence, the smallest 3-digit prime number is 101

25) Answer - D (1957201)

For details on our programs: - Contact: +91 9930202329, +91, 8291924300 or visit www.campuscredentials.com (71)
Chapter 02: Percentages 8) Answer:
Practice Set 1: Explanation: C
1) Answer: B This is a simple question of percentage change
Explanation: The percentage change = (final value – initial value ) / (initial
Number of runs made by running = 110 - (3 x 4 + 8 x 6) value) x 100
= 110 - (60) So in 2011 Sita’s salary was 1000 i.e the initial value and then
= 50. her salary increased to 1250, that is final value.
𝟓𝟎 𝟓 So the percentage change is (1250-1000) / 1000 x 100 = 25%
Required percentage = ( × 𝟏𝟎𝟎) % = 𝟒𝟓 %
𝟏𝟏𝟎 𝟏𝟏
Or 9) Answer
Answer – (B) Explanation : Option B
Answer:: Interest on Rs.22500 = 0.1 x 22500 = 2250
Number of runs made by running =110−(3*4+8*6) Charges for managing the concern = 60 x 12 = 720
= 110 - (60) If yearly profit is x, then B's share and A's share = (x/2)
= 50. ∴ (x/2)-2250 + 720
𝟓𝟎 𝟓
Required percentage = × 𝟏𝟎𝟎% = 45 % = (1/2)((x/2) -720 + 2250)
𝟏𝟎𝟎 𝟏𝟏
=> x = Rs. 9180.
2) Answer - B (8.5)
Explanation - Let a% of 25 = 2.125. Then , (a/100) x 25 = 10) Answer: Option E
2.125 a = (2.125 x 4) = 8.5 Explanation:
Let the number of students be x. Then,
3) Answer : D Number of students above 8 years of age = (100 - 20)% of x =
Explanation : 80% of x.
𝟐
Let 218% of 1674 = x x 1800. 80% of x = 48 + of 48
𝟑
𝟐𝟏𝟖 𝟏 𝟖𝟎
Then, 𝒙 = × 𝟏𝟔𝟕𝟒 × = 𝟐. 𝟎𝟐𝟕𝟒. x = 80
𝟏𝟎𝟎 𝟏𝟖𝟎𝟎 𝟏𝟎𝟎
x = 100.
4) Answer : C
Explanation : 11) Answer - C (14)
Let √𝟕𝟖𝟒 + x = 78% of 500. Explanation - Required difference
𝟕𝟖
Then, 𝒙 = × 𝟓𝟎𝟎 − √𝟕𝟖𝟒 = (𝟑𝟗𝟎 − 𝟐𝟖) = 𝟑𝟔𝟐. = 3 ½ % of Rs.8400 – 3 1/3 % of Rs.8400
𝟏𝟎𝟎
= (7/2-(10/3)% of Rs.8400
5) Answer: =1/6 % of Rs.8400
= Rs. (1/6)x (1/100) x 8400
Explanation:
Let the number of candidates appeared from each state be x. = Rs. 14
Then, 7% of x - 6% of x = 80
‹=›1% of x = 80 12) Answer : B
‹=› x= 80 ×100 Explanation :
Total number of students = 1100 + 700 = 1800.
=8000.
Numbers of student passed = (42% of 1100 + 30% of 700) =
(642 + 210) = 672
6) Answer:
Explanation: Number of failures = 1800 – 672 = 1128.
𝟏𝟏𝟐𝟖 𝟐
Let A be the income. Percentage failure = × 𝟏𝟎𝟎% = 𝟔𝟐 %
𝟏𝟖𝟎𝟎 3
Expenditure = 0.8 A
Savings = 0.2 A => 20% 13) Answer : C
New income = 1.2 A (since 20% rise) Explanation :
New expenditure = (0.8 A) X 1.3 (Since 30% rise) Total sales tax paid = 7% of Rs. 400 + 9% of Rs. 6400
= 1.04A 𝟕 𝟗
= 𝑹𝒔. × 𝟒𝟎𝟎 + × 𝟔𝟒𝟎𝟎 = 𝑹𝒔. (𝟐𝟖 + 𝟓𝟕𝟔)
So, new savings = 1.2A – 1.04A = 0.16A => 16% 𝟏𝟎𝟎 𝟏𝟎𝟎
% decrease = (20-16)/20 X 100 = 20% = 𝑹𝒔. 𝟔𝟎𝟒.
Total cost of the items = Rs. (400 + 6400) = Rs. 6800.
7) Answer 𝟔𝟎𝟒 𝟏𝟓
Required percentage = × 𝟏𝟎𝟎% = 𝟖 %
𝟔𝟖𝟎𝟎 𝟏𝟕
Explanation:
14) Answer : B
Let the total marks be 100
Explanation :
Minimum marks required to pass the exam =40 𝟑𝟖𝟒 𝟏
Marks obtained by A = 40 – (40 x 10/100) = 36 marks × 𝟏𝟎𝟎% = 𝟕𝟏 %
𝟓𝟒𝟎 𝟗
Marks obtained by B= 36 – (100/9 x 36/100) = 32 marks 𝟒𝟐𝟓
Marks obtained by C = (36 +32) – ( 36+32) x 700/(17 x100) × 𝟏𝟎𝟎% = 𝟖𝟓%
𝟓𝟎𝟎
= 68 – 28 𝟓𝟕𝟎 𝟑
= 40 marks × 𝟏𝟎𝟎% = 𝟖𝟏 %
𝟕𝟎𝟎 𝟕

For details on our programs: - Contact: +91 9930202329, +91, 8291924300 or visit www.campuscredentials.com (72)
𝟒𝟖𝟎 𝟖
× 𝟏𝟎𝟎% = 𝟕𝟐 %
𝟔𝟔𝟎 𝟏𝟏 21) Answer: Option D
𝟒𝟐𝟓
shows the best percentage Explanation:
𝟓𝟎𝟎 𝟐
5% of A + 4% of B = (6% of A + 8% of B)
𝟑
15) Answer: 𝟓 𝟒 𝟐 𝟔 𝟖
𝑨+ 𝑩= ( 𝑨+ 𝑩)
Explanation: 𝟏𝟎𝟎 𝟏𝟎𝟎 𝟑 𝟏𝟎𝟎 𝟏𝟎𝟎
𝟏 𝟏 𝟏 𝟒
Amount paid to car owner 𝑨+ 𝑩= 𝑨+ 𝑩
𝟐𝟎 𝟐𝟓 𝟐𝟓 𝟕𝟓
= 90% of 85% of Rs. 3,25,000. 𝟏 𝟏 𝟒 𝟏
( − )𝑨 = ( − )𝑩
= Rs. (90/100 ×85/100 ×325000) 𝟐𝟎 𝟐𝟓 𝟕𝟓 𝟐𝟓
𝟏 𝟏
= Rs. 2,48,625. 𝑨= 𝑩
𝟏𝟎𝟎 𝟕𝟓
Required differene = Rs. (325000 - 248625) 𝑨 𝟏𝟎𝟎 𝟒
= =
𝑩 𝟕𝟓 𝟑
= Rs. 76,375.
Required ratio = 4 : 3
16) Answer: C
22) Answer: Option A
Explanation:
Explanation:
We solve this question using the principle of constancy. 𝟔
Let original price = P Rebate = 6% of Rs. 6650 = Rs. ( × 𝟔𝟔𝟓𝟎) = 𝑹𝒔. 𝟑𝟗𝟗
𝟏𝟎𝟎
Increased Price- 1.25P 𝟏𝟎
Sales tax = 10% of Rs. (6650 - 399) = Rs. ( × 𝟔𝟐𝟓𝟏) =
𝟏𝟎𝟎
Original Consumption= C
𝑹𝒔. 𝟔𝟐𝟓. 𝟏𝟎
Increased Consumption= I
Final amount = Rs. (6251 + 625.10) = Rs. 6876.10
It is given that expense is the same, that is:
1.25 x I = P x C
23) Answer - B (2490 and 4150)
I=0.8 C
Explanation - Let the numbers be a and b.
% decrease = {(1-0.8)/1} X 100 = 20%
Then , 7.5 % of a =12.5% of b
a = 125 x b/75 = 5 x b/3.
17) Answer:
Now, a - b =1660
Explanation: B
5 x b/3 –b =1660
Let the monthly salary be (s)
2 x b/3= 1660
So going by the condition given in the question, we have:
b = (1660 x 3)/2) =2490.
4s/10 + (10s-4s)/10 x 50/100 +(6s-3s)/10 x 30s/100 + saving =
One number = 2490, Second number =5 x b/3 =4150
s
4s/10 + 6s/10 x 50/100 +3s/10 x 30s/100 + saving = s
24) Answer - B (2%)
=> 4s/10 + 3s/10 + 9s/10 + 630 =s
Explanation - Required Percentage = (130/(6.5 x 1000)) x
=> 630 = (100s – 70s – 9s)/100
100) % = 2%.
=> 630 = 21s/100
=> s = Rs 3000
25) Answer - A (16800)
Explanation - Let the number of votes enrolled be a.
18) Answer: Option C
Then,Number of votes cast =75% of a.
Explanation :
Valid votes = 98% of (75% of a). 75% of (98% of (75%of a))
Growth rate of rat population in 3 months = 20 *(3/12) = 5%
=9261.
Increase in first 3 months = 3200 x 1.05 = 3360
(75/100) x (98/100) x (75/100)x a) =9261.
Also, net decrease in 3 months = 160
a = (9261 x 100x 100 x 100)/(75 x 98 x 75)) =16800
Rat population = 3360 - 160 = 3200
In the same way, after every 3 months, rat population remains
26) Answer : B)
the same
Explanation :
Hence, even after 3 x 8 months i.e., 2 years, the population is
A = 150% of B
Maintained

19) Answer: Option A


Explanation :
CP of the merchant who calculates his % profit on CP =
(3760/1.175) =Rs.3200
His profit = 0.175 x 3760 = Rs.560
Profit of the merchant who calculates his % profit on SP =
0.175 x 3760 = Rs.658
Difference in profit = 658 - 560 =Rs.98.

20) Answer: Option E Required percentage =


Explanation:
𝑿 𝒚
x% of y = ( × 𝒚) = ( × 𝒙) == y% of x
𝟏𝟎𝟎 𝟏0𝟎
A = B.

For details on our programs: - Contact: +91 9930202329, +91, 8291924300 or visit www.campuscredentials.com (73)
27) Answer : B Let the original length and breadth be 10 cm each then original
Explanation : area = 100 cm²
Let one number = x, Then, other number = 80% of x =
𝟒
New length = 10  1.1 = 11 cm
𝟓
Let new breadth be x.
Then,11 x = 100

Hence % reduction in breadth =

So, the numbers are 10 and 8.


5) Answer: D
28) Answer : C Explanation:
Explanation : Rate is increased from Rs. 13 per h to Rs. 19.5 per h.
Amount paid to car owner = 90% of 85% of Rs. 3,25,000 Commission = 19.5  50 = Rs. 975

= Rs. 2,48,625 6) Answer: D


Required difference = Rs. (325000 – 248625) = Rs. 76,375 Explanation:
A = 5B
29) Answer: B is less than A by (5B – B = 4B
𝟒𝑩
Explanation: ( × 𝟏𝟎𝟎) % = 𝟖𝟎%
Let the number of applicants be x.Number of eligible 𝟓𝑩
candidates = 95% of x.
Eligible candidates of each other categories = 15% of (95% of 7) Answer: C
Explanation:
x).
=(15/100×95/100×x) Marks obtained in Hindi = 24.
Marks obtained in English = 30
= 57/400×x.
Marks obtained in Maths = 36
Therefore, 57/400×x = 4275
‹=› x =(4275×400 / 57) Marks obtained in Science = 42.
Total marks obtained = 132.
‹=› 30000.
8) Answer: C
30) Answer:
Explanation: Explanation:
Total number of votes polled = (1136 + 7636 + 11628 ) = Quantity of water in 15 liters = 6% of 15 liters = 0.9 liters.
Let x liters of pure milk be added. Then,
20400.
Required percentage = (11628 / 20400 ×100)% = 57%.

Practice Set 2: 9) Answer: C


1) Answer: D Explanation:
Explanation: Let the total income be x.
Let the maximum marks be x Then, income left = (100 - 70)% of [100 – (25 + 45)]% of x =
Then, 35% of x = 165 + 45 30% of 30% of x

2) Answer: C 10) Answer: D


Explanation: Explanation:
A  1.4  0.5 = B  1.5  0.6
 .7A = .9B
 B/A = 7/ 9

3) Answer: A
Explanation:

 A% of B is B% of A.

4) Answer: D
Explanation:

For details on our programs: - Contact: +91 9930202329, +91, 8291924300 or visit www.campuscredentials.com (74)
17) Answer: C
11) Answer: D Explanation:
Explanation: Since sales tax and profit are to be calculated on
Percentage change in area manufacturing cost, therefore
single percentage change = 30%
Now manufacturing cost  1.3 = 16,900

 Manufacturing cost =
12) Answer: B
Explanation:
18) Answer: D
Let the original price be Rs. x per kg
Explanation:
We know that = Price  Consumption = Expenses
 Original expenses = x  25 = Rs. 25 x
After increase, let the new consumption be y kg
Given, New price = 1.30 x
New expenses = 1.3  25 x 1.30 x  y = 1.3  25x

19) Answer: A
Hence, new monthly consumption = 25 kg. Explanation:
Number of questions attempted correctly = (80% of 40 + 60%
13) Answer: A of 30 + 50% of 10)
Explanation: = (32 + 18 + 5) =55
Let total marks = x. Then, (30% of x) + 18 = (35% of x) – 12 Questions to be answered correctly for 75% grade = 75% of 80
= 60.
 Required Number of questions = (60 -55) = 5

20) Answer: C
Explanation:
Let the original salary be Rs. 100 New salary = Rs. 80.
Increase on 80 = 20. Increase on 100 =

14) Answer: B
Explanation:
Let the price of a chair be Rs. x. Then, price of a table = Rs. (x 21) Answer: C
+ 350). Explanation:
So, 7 (x + 350) + 7x = 4550 Population in the beginning of the first year
 14 x = 4550 – 2450

Price of a table = Rs. 500 ; Price of a chair = Rs. 150.


350 22) Answer: B
Required percentage = ( × 100) % = 70%
500
Explanation:
Let original rate be Rs. x per kg.
15) Answer: B
Explanation:
Suppose a person used to purchase x kg rice for Rs. 100 before
reduction. Then after reduction, he is getting 11.5 kg more in
addition to x kg for the same price i.e. Rs. 100.
It means a reduction of 23% in Rs. 100, i.e. Rs 23 enables him
to buy 11.5 kg more.
Therefore reduced price is Rs. 23 for 11.5 kg or Rs. 2 per kg.

16. Answer: B
Explanation: 23) Answer: D
Explanation:
Value of the machine after 2 years

So people who had either or both types of dinner = 96.


Hence, people who had neither type of dinner = (120 - 96) = 24
24) Answer: C

For details on our programs: - Contact: +91 9930202329, +91, 8291924300 or visit www.campuscredentials.com (75)
Explanation: = 110 - (60)
Let the total number of student be x = 50.
Let A and B represent the sets of students who passed in 5𝟎 𝟓
Required percentage = ( × 𝟏𝟎𝟎)%=45 %
𝟏𝟏𝟎 𝟏𝟏
English and Mathematics respectively.
Or
Then, number of students passed in one or both the subjects
Answer – (B)
Answer::
Number of runs made by running =110−(3*4+8*6)
= 110 - (60)
= 50.
𝟓𝟎 𝟓
Required percentage = ×100% = 45 %
𝟏𝟎𝟎 𝟏𝟏

2) Answer - B (8.5)
Explanation - Let a% of 25 = 2.125. Then , (a/100) x 25 =
2.125 a = (2.125 x 4) = 8.5
25) Answer: D
Explanation: 3) Answer : D
Let the total number of votes polled be x. Explanation :
Then, votes polled in favor of other candidate = (100 - 82)% of Let 218% of 1674 = x x 1800.
𝟐𝟏𝟖 𝟏
x = 18% of x. Then, 𝑥= ×1674× =2.0274.
𝟏𝟎𝟎 𝟏𝟖𝟎𝟎
 82% of x – 18% of x = 512 4) Answer : C
Explanation :
Let √𝟕𝟖𝟒 + x = 78% of 500.
𝟕𝟖
26) Answer: B Then, 𝑥= ×500−√𝟕𝟖𝟒=(𝟑𝟗𝟎 − 𝟐𝟖)=362.
𝟏𝟎𝟎
Explanation:
Given M + F = 6400 5) Answer:
1.4 M + 1.3 F = 8380 Explanation:
 M = 600 and F = 5800 Let the number of candidates appeared from each state be x.
Then, 7% of x - 6% of x = 80
27) Answer: B ‹=›1% of x = 80
Explanation: ‹=› x= 80 ×100
Let the inspector examined x meters, then 0.06% of x = 3 =8000.

6) Answer:
28) Answer: C Explanation:
Explanation: Let A be the income.
Let the quantity of milk and water be 40 and 60 respectively, Expenditure = 0.8 A
after removing 50% of solution Savings = 0.2 A => 20%
Quantity of milk = 20 and quantity of water = 30 New income = 1.2 A (since 20% rise)
Therefore, the concentration of the solution is reduced from 40 New expenditure = (0.8 A) X 1.3 (Since 30% rise)
to 20 i.e., reduced by 50% = 1.04A
29) Answer: C So, new savings = 1.2A – 1.04A = 0.16A => 16%
Explanation: % decrease = (20-16)/20 X 100 = 20%
Number of literate women =
7) Answer
Explanation:
Let the total marks be 100
30) Answer: C Minimum marks required to pass the exam =40
Explanation: Marks obtained by A = 40 – (40 x 10/100) = 36 marks
Amount of salt in 40 kg solution = Marks obtained by B= 36 – (100/9 x 36/100) = 32 marks
Marks obtained by C = (36 +32) – ( 36+32) x 700/(17 x100)
= 68 – 28
Let x kg of pure salt be added. = 40 marks

8) Answer:
Explanation: C
This is a simple question of percentage change
Chapter 02: Percentages The percentage change = (final value – initial value ) / (initial
1) Answer: B value) x 100
Explanation: So in 2011 Sita’s salary was 1000 i.e the initial value and then
Number of runs made by running = 110 - (3 x 4 + 8 x 6)

For details on our programs: - Contact: +91 9930202329, +91, 8291924300 or visit www.campuscredentials.com (76)
her salary increased to 1250, that is final value. = Rs. 2,48,625.
So the percentage change is (1250-1000) / 1000 x 100 = 25% Required differene = Rs. (325000 - 248625)
= Rs. 76,375.
9) Answer
Explanation : Option B 16) Answer: C
Interest on Rs.22500 = 0.1 x 22500 = 2250 Explanation:
Charges for managing the concern = 60 x 12 = 720 We solve this question using the principle of constancy.
If yearly profit is x, then B's share and A's share = (x/2) Let original price = P
∴ (x/2)-2250 + 720 Increased Price- 1.25P
= (1/2)((x/2) -720 + 2250) Original Consumption= C
=> x = Rs. 9180. Increased Consumption= I
It is given that expense is the same, that is:
10) Answer: Option E 1.25 x I = P x C
Explanation: I=0.8 C
Let the number of students be x. Then, % decrease = {(1-0.8)/1} X 100 = 20%
Number of students above 8 years of age = (100 - 20)% of x =
80% of x. 17) Answer:
𝟐 Explanation: B
80% of x = 48 + of 48
𝟑
𝟖𝟎 Let the monthly salary be (s)
x = 80 So going by the condition given in the question, we have:
𝟏𝟎𝟎
x = 100. 4s/10 + (10s-4s)/10 x 50/100 +(6s-3s)/10 x 30s/100 + saving =
s
11) Answer - C (14) 4s/10 + 6s/10 x 50/100 +3s/10 x 30s/100 + saving = s
Explanation - Required difference => 4s/10 + 3s/10 + 9s/10 + 630 =s
= 3 ½ % of Rs.8400 – 3 1/3 % of Rs.8400 => 630 = (100s – 70s – 9s)/100
= (7/2-(10/3)% of Rs.8400 => 630 = 21s/100
=1/6 % of Rs.8400 => s = Rs 3000
= Rs. (1/6)x (1/100) x 8400
= Rs. 14 18) Answer: Option C
Explanation :
12) Answer : B Growth rate of rat population in 3 months = 20 *(3/12) = 5%
Explanation : Increase in first 3 months = 3200 x 1.05 = 3360
Total number of students = 1100 + 700 = 1800. Also, net decrease in 3 months = 160
Numbers of student passed = (42% of 1100 + 30% of 700) = Rat population = 3360 - 160 = 3200
(642 + 210) = 672 In the same way, after every 3 months, rat population remains
Number of failures = 1800 – 672 = 1128. the same
𝟏𝟏𝟐𝟖 𝟐
Percentage failure = ×100%=62 % Hence, even after 3 x 8 months i.e., 2 years, the population is
𝟏𝟖𝟎𝟎 𝟑
Maintained
13) Answer : C
Explanation : 19) Answer: Option A
Total sales tax paid = 7% of Rs. 400 + 9% of Rs. 6400 Explanation :
𝟕 𝟗 CP of the merchant who calculates his % profit on CP =
=𝑅𝑠. ×400+ ×6400=𝑅𝑠. (𝟐𝟖 + 𝟓𝟕𝟔)=𝑅𝑠. 604. (3760/1.175) =Rs.3200
𝟏𝟎𝟎 𝟏𝟎𝟎
Total cost of the items = Rs. (400 + 6400) = Rs. 6800. His profit = 0.175 x 3760 = Rs.560
𝟔𝟎𝟒 𝟏𝟓
Required percentage = ×100%=8 % Profit of the merchant who calculates his % profit on SP =
𝟔𝟖𝟎𝟎 𝟏𝟕
0.175 x 3760 = Rs.658
14) Answer : B Difference in profit = 658 - 560 =Rs.98.
Explanation :
𝟑𝟖𝟒 𝟏 20) Answer: Option E
×100%=71 %
𝟓𝟒𝟎 𝟗 Explanation:
𝟒𝟐𝟓 𝑿 𝒚
×100%=85% x% of y = ( × 𝒚)= ( × 𝒙)== y% of x
𝟓𝟎𝟎 𝟏𝟎𝟎 𝟏𝟎𝟎
𝟓𝟕𝟎 𝟑
×100%=81 % A = B.
𝟕𝟎𝟎 𝟕
𝟒𝟖𝟎 𝟖
×100%=72 %
𝟔𝟔𝟎 𝟏𝟏 21) Answer: Option D
𝟒𝟐𝟓
shows the best percentage Explanation:
𝟓𝟎𝟎
𝟐
5% of A + 4% of B = (6% of A + 8% of B)
𝟑
15) Answer: 𝟓 𝟒 𝟐 𝟔 𝟖
Explanation: 𝟏𝟎𝟎
𝐴+𝟏𝟎𝟎𝐵=𝟑 (𝟏𝟎𝟎 𝑨 + 𝟏𝟎𝟎 𝑩)
𝟏 𝟏 𝟏 𝟒
Amount paid to car owner
𝟐𝟎
𝐴+𝟐𝟓𝐵=𝟐𝟓 𝑨 + 𝟕𝟓 𝑩
= 90% of 85% of Rs. 3,25,000. 𝟏 𝟏 𝟒 𝟏
( − )𝑨=( − )𝑩
= Rs. (90/100 ×85/100 ×325000) 𝟐𝟎 𝟐𝟓 𝟕𝟓 𝟐𝟓

For details on our programs: - Contact: +91 9930202329, +91, 8291924300 or visit www.campuscredentials.com (77)
𝟏 𝟏
𝟏𝟎𝟎 𝟕𝟓
𝑨= 𝑩
𝑨 𝟏𝟎𝟎 𝟒
= = So, the numbers are 10 and 8.
𝑩 𝟕𝟓 𝟑
Required ratio = 4 : 3 28) Answer : C
Explanation :
22) Answer: Option A Amount paid to car owner = 90% of 85% of Rs. 3,25,000
Explanation:
𝟔
Rebate = 6% of Rs. 6650 = Rs. ( × 𝟔𝟔𝟓𝟎)=𝑅𝑠.399 = Rs. 2,48,625
𝟏𝟎𝟎
𝟏𝟎 Required difference = Rs. (325000 – 248625) = Rs. 76,375
Sales tax = 10% of Rs. (6650 - 399) = Rs. ( ×
𝟏𝟎𝟎
𝟔𝟐𝟓𝟏)=𝑅𝑠.625.10 29) Answer:
Explanation:
Final amount = Rs. (6251 + 625.10) = Rs. 6876.10
Let the number of applicants be x.Number of eligible
candidates = 95% of x.
23) Answer - B (2490 and 4150)
Eligible candidates of each other categories = 15% of (95% of
Explanation - Let the numbers be a and b.
x).
Then , 7.5 % of a =12.5% of b
=(15/100×95/100×x)
a = 125 x b/75 = 5 x b/3.
= 57/400×x.
Now, a - b =1660
Therefore, 57/400×x = 4275
5 x b/3 –b =1660
‹=› x =(4275×400 / 57)
2 x b/3= 1660
‹=› 30000.
b = (1660 x 3)/2) =2490.
One number = 2490, Second number =5 x b/3 =4150
30) Answer:
Explanation:
24) Answer - B (2%)
Total number of votes polled = (1136 + 7636 + 11628 ) =
Explanation - Required Percentage = (130/(6.5 x 1000)) x
20400.
100) % = 2%.
Required percentage = (11628 / 20400 ×100)% = 57%.
25) Answer - A (16800)
Explanation - Let the number of votes enrolled be a. Chapter – 3: Profit & Loss, Partnership
Then,Number of votes cast =75% of a. 1) Answer: : A , 85 : 9 = 105 : x
Valid votes = 98% of (75% of a). 75% of (98% of (75%of a)) Explanation:
=9261. x= (9×105/80)
(75/100) x (98/100) x (75/100)x a) =9261. = Rs.11.81
a = (9261 x 100x 100 x 100)/(75 x 98 x 75)) =16800 Hence, S.p per Kg = Rs.11.81

26) Answer : B 2) Answer: C


Explanation : Explanation:
A = 150% of B C.P= Rs.(100/122.50x392)
= Rs.(1000/1225×392)
= Rs.320.
Therefore, Profit = Rs.(392-320)
= Rs.72.

3) Answer:: D
Explanation:
Let C.P be Rs.x
Then, 2% of x =(400 - 380)
=20
x/50 = 20
Required percentage =
x=1000.

4) Answer: : A
Explanation :
27) Answer : B
Let C.P. of each mango be Re. 1.
Explanation :
𝟒 C.P. of 110 mangoes = Rs. 110; S.P. of 110 mangoes = Rs. 120
Let one number = x, Then, other number = 80% of x = 𝟏𝟎 𝟏
𝟓 Gain% = ×100%=9
𝟏𝟏𝟎 𝟏𝟏

5) Answer: : D
Explanation :
Suppose he buys 6 eggs of each kind

For details on our programs: - Contact: +91 9930202329, +91, 8291924300 or visit www.campuscredentials.com (78)
𝟏 𝟐
C.P. of 12 eggs = Rs. ×6+ ×6=𝑅𝑠. 7,
𝟐 𝟑
𝟑 15) Answer:: Option B
S.P. of 12 eggs = Rs. ×12=𝑅𝑠. 7.20, Explanation:
𝟓
𝟎.𝟐𝟎 𝟔
Gain = ×100%=2 % Let C.P. of each article be Re. 1 C.P. of x articles = Rs. x.
𝟕 𝟕
S.P. of x articles = Rs. 20.
6) Answer:: Option B Profit = Rs. (20 - x).
𝟐𝟎−𝒙
Explanation: ∴( 𝒙
× 𝟏𝟎𝟎 = 𝟐𝟓)
Let C.P. be Rs. x and S.P. be Rs. y. 2000 - 100x = 25x
Then, 3(y - x) = (2y - x) y = 2x. = 125x = 2000
Profit = Rs. (y - x) = Rs. (2x - x) = Rs. x. x = 16.
𝒙
∴𝑃𝑟𝑜𝑓𝑖𝑡 %= (𝒙 × 𝟏𝟎𝟎)%=100%
16) Answer:: Option D
7) Answer:: Option C Explanation:
Explanation: (C.P. of 17 balls) - (S.P. of 17 balls) = (C.P. of 5 balls)
𝟖𝟓 C.P. of 12 balls = S.P. of 17 balls = Rs.720.
S.P. = 85% of Rs. 1400 = Rs. ( × 𝟏𝟒0𝟎)=𝑅𝑠. 1190 𝟕𝟐𝟎
𝟏𝟎𝟎 C.P. of 1 ball = Rs.( )=𝑅𝑠. 60
𝟏𝟐

8) Answer: C
Explanation: 17) Answer:
S.P = 85% of Rs.1400 Explanation:
= Rs.(85/100×1400) = Rs.1190. Let the original price =Rs.100.
Then, C.P = Rs.90.
9) Answer: C S.P =130% of Rs. 90 = Rs.(130/100×90) = Rs.117.
Explanation: Required percentage = (117 - 100)% = 17%.
C.P = Rs.(80000+5000+1000) = Rs.86000
Profit= 25%. 18) Answer:
S.P = 12.5% of Rs. 86000 Explanation:
=Rs.(125/100×86000) =Rs.107500. S.P = Rs.27.50
Then Profit = 10%
10) Answer: B So, C.P = Rs.(100/110 x 27.50) = Rs.25.
Explanation: when S.P = Rs.25.75
Gain% =(0.70/70×100)% = 1%. Profit = Rs.(25.75 - 25) = Rs.0.75.
Profit % = (0.75/25×100)% =3%.
11) Answer:: Option A
Explanation: 19) Answer:: Option C
Let marked price = Rs. 100. Explanation:
Then, C.P. = Rs. 64. S.P. = Rs. 88. Let C.P. = Rs. 100.
𝟐𝟒 Then, S.P. = Rs. 120
∴ Gain % = (𝟔𝟒 × 𝟏𝟎𝟎)%=37.5% Let marked price be Rs. X.
Then, 90% of x = 120
12) Answer: - C (10%) =x=(
𝟏𝟐𝟎×𝟏𝟎𝟎
)
Explanation - Suppose he bought 2 kg, 4 kg and 3 kg of the 𝟗𝟎
𝟏
three varieties. ∴ Marked Price = 33𝟑% above C.P.
C.P. of 9 kg = Rs. (2 x 50 + 4 x 20 + 3 x 30) = Rs. 270
S.P. of 9 kg = Rs. (9 x 33) = Rs. 297 20) Answer:: Option C
𝟐𝟕
Profit% = ×100%=10% Explanation:
𝟐𝟕𝟎
Let C.P. of each clock be Rs. x.
13) Answer: - B (10%) Then, C.P. of 90 clock = Rs. 90x.
Explanation - (C.P. of 36 mangoes) - (S.P. of 36 mangoes) - ∴ (110% of 40x) + (120% of 50x) – (115% of 90x) = 40
Loss = (S.P. of 4 mangoes)  44x + 60x – 103.5x = 40
S.P. of 40 mangoes = C.P. of 36 mangoes  0.5x = 40
Let C.P. of each mango be Re. 1.  x = 80
C.P. of 40 mangoes = Rs. 40; S.P. of 40 mangoes = Rs. 36
𝟒 21) Answer: : D
Loss% = ×100%=10%
𝟒𝟎 Explanation :
Let the article be worth Rs. x.
14) Answer: - C (25%)
Explanation -
Let C.P. = Rs. 4x. Then, S.P. = Rs. 5x. Gain = Rs. (5x - 4x)
= Rs. X
𝒙
Gain% = ×100%=25%
𝟒𝒙

For details on our programs: - Contact: +91 9930202329, +91, 8291924300 or visit www.campuscredentials.com (79)
 100 + x = 140
 X = 40%

28) Answer: : C
Explanation :
C.P. for B = 120% of Rs. 400
22) Answer: : C
Explanation :
Let the C.P. be Rs. x. Then, 20% of x = 1100
C.P. for C = 110% of Rs. 480

= x = 5500

C.P. = Rs. 5500, Expenditure on repairs = 10%.


29) Answer: : B
Explanation :
Expenditure on repairs = Rs. (5500 – 5000) = Rs. 500 Let C.P. = Rs. 100. Then, Profit = Rs. 320, S.P. = Rs. 420
New C.P. = 125% of Rs. 100 = Rs. 125; New S.P. = Rs. 420
23) Answer: - A (18) Profit = Rs. (420 - 125) = Rs. 295
𝟐𝟗𝟓 𝟏𝟒𝟕𝟓
Explanation - Required percentage = ×100%= %=70%
𝟒𝟐𝟎 𝟐𝟏
Let S.P. of 45 lemons be Rs. x. Then, 80 : 40 = 120 : x or x =
𝟏𝟐𝟎×𝟒𝟎
=60 30) Answer: - B (16.30)
𝟖𝟎
For Rs. 60, lemons sold = 45. For Rs. 24, lemons sold = Explanation - C.P. of 50 kg wheat = Rs. (30 x 11.50 + 20 x
𝟒𝟓×𝟐𝟒 14.25)
=180
𝟔𝟎 = Rs. (345 + 285) = Rs. 630
𝟏𝟑𝟎
S.P. of 50 kg wheat = 130% of Rs. 630 = ×630=𝑅𝑠.819
24) Answer: - D (more than 20% profit) 𝟏𝟎𝟎
𝟖𝟏𝟗
Explanation - S.P. per kg = = Rs. 16.38 = Rs. 16.30.
𝟓𝟎
Let the article be worth Rs. x.
𝟗𝒙
C.P. = 90% of Rs. x. = Rs. 31) Answer -C) 1040
𝟏𝟎
S.P. = 110% of Rs, x = Rs
𝟏𝟏𝒙 Explanation :
𝟏𝟎 A B G
𝟏𝟏𝒙 𝟗𝒙 𝒙
Gain = − =𝑅𝑠. 6000×6 4000×12 8000×12
𝟏𝟎 𝟏𝟎 𝟓
𝒙 𝟏𝟎 𝟐 3 4 8
Gain = × ×100%=22 %>20%
𝟓 𝟗𝒙 𝟗
A:B:G = 3:4:8
Akash's share = 5200×(3/15) = 1040
25) Answer:: Option B
Explanation:
32)Answer -C) 10 months
Cost Price (C.P.) = Rs. (4700 + 800) = Rs. 5500.
Explanation :
Selling Price (S.P.) = Rs. 5800.
Lets profit x
Gain = (S.P.) - (C.P.) = Rs.(5800 - 5500) = Rs. 300.
Yokesh profit = 2x/3
Kiran profit = x – (2x/3) = x/3
K:Y =1:2
Lets capital C, Yokesh time = N
26) Answer:: Option A
Explanation:
Let C.P. be Rs. x. 33)Answer -B) 28:49:64
𝟏𝟗𝟐𝟎−𝒙 𝒙−𝟏𝟐𝟖𝟎 Explanation :
Then, ×100= ×100
𝒙 𝒙
10×a : 8×b ; 7×c = 5 : 7: 8
1920 - x = x - 1280
10a/8b = 5/7 => b = 7/5 a
2x = 3200
10a/7c = 5/8 => c = 16/7a
x = 1600
a:b:c = a:7/5a:16/7a = 28a:49a:64a => 28:49:64
Required S.P. = 125% of Rs. 1600
34)Answer -C) 55.55
Explanation :
X:Y:Z = 4:3:2
27) Answer:: Option D Z = 250×(2/9) = 55.55
Explanation:
Let the required gain percent be x%. 35) Answer – B)12:20:25
Then, (110% of 3000) + (100 + x)% of 3000 = 125% of 6000 Solution:
𝟏𝟏𝟎 (𝟏𝟎𝟎+𝒙) 𝟏𝟐𝟓 Investments: 2x,4x,5x
 ( × 𝟑𝟎𝟎𝟎)+[ × 𝟑𝟎𝟎𝟎]= ×6000
𝟏𝟎𝟎 𝟏𝟎𝟎 𝟏0𝟎
 30(𝟏𝟎𝟎 + 𝒙)=4200

For details on our programs: - Contact: +91 9930202329, +91, 8291924300 or visit www.campuscredentials.com (80)
A:B:C=2x*8+2x*4 : 4x*8+(4x/2)*4 : 5x*8+(5x/2)*4 3) Answer:: Option D
A:B:C= 12:20:25 Explanation:

36) Answer – B)Rs.18000


Solution:
Let C = x then B = x + 5000 and A = x + 5000 + 8000 = x +
13000.
x + x + 5000 + x + 13000 = 60000 <=> 3x = 42000 , x =
14000.
4) Answer: - C (Rs 62,500 ; Rs 37,500)
A : B : C = 27000 : 19000 : 14000 = 27 : 19 : 14
Explanation -
A’s share = Rs. (40000 * 27/60 ) = Rs.18,000
Let the sum invested at 9% be Rs. a and that invested at 11% be
Rs. (100000 - a).
37) Answer – B)Rs.7548.96
Solution:
Ratio = 75:40:42
A:B:C = 75x *4+ (123/100 *75x *8) : 40x *12 : 42x*12 = 173:
80 :84
B’s share = 80/337 * 31800 = Rs. 7548.96

38) Answer - B) Rs 18,000


Explanation:
Let B invests Rs x, then ratio of their profits 2a = (1100000 - 975000) = 125000
20000*12 : x*8 = 30,000 : x a = 62500
So 30,000/x = 5/3 Sum invested at 9% = Rs. 62500
Solve, x = 18,000 Sum invested ar 11% = Rs. (100000 - 62500) = Rs. 37500

39) Answer - C) Rs 5560 5) Answer: - C (640)


Explanation: Explanation -
8000*4 + 7600*8 : 9000*4 + 9400*8
116 : 139
Share of B = 139/(116+139) * 10200 = 5560

40) Answer - D) Rs 16,100


Explanation:
Their shares are 5x, 4x, 7x resp.
5x – 4x = 2300
6) Answer: - D (None of these)
x = 2300
Explanation -
So C got 7*2300
Difference in C.I. ans S.I. for 2 years = Rs. (696.30 - 660) = Rs.
36.30
S.I. for one year = Rs. 330.
Chapter 04: SI & CI S.I. on Rs. 330 for 1 year = Rs. 36.30

Practice set 1 Answers:


1) Answer:: Option A Rate =
Explanation :
Let the sum be Rs. x. Then, 7) Answer: - B (2028 and 1875)
𝟒 𝟔𝟕𝟔 𝟓𝟏 Explanation -
C.I. = [𝒙 (𝟏 +
𝟏𝟎𝟎
) 𝟐 − 𝒙]=(
𝟔𝟐𝟓
𝒙 − 𝒙)=
𝟔𝟐𝟓
𝑥
𝟒 𝟕
S.I. = (
𝒙×𝟒×𝟐
)=
𝟐𝒙 We have (A's present share) = (𝟏 + )
𝟏𝟎𝟎
𝟏𝟎𝟎 𝟐𝟓 𝟗
𝟓𝟏𝒙 𝟐𝒙 𝟒
− =1 = (B's present share) =(𝟏 + )
𝟔𝟐𝟓 𝟐𝟓 𝟏𝟎𝟎
𝟐
x = 625. 𝑨′𝒔 𝒑𝒓𝒆𝒔𝒆𝒏𝒕 𝒔𝒉𝒂𝒓𝒆
= (𝟏 +
𝟒
)
𝑩′𝒔 𝒑𝒓𝒆𝒔𝒆𝒏𝒕 𝒔𝒉𝒂𝒓𝒆 𝟏𝟎𝟎
𝟐𝟔 𝟐 𝟔𝟕𝟔
2) Answer:: Option D ( ) =
𝟐𝟓 𝟔𝟐𝟓
Explanation :
Dividing Rs. 3903 in the ratio of 676:625
𝟔𝟕𝟔
A's present share = (𝟔𝟐𝟓+𝟔𝟐𝟓) of Rs. 3903 = Rs. 2028
B's present share = Rs. 3903 - Rs. 2028 = Rs. 1875

8) Answer:
Effective rate = (106.09 - 100) % = 6.09% Explanation : A

For details on our programs: - Contact: +91 9930202329, +91, 8291924300 or visit www.campuscredentials.com (81)
First we calculate the SI for 10 years SI for 10 years is
=> (1000 x 5 x 10)/100 = Rs 500
Now new principal is
P = Rs. 1500
A = Rs. 2000
SI = Rs. 500
SI = (P x R xT)/100
T = (500 x 100)/ 1500 x 5 = 62/3 yr 15) Answer:
So, the total amount of Time is 10 + 62/3 yr = 162/3 yr Explanation : B
Let the amount borrowed by Amit is A
9) Answer: Then,
Explanation : B A (1+8/100)2- 1 – (A x 8×2)/100 =16
Let Sona invested for Y years = 0.1664A – 0.16A = 16
A sum will be double when interest is equal to principal = A = (16/0.0064) = Rs. 2500
9535 = (9535 x 4 x Y)/100
4Y= 100 16) Answer:
Y= 25 years Explanation : A
Let a be the amount lent by Sumit at 5% rate
10) Answer: - C (94.50) So therefore according to the question
Explanation - 81/2 % of a- 5% of a = 350
17a/200 – 5a/100 = 350
7a/200 =3500
a = 10000

17) Answer:
Explanation : A
= Rs. 94.50 Let the equal amount = A (each)
And the interest = r
11) Answer: - C (2000) Using the equation for compound interest:
Explanation - A(1+5/100)6 = A(1+r/100)3(1+5/100)2 = (1+r/100)(105 x
105) / 100 x 100 = (100 + r)
r = 10.25%

18) Answer:: Option C


Explanation :
= 2000

12) Answer: - B (992)


Explanation -
S.I = Rs. (920 – 800) = Rs. 120; P = Rs. 800, T = 3 yrs
= Rs. 8820

.New rate = (5 + 3) % = 8% 19) Answer:: Option B


Explanation :

New amount = Rs. (800 + 192) = Rs. 992

13) Answer:: Option C


20) Answer: - A (Rs 2950)
Explanation :
Explanation -
S.I. for 1 year = Rs. (854 - 815) = Rs. 39.
Here, I – Rs. 1770, R = 8% per annum,
S.I. for 3 years = Rs.(39 x 3) = Rs. 117. 𝟏𝟓
Principal = Rs. (815 - 117) = Rs. 698. T=
𝟐
𝑦𝑒𝑎𝑟𝑠
𝟏𝟎𝟎 𝒙 𝑰 𝟏𝟎𝟎 𝒙 𝟏𝟕𝟕𝟎
Principal (P) = = 𝟏𝟓 =𝑅𝑠.2950
𝑹𝒙𝑻 𝟖×
14) Answer:: Option C 𝟐

Explanation :
Let P = Rs. 100. Then, S.I. Rs. 60 and T = 6 years. 21) Answer: - A (20,000)
Explanation -
Here, P1 = Rs. 12000, R1 = 10%, P2 = ?, R2 = 20%, R = 14%
Therefore, using the formula
Now, P = Rs. 12000. T = 3 years and R = 10% p.a. 𝑷𝟏𝑹𝟏 + 𝑷𝟐𝑹𝟐
𝑅= 𝑷𝟏 + 𝑷𝟐

For details on our programs: - Contact: +91 9930202329, +91, 8291924300 or visit www.campuscredentials.com (82)
𝟏𝟐𝟎𝟎𝟎 𝒙 𝟏𝟎 + 𝑷𝟐 𝒙 𝟐𝟎 28) Answer:: Option A
We get, 14 =
𝟏𝟐𝟎𝟎𝟎 + 𝒑𝟐
Explanation :
or, P2 = Rs. 8000
Let the sum invested in Scheme A be Rs. x and that in Scheme B
Total amount invested = Rs.(12000 + 8000) = Rs. 20000
be Rs. (13900 - x).
22) Answer: - A (1)
Explanation -
Here 28x - 22x = 350800 - (13900 x 22)
P1 = Rs. 3000 6x = 45000
P2 = Rs. 2000 x = 7500.
(𝑫𝒊𝒇𝒇𝒆𝒓𝒆𝒏𝒄𝒆 𝒊𝒏 𝑷) 𝒙 𝑵 𝒙 𝑹 So, sum invested in Scheme B = Rs. (13900 - 7500) = Rs. 6400.
Difference in interest =
𝟏𝟎𝟎
𝟏𝟎𝟎𝟎 𝒙 𝟓 𝒙 𝑵
50 = = N = 1 year 29) Answer: - C (Rs. 14800)
𝟏𝟎𝟎
Explanation -
23) Answer: - A (100) We have, P = Rs.. 10000, R = 8% per annum, T = 6 years
𝑷×𝑹×𝑻 𝟏𝟎𝟎𝟎𝟎×𝟖×𝟔
Explanation - 𝐼= 𝟏𝟎𝟎
=
𝟏𝟎𝟎
Let the maximum marks be x. A = P + 1 = 10000 + 4800 = Rs. 14800
From the given statement pass percentage is 42% - 12% = This, Mr. Jumbo returned Rs. 14800 to the finance company
30%
By hypothesis, 30% of x – 20% of x = 10 (marks) 30) Answer: - C (10%)
i.e., 10% of x = 10 Explanation:
Therefore, x = 100 marks. We have, T = 2 years
Let the principal be Rs. X
24) Answer: 𝒙
Then, simple interest (I) = Rs
Explanation : B 𝟓
𝒙
Amount invested at 12% = Rs A 𝟏𝟎𝟎 × 𝑰 𝟏𝟎𝟎×𝟓
Rate of interest (R) = =
𝑷×𝑻 𝒙×𝟐
Amount invested at 10% = Rs B 𝟏𝟎𝟎
= 130 =(A x 12 x1 )/100 +(B x 10 x1 )/100 =10% 𝑝.𝑎.
𝟓×𝟐
= 13000 =12A + 10B……1
= 134 =(A x 10 x1 )/100 +(B x 12 x1 )/100 Practice set 2 Answers:
=13400 = 10A + 12B……..2 1. Answer: B
Solving equations 1 and 2 we get Explanation:
A = Rs 500 It will become (4)² times in 2  3 = 6 yr.
So the amount invested at the rate of 12% is Rs 500

25) Answer:
2. Answer: C
Explanation : D
Present value of money = v
3. Answer: B
Then (v x 12)/100 + v = 10028
Explanation:
= (0.12v x v) = 10028
Let the rate of interest be r%
= (v) = 10028/1.12
Now this amount will become after 3 months
{(10028/1.12) x 12 x 3}/(12 x 100) + 10028/1.12
= {(10028 x 3) / 1.12 x100} + 10028/1.12
4. Answer: B
= (10028/103)/ 112 = 9222.17 = Rs9200
Explanation:
26) Answer:
Explanation : D
Let each sum be Rs a, then
From the given condition
{(a x 4 ½ x7)/100} – {(a x 4 x7)/100} = 31.50
= 7a/100 x ½ = 63/2
= a = Rs900

27) Answer:: Option B


Explanation : 5. Answer: B
Explanation:

So, n = 4 years.

For details on our programs: - Contact: +91 9930202329, +91, 8291924300 or visit www.campuscredentials.com (83)
6. Answer: C
Explanation:
Let the sum be Rs. 100
14. Answer: C
After 20 years it becomes Rs. 200
Explanation:
 Interest = 200 – 100 = 100

7. Answer: A 15. Answer: A


Explanation: Explanation:
According to question, 10½% of P - 4% of P = 650  6.5% of It will become (5)² times in 2  4 = 8 yr
P = 650
16. Answer: C
Explanation:
Let the sum be Rs. x
8. Answer: B
Explanation:

9. Answer: B 17. Answer: B


Explanation: Explanation:
The second year (in terms of CI) is In the case R = 6% ; T = months or 2

CI = (16854 - 15000) = Rs. 1854


10. Answer: D
18. Answer: D
Explanation:
Explanation:

11. Answer: A
Explanation:
Compound interest = Rs. 424.36 – Rs. 400 = Rs. 24.36
First Year’s SI
CI – SI = Rs. 61.80 – Rs. 60 = Rs. 1.80 19. Answer: C Explanation:
Interest on Rs. 30 for 1 year = Re. 1.80
Interest on Rs. 100 for 1 year = Rs.

20. Answer: B
 rate = 6% Explanation:
From the question,

12. Answer: C
Explanation:

Hence, the required time is 12 years.


21. Answer: A
13. Answer: D Explanation:
Explanation: Let the sum be Rs. x, then

For details on our programs: - Contact: +91 9930202329, +91, 8291924300 or visit www.campuscredentials.com (84)
28. Answer: D
Explanation:
22. Answer: B First rate of interest =

New rate= 6 – 3 = 3%

 New Amount = 600 + 72 = Rs. 672


23. Answer: C
Explanation:
29. Answer: C
Explanation:
First rate of interest =

New rate= 11 + 4 = 15%


24. Answer: D
Explanation:
 New Amount = 500 + 225 = Rs. 725

30. Answer: D
Where , A = Amount, P = Principal, and r1, r2, r3, are the rates Explanation:
of interest for different years. Each Interest =
In the above case,

25. Answer: C Chapter 05 Averages


Explanation: Practice set 1 Answers
Let the sum be Rs x and the original rate be y% per annum. 1) Answer:: B
Then, new rate = (y+4)% per annum. Explanation:
Clearly, we have (3 + 11 + 7 + 9 + 15 + 13 + 8 + 19 + 17 +
21 + 14 + x/12) = 12
137 + x = 144
X = 144 – 137
26. Answer: B X=7
Explanation:
Let the deposit = Rs. 100 2) Answer: C
Interest for first 3 years = Rs. 12 Explanation:
Interest for next 2 years = Rs. 12 Age of the teacher = (37 x 15 – 36 x 14) years = 51 years.
Interest for the last 2 year = Rs. 16
Total interest = Rs. 40 3) Answer: : D
When interest is Rs. 40 deposited amount is Rs. 100 Explanation:
When interest is Rs. 1200, deposited amount = Age decreased = (5 x 3) years = 15 years
So, required difference = 15years.

4) Answer - D (39.8)
27. Answer: B Explanation -
Explanation: There are 5 prime numbers- 31, 37, 41, 43, 47
𝟑𝟏+𝟑𝟕+𝟒𝟏+𝟒𝟑+𝟒𝟕 𝟏𝟗𝟗
Amount for 3 yrs. = 5320 𝐴𝑣𝑒𝑟𝑎𝑔𝑒= = =39.8
𝟓 𝟓
Amount for 4½ yrs. = 5980
 So interest for 1½ yrs. = 660 5) Answer - B (56)
Explanation -
 Interest for 1 yrs = The total of 11 results = 11 x 50 = 550
 So interest for 3 yrs = 1320 The total of first 6 results = 6 x 49 = 294
P = A for 3yrs – I for 3yrs. = 5320 – 1320 = 4000 The total of last 6 results = 6 x 52 = 312
The sixth result is common to both:
Sixth result = 294 + 312 – 550 = 56

For details on our programs: - Contact: +91 9930202329, +91, 8291924300 or visit www.campuscredentials.com (85)
15) Answer: Option b
6) Answer B Explanation:
Explanation : Sum of the present ages of husband, wife and child =
𝟏𝟎+𝟏𝟓+𝟐𝟎+𝟐𝟓+𝟑𝟎 𝟏𝟎𝟎 (27*3+3*3) years = 90 years.
𝐴𝑣𝑒𝑟𝑎𝑔𝑒= 𝟓
=
𝟓
=20
Sum of the present ages of husband, wife and child =
(20*2+5*2) years = 90 years.
7) Answer : C
Husband’s present age = (90 – 50) years = 40 years
Explanation :
Let the initial number of persons be x. Then,
16) Answer: : B
16x + 20 x 15 -15.5 (x + 20)
Required average speed = (2xy / x + y)km/hr
0.5x = 10
= 2 x 84 x 56 /(84 + 56)
x = 20
= (2 x 84 x 85 /140)
= 67.2 km/hr.
8) Answer: Option A
Explanation:
17) Answer: : B
Let the number of girls be x. Then, number of boys = (600 – x).
𝟑 Clearly, we have X = (3y+3z/6) or 2x= y + z
Then, (𝟏𝟏 × 𝟔𝟎𝟎)=11𝑥+12(𝟔𝟎𝟎 − 𝒙)
𝟒
= x = 7200 – 7050 18) Answer - B (73.5)
= x = 150 Explanation -
𝟕(𝟏+𝟐+𝟑+⋯……..+𝟐𝟎) 𝟕×𝟐𝟎×𝟐𝟏
Average= = =73.5
𝟐𝟎 𝟐𝟎×𝟐
9) Answer: Option A
Explanation:
19) Answer: Option E
Let their prices be 3x, 5x, and 7x.
Explanation:
Then, 3x + 5x + 7x = (15000*3), or x = 3000
Let the number be x, y, and z.
Hence, cost of cheapest item = 3x = Rs. 9000

10) Answer: : A Then,


Explanation: = 15 or (x + y) – (y + z) = 30 or x – z = 30
Average of 20 numbers = 0
Sum of 20 numbers = (0 x 20) =0. 20) Answer – (D)
It is quite possible that 19 of these numbers may be positive and Explanation:
if there sum id a, then 20th number is (-a). Total weight of (36+44) Students = (36x40+44x35)kg =
2980kg
11) Answer: : D Therefore average weight of the whole class = (2980/80) kg
Explanation: Therefore average weight = 37.25kg
Clearly, to find the average,we ought to know the numbers of
boys , girls or students in the class, neither of which has been 21) Answer - B (37)
given. Explanation -
so the data provided is inadequate. Let the average after 16th innings be a, then total score after
17thinnings =
12) Answer – (A) 16a+85 = 17 (a+3)
Explanation: a = 85-51 = 34
Basic Formula: Average after 17 innings = a + 3 = 34 + 3 = 37
1, 2, 3 ..… n
𝒏+𝟏 22) Answer : A)
If n is odd, the formula is th term
𝟐 Explanation :
The five multiples of 3 is 3, 6, 9, 12, 15 Total quantity of petrol consumed in 3 years =
𝒏+𝟏 𝟓+𝟏
= = th term
𝟐 𝟐
𝟔
=> th term =3rd term
𝟐
Here 3rd term is 9
Total amount spent = Rs. (3 x 4000) = Rs. 12000
13) Answer - C (45)
Explanation -
Total ages of 30 boys = 14 x 30 = 420 years Average cost = Rs
Total age when class teacher is included = 15 x 31 = 465 years
Age of class teacher = 465 – 420 = 45 years 23) Answer: Option c
Explanation:
14) Answer : A) Let the third number be x. Then, second number = 2x. First
Explanation : number = 4x
𝟐𝟖𝟐−(𝟑.𝟐×𝟏𝟎) 𝟐𝟓𝟎
Required run rate = = =6.25
𝟒𝟎 𝟒𝟎

For details on our programs: - Contact: +91 9930202329, +91, 8291924300 or visit www.campuscredentials.com (86)
Or 4x = 24 or x = 6 Suppose the average expenditure was Rs. x. Then total
So, the numbers are 24, 12 and 6. expenditure = 40x.
When 8 more students join the mess, total expenditure = 40x +
24) Answer - (D) 48.
Explanation:
𝟐𝒙𝒚
Average speed = 𝑘𝑚/ℎ𝑟
𝒙+𝒚 Now, the average expenditure =
𝟐×𝟖𝟒×𝟓𝟔 𝟐×𝟖𝟒×𝟓𝟔
=
𝟖𝟒+𝟓𝟔
𝑘𝑚/ℎ𝑟 = 𝟖𝟒+𝟓𝟔
𝑘𝑚/ℎ𝑟= 67.2km/hr. Now,we have

25) Answer - B (33 1/3)


Explanation - Let the journey by a km. Then a/3 km at the
speed of 25 km/hr and a/4 km at 30 km/hr and the rest distance
( a- a/3 –a/4 ) = 5/12 x a at the speed of 50 km/hr.
Total time taken during the journey of a km
𝒂 𝒂 𝟓𝒂 𝟏𝟖𝒂
+ + =
𝟑×𝟐𝟓 𝟒×𝟔𝟎 𝟏𝟐×𝟓𝟎 𝟔𝟎𝟎
𝟑𝒂 𝒂 𝟏𝟎𝟎 𝟏
=
𝟏𝟎𝟎
= 𝟑𝒂 =
𝟑
=33 𝑘𝑚/ℎ𝑟
𝟑  40x + 48 = 48(x - 2)
𝟏𝟎𝟎
 40x + 48 = 48x – 96
26) Answer : A)  8x = 144
Explanation :  x = 18.
Present age of (A + B) = (18 x 2 + 3 x 2) years = 42 years. Thus the original expenditure of the mess = 40  18 = Rs. 720
Present age of (A + B +C) = (22 x 3) years = 66 years.
C's age = (66 - 42) years = 24 years. 3. Answer : C
Explanation :
27) Answer: Option A
Explanation:
𝟐𝟖𝟐− (𝟑.𝟐×𝟏𝟎) 𝟐𝟓𝟎
we have the required average =
Required run rate = = =6.25
𝟒𝟎 𝟒𝟎
4) Answer : A
28) Answer: : C Explanation :
Explanation: Let the ratio be k : 1. Then, k  17.9 + 1  16.8 = (k + 1)  17.4
Average Speed = (2xy/x +y ) km/hr  k×17.9 + 1×16.8 = (k + 1)17.4
= (2x50 x30/ 50+30)  (17.9 – 17.4)k = 17.4 – 16.8
= 37.5 km/hr.  .5k = .6

29) Answer - B (100)


Explanation -
Let the number of passed candidates be a
Then total marks =>120 x 35 = 39 a + (120 – a) x 15
4200 = 39 a + 1800 – 15 a
a = 100
5) Answer : C
Explanation :
30) Answer : C)
we have the required answer = 2 (7 - 1) = 12
Explanation :
Sum of odd numbers upto 100 = 1 + 3 + 5 + 7 + …… + 95 +
6) Answer : A
97 + 99.
Explanation :
= (1 + 99) + (3 + 97) + (5 + 95) + .... + upto 25 pairs.
= 100 + 100 + 100 + ....... (25 times) = 2500. Weight of the teacher = (45.4  35 – 45  34) kg = 59 kg.
𝟐𝟓𝟎𝟎
Average = =50 7) Answer : C
𝟓𝟎
Explanation :
Practice set Answers: 2 Total age increased = (7  2) years = 14 years
1) Answer : A Sum of ages of two new men = (22 + 26 + 14) years = 62
Explanation : years.
Let the required number of assistance = x Average age of 2 new member = 62/2 = 31 years.
 130x + 440×16 = 140(16 + x)
 130x + 7040 = 2240 + 140x 8) Answer : D
 4800 = 10x Explanation :
 x = 480 Total increase in weight of 8 Oranges (22  8) = 176 gm. This
increase in the weight is due to replacement of a orange with a
2) Answer : D new orange whose weight is 176 gm more than the orange
Explanation :

For details on our programs: - Contact: +91 9930202329, +91, 8291924300 or visit www.campuscredentials.com (87)
replaced. Therefore, weight of new orange = (124 + 176) = Explanation :
300 gm. Let L, M and N represent their respective monthly incomes.
Then, we have :
9) Answer : D L + M = (4040  2) = 8080 ….(i)
Explanation : M + N = (5240  2) = 10480 …(ii)
N + L = (4200  2) = 8400 …(iii)
Adding (i), (ii) and (iii), we get : 2(L+M+N) = 26960 or
L+M+N = 13480….(iv)
Subtracting (ii) from (iv), we get L = 3000.
10) Answer : A
Explanation :  L’s monthly income = Rs. 3000
Total weight of 39 students = 39  38 = 1482 kg
17) Answer : C
Total weight of 40 student = 40  37.5 = 1500 kg
Explanation :
 Weight of new student = (1500 - 1482)kg = 18 kg
Average of remaining numbers
11) Answer : D
Explanation :
Mon. + Tues. + Wed. = 51  3 = 153ºC …..(i)
Mon.+Wed.+ Thu. = 50  3 = 150º C ….(ii) 18) Answer : C
Thu. = 49ºC Mon. + Wed. = 150– 49 = 101ºC ….(iii) Explanation :
From Eqs. (i) and (iii), Tues. = 153 – 101 = 52ºC Total age of 28 students = 28  9 = 252 yr.
Total age of 29 persons including one teacher = 29  10 = 290
12) Answer : D yr.
Explanation :  Age of teacher = (290 - 252) = 38 yr.
Average of the first and the second numbers =
19) Answer : C
Explanation :
Total income of Mohan and Ram = 2  300 = Rs. 600
Average of the second and the third numbers =
Total income of Raghu and Jaya = 2  350 = Rs. 700
Average income of all the four persons =

20) Answer : D
 First – Third = 40 Explanation :
Age of the mother = (15  5 – 9  4) years = 39 years.
13) Answer : C
Explanation : 21) Answer : B
Total of 100 observations = 3000 Explanation :
Correct total of 100 observations = (3000 – 76 + 46) = 2970 Total weight increased = (6  3.5) = 21 kg.
 Correct average = 29.7 So the weight of new person = 59 + 21 = 80 kg.

14) Answer : D 22) Answer : B


Explanation : Explanation :
Total age of 5 members, 2 years ago = (15  5) years = 75 Total runs in 9 innings = 60  9 = 540
years Total runs in 10 inning = 540
Total age of 5 members now = (75 + 2  5) years = 85 years
Total age of 6 members now = (15  6) years = 90 years
Age of the baby = (90 -85) years = 5 years Average runs after 10th inning =

15) Answer : C 23) Answer : C


Explanation : Explanation :
Let A, B, C represent their respective weights. Then, we have : Weight of bag = Old average + Increase in average  Total no.
A + B + C = (35  3) = 105 …..(i) of objects
A + B = (30  2) = 60 ….(ii) = 7 + 0.07  41 = 9.87 gm
B + C = (33  2) = 66 ….(iii)
Adding (ii) and (iii), we get : A + 2B + C = 126 …(iv) 24) Answer : C
Subtracting (i) from (iv), we get : B = 21 Explanation :
B’s weight = 21 kg.

16) Answer : D

For details on our programs: - Contact: +91 9930202329, +91, 8291924300 or visit www.campuscredentials.com (88)
2) Answer - A (20:36:63)
Explanation -
a:b=5:9 and b:c=4:7
= (4x9/4): (7x9/4) = 9:63/4
25) Answer : D
a:b:c = 5:9:63/4 =20:36:63
Explanation :
Total ages of 28 boys = 28  15 = 420 years 3) Answer : D
Total ages when class teacher is included = 29  16 = 464 Explanation :
years.
 Age of class teacher = 464 – 420 = 44 years

26) Answer : B 4) Answer : B


Explanation : Explanation :
If the average is increased by 4 kg, then the sum of weights
increases by 5  4 = 20 kg.
And this increase in weight is due to the extra weight included x : y = 2 : 1.
due to the inclusion of new person.
 Weight of new person = 90 + 20 = 110 kg 5) Answer - B (32/7)
Explanation -
27) Answer : A x/y = 3/4
Explanation : (4x + 5y)/(5x + 2y) = (4( x/y) + 5)/(5 (x/y) -2)
Let the number of passed candidates be x. =(4(3/4) + 5)/(5(3/4) - 2)
Then total marks = 120  33 = 37x + (120 - x)  13 =(3 + 5)/(7/4) = 32/7
3960 = 37x + 1560 – 13x
 3960 – 1560 = 24x 6) Answer : B
 2400 = 24x Explanation :
 x = 100
 Number of passed candidate = 100

28. Answer : C Required ratio is 4 : 1


Explanation :
Let his average after 14th innings be x 7) Answer : C
So, 14x + 90 = 15(x + 4) Explanation :
 14x + 90 = 15x + 60 A : B = 2 : 3, B : C = 4 : 5 =
 30 = x

29) Answer : D
Explanation : and C : D = 6 : 7 =

A:B:C:D=2:3: = 16: 24: 30: 35

8) Answer – (C)
Explanation :
Let the shares of A, B, C and D be Rs. 5x, Rs. 2x, Rs. 4x and Rs.
30) Answer : B 3x respectively.
Explanation : Then, 4x−3x=1000
We have the required answer => x=1000.
B's share =Rs.2x=Rs.(2*1000)= Rs. 2000.

9) Answer - C (27)
Explanation -
Chapter 06: Ratio & Proportion Let the fourth proportional to 4, 9, 12 be a.
Practice set Answers 1: Then, 4 : 9 : : 12 : a
1) Answer : A 4 a =9 x 12
Explanation : a=(9 x 12)/14=27;
Let the third proportional to 0.36 and 0.48 be x Fourth proportional to 4, 9, 12 is 27
Then, 0.36 : 0.48 :: 0.48 : x
𝟎.𝟒𝟖×𝟎.𝟒𝟖 10) Answer : C
𝑥= 𝟎.𝟑𝟔
=0.64
Explanation :

For details on our programs: - Contact: +91 9930202329, +91, 8291924300 or visit www.campuscredentials.com (89)
18) Answer : C
Explanation :
= 19(4x2 - 3y2) = 12(2x2 + 5y2)
For 9 kg zinc, mixture melted = (9 + 11) kg.
= 52x2 = 117y2 𝟐𝟎
= 4x2 = 9y2 For 28.8 kg zinc, mixture melted = ×28.8 𝑘𝑔. =64 𝑘𝑔.
𝟗

19) Answer : B
= Explanation :
Let the ages of A and B be 3x years and x years respectively.

11) Answer : B 2x + 30 = 3x + 15 = x = 15
Explanation : So, A's age = (3 x 15) years = 45 years and B's age = 15 years

20) Answer : Option B


Explanation:
Let the fourth proportional to 5, 8, 15 be x.
Then, 5 : 8 : 15 : x
12) Answer - (B) 5x = (8 x 15)
Answer:: = x = (8 x 15)/5 =24
x*5=0.75*8
𝟔 21) Answer - A (10)
= > x= = 1.20
𝟓 Explanation -
Let the quantity of alcohol and water be 4x litres and 3x litres
13) Answer : Option B respectively
Explanation: 4x/(3x+5)=4/5
Let the numbers be 3x and 5x. 20x=4(3x+5)
8x=20
x=2.5
Quantity of alcohol = (4 x 2.5) litres = 10 litres
23(3x - 9) = 12(5x - 9)
22) Answer : D
9x = 99
Explanation :
x = 11.
G = 19W and C = 9W
The smaller number = (3 x 11) = 33.
Let 1 gm of gold be mixed with x gm of copper to get (1 + x) gm
of the alloy
14) Answer - A (420 and 252)
(1 gm gold) + (x gm copper) = (x + 1) gm of alloy
Explanation -
19W + 9Wx = (x + 1) x 15W
Sum of ratio terms = (5 + 3) = 8.
19 + 9x = 15(x + 1)
First part = Rs. (672 x (5/8)) = Rs. 420;
6x = 4
Second part = Rs. (672 x (3/8)) = Rs. 252
= x = 2/3
Ratio of gold with copper = 2/3 = 3:2
15) Answer : D
Explanation :
23) Answer : C
Let A = 2x, B = 3x and C = 4x. Then
Explanation :
For dividing 12 into two whole numbers, the sum of the tatio
terms must be a factor of 12.
So, they cannot be in the ratio 3 : 2.

24) Answer – (B)


16) Answer - (D) Explanation :
Explanation : Step (i) Let x be the number of boys and y be the number of
𝟏 𝟐 𝟑
Given ratio = : : =6:8:9 girls.
𝟐 𝟑 𝟒
𝟔 Given total number of boys and girls = 100
=> 1stpart=Rs.(782* )= Rs. 204
𝟐𝟑 x+y= 100-------------- (i)
Step (ii) A boy gets Rs. 3.60 and a girl gets Rs. 2.40
17) Answer - C (490 , 392 and 280) The amount given to 100 boys and girls = Rs. 312
Explanation - 3.60x + 2.40y = 312 -------------- (ii)
Sum of ratio terms = (35 + 28 + 20) = 83. Step (iii)
A's share = Rs. (1162 x (35/83))= Rs. 490; Solving (i) and (ii)
B's share = Rs. (1162 x(28/83))= Rs. 392; 3.60x + 3.60y = 360 ------- Multiply (i) by 3.60
C's share = Rs. (1162 x (20/83))= Rs. 280

For details on our programs: - Contact: +91 9930202329, +91, 8291924300 or visit www.campuscredentials.com (90)
=> 3.60x + 2.40y = 312 --------- (ii)
1.20y = 48
𝟒𝟖
y= = 40
𝟏.𝟐𝟎
= Number of girls = 40

25) Answer - B (360)


Explanation -
Let the number of 50 p, 25 P and 10 p coins be 5x, 9x and 4x
respectively.
(5x/2)+( 9x/ 4)+(4x/10)=206
50x + 45x + 8x = 4120
103 x = 4120
x=40.
Number of 50 p coins = (5 x 40) = 200;
Number of 25 p coins = (9 x 40) = 360;
Number of 10 p coins = (4 x 40) = 160.

26) Answer : A
Explanation :
Let the prices of a scooter and a T.V. set be Rs. 7x and Rs. 5x
respectively.
Then, 7x - 5x = 8000
2x = 8000
x = 4000
Price of a T.V. set = Rs. (5x x 4000) = Rs. 20000.
Practice set Answers 2:
27) Answer – (B) 1) Answer: A
Explanation : Explanation:
𝟒 𝟐
×𝐴= ×𝐵
𝟏𝟓 𝟓
𝟐 𝟏𝟓
=> A = ( × )𝐵
𝟓 𝟒
𝟑
=> A = 𝐵 2) Answer: D
𝟐
𝑨 𝟑 Explanation:
=> =
𝑩 𝟐
=> A:B=3:2.
𝟐 LCM of 5,10 and 15 is 30 So, (1/5: 1/10: 1/15) 30
B's share = 𝑅𝑠. 1210× = Rs. 484.
𝟓
= 6:3:2
28) Answer : C
Explanation : So, share of first is

3) Answer: D
Explanation:
Given A : B = 7: 3, B:C = 6: 10 = 3:5,

C:D = 15: 17 =
29) Answer - (C)
Explanation :
Originally, let the number of boys and girls in the college be 7x
4) Answer: B
and 8x respectively.
Explanation:
Their increased number is (120% of 7x) and (110% of 8x).
𝟏𝟐𝟎 𝟏𝟏𝟎
 ( × 𝟕𝒙)=( × 𝟖𝒙)
𝟏𝟎𝟎 𝟏𝟎𝟎
𝟒𝟐𝒙 𝟒𝟒𝒙
 𝑎𝑛𝑑 180AB2 + 5AC2 – 60ABC= 150A2 B + 6BC2 – 60ABC
𝟓 𝟓
𝟒𝟐𝒙 𝟒𝟒𝒙
 So, the required ratio = : 180AB2 – 150A2 B = 6BC2 – 5AC2
𝟓 𝟓
= 21 : 22 30AB(6B-5A) = C2 (6B-5A)
30AB = C2
30) Answer : C
Explanation : 5) Answer: D
Explanation:
The total number of students should be divisible by (6 + 7) = 13

For details on our programs: - Contact: +91 9930202329, +91, 8291924300 or visit www.campuscredentials.com (91)
Explanation:
6) Answer: A
Explanation:
Let the third proportional be x, then 4 : 16 = 16 : x  x = 64
14) Answer: C
7) Answer: B Explanation:
Explanation: Let the no. of coins be 5x, 6x and 8x respectively.
So, Money Value:

 Speed of dog : Speed of cat = 10:9


10x = 6500, x = 650, So the number of 25 paise in the bag is =
8) Answer: B 8x = 8  650 = 5200 Rs.
Explanation:
Let first and third part be 2x and 7x, then second part be 2x – 15) Answer: C
15 Explanation:
(2x) + (2x – 15) + 7x =150 Let Salary of Vijay = Rs. 1500
 11x = 150 +15 = 165
 x = 15
Therefore, three parts are 30, 15 and 105. Salary of Dhiraj =

9) Answer: C Salary of Sunil =


Explanation: Salary of Sneha = (Rs. (1500 + 1800 + 2100) = Rs. 5400
Let the number x be subtracted from each of the numbers,
16) Answer: D
Explanation:
For x = 4, this relation is correct. Let their ages 6 years ago be 2x, 3x, 5x and 7x.
Their ages now 2x + 6, 3x + 6, 5x + 6, 7x + 6.
10) Answer: B Or, 17x + 24 = 143
Explanation:  x=7
Present age of Aman = 3x+6= 21+6 = 27 years
Present age of Father = 7x+6= 49+6 = 55 years
Hence, required years (55 - 27) years = 28 years.

17) Answer: C
Explanation:
It cannot be determined because the total money to be
distributed is not given.

18) Answer: C
Explanation:
11) Answer: C If the numbers are 4a and 5a, then 20a= 260 -> a = 13
Explanation:  the first number is (4  13) = 52 and Second number is (5 
13) = 65

19) Answer: C
Explanation:

12) Answer: B
Explanation:
Let the weight of water melon be x kg. Numbers are 21 and 35.

20) Answer: D
Explanation:
Quantity of milk in 680 ml. mixture is

Lext x ml water be added :


Required ratio = 65 : 36 : 42.

13) Answer: B

For details on our programs: - Contact: +91 9930202329, +91, 8291924300 or visit www.campuscredentials.com (92)
21) Answer: C
Explanation:

So, The sum of their monthly incomes be Rs. 2800.

27) Answer: D
Explanation:
A : B = 6 : 3 =2 : 1 , B : C = 9 : 6 = 3 : 2
A: B 2:1

22) Answer: B
Explanation:

28) Answer: C
Explanation:

29) Answer: B
Explanation:
23) Answer: C
Explanation:
Let the no. of Book, pen and pencil in the Shop be 18x
Difference between the no of Pens & Pencils= 2x
Now, 2x to be multiple of 21
So, x has to be a multiple of 21.
So, total number of book, pen and pencil = 18x = 18  21 = 30) Answer: D
378 Explanation:

24) Answer: C
Explanation:

 102x – 204 = 63x + 108


 39x = 312
x=8
So, the quantity be 13x = 13  8 = 104 lit.
25) Answer: B
Explanation: Chapter 07: Mixture & Allegation
Practice set 1 Answers:
1) Answer::
Explanation :
24x = 1176 By the rule of alligation:
 x = 49 C.P. of 1 litre of water C.P. of 1 litre of milk
The Original prices of two horses was 18x and 25x so, 18 
49= Rs. 882, 25  49 = Rs.1225

26) Answer: C
Explanation:
Let the incomes be 6x and 8x so

Ratio of water to milk = 4 : 8 = 1 : 2

For details on our programs: - Contact: +91 9930202329, +91, 8291924300 or visit www.campuscredentials.com (93)
𝟗 𝟗 𝟗
(𝟒𝟎 × × × ) =29.16 𝑙𝑖𝑡𝑒𝑟𝑠.
𝟏𝟎 𝟏𝟎 𝟏𝟎
2) Answer:: :
Explanation :
7) Answer : C
S.P. of 1 kg of the mixture = Rs. 68.20, Gain = 10 %
Explanation :
C.P. of 1 kg of the mixture = Rs. (100 / 110 x 68.20) = Rs. 62.
Since first and second varieties are mixed in equal proportions,
By the rule of alligation:
so their average price
C.P. of 1 kg tea of 1st kind C.P. of 1 kg tea of 2nd kind

So, the mixture is formed by mixing two varieties, one at Rs.


130.50 per kg and the other at say, Rs. x per kg in the ratio 2 :
2, i.e., 1 : 1. We have to find x.
By the rule of alligation, we have :

Required ratio = 3 : 2

3) Answer: Option C
Explanation:
By the rule of alligation:

= x - 153 = 22.50
= x = 175.50
Hence, price of the third variety = Rs. 175.50 per kg.
Required rate = 3.50 : 1.50 = 7 : 3.
8) Answer:: :
4) Answer:: : Explanation :
Explanation : Let the C.P. of spirit be Re. 1 litre.
By the rule of alligation: Spirit in 1 litre mix. of A = 5/7 litre, C.P. of 1 litre mix. in A =
C.P. of 1 kg rice of 1st kind (in paise) C.P. of 1 kg rice of 2nd Re. 5/7
kind (in paise) Spirit in 1 litre mix. of B = 7/13 litre, C.P. of 1 litre mix. in B =
Re. 7/13
Spirit in 1 litre mix. of C = 8/13 litre, Mean price = Re. 8/13.
By the rule of alligation, we have:
Cost of 1 litre mixture in A Cost of 1 litre mixture in B

Required ratio = 80 : 70 = 8 : 7.

5) Answer:: :
Explanation :
Since first second varieties are mixed in equal proportions, so
their average price = Rs.(126+135/2) = Rs.130.50 Required ratio = 1/13 : 9/91 = 7:9.
So, the mixture is formed by mixing two varieties, one at Rs.
130.50 per kg and the other at say, Rs. x per kg in the ratio 2 : 9) Answer: Option C
2, i.e., 1 : 1. We have to find x. Explanation:
Cost of 1 kg tea of 1st kind Cost of 1 kg tea of 2nd kind By the rule of allegation, we have:

Ration of 1st and 2nd parts = 4 : 6 = 2 : 3


𝟑
Quantity of 2nd kind = ( × 𝟏𝟎𝟎𝟎)𝑘𝑔= 600 kg.
x-153/22.50 = 1 =› x - 153 = 22.50 =› x=175.50. 𝟓

Hence, price of the third variety = Rs.175.50 per kg.


10) Answer : C
6) Answer: Option D Explanation :
Explanation: Suppose the can initially contains 7x and 5x litres of mixtures A
𝟒 𝟑 and B respectively.
Amount of milk left after 3 operations = [𝟒𝟎 (𝟏 − ) ] litres
𝟒𝟎
Quantity of A in mixture left =

For details on our programs: - Contact: +91 9930202329, +91, 8291924300 or visit www.campuscredentials.com (94)
Quantity of B in mixture left =

Ratio of water to milk = 4 : 8 = 1 : 2

15) Answer:: :
Explanation :
Let the cost of 1 litre milk be Re. 1
Milk in 1 litre mix. in 1st can = 3/4 litre, C.P. of 1 litre mix. in
1st can Re. ¾
Milk in 1 litre mix. in 2nd can = 1/2 litre, C.P. of 1 litre mix. in
252x - 189 = 140x + 147 2nd can Re. ½
112x = 336 Milk in 1 litre of final mix. = 5/8 litre, mean price = Re. 5/8.
x=3 By the rule of alligation, we have:
So, the can container 21 litres of A. Cost of 1 kg mixture of 1st kind Cost of 1 kg mixture of 2nd
kind
11) Answer:: :
Explanation :
Suppose the can initially contains 7x and 5x litres of mixtures A
and B respectively
Quantity of A in mixture left
= (7x - 7/12 x 9) litres = (7x - 21/4) litres.
Quantity of B in mixture left
= (5x - 5/12 x 9) litres = (5x - 15/4) litres. Ratio of two mixtures = 1/8 : 1/8 = 1:1.
(7x - 21/4) / (5x - 15/4)+9 = 7/9 So, quantity of mixture taken from each can = (1/2 X 12) = 6
= 28x - 21/20x + 21 = 7/9 litres.
= 252x - 189 = 140x + 147
= 112x = 336 = x = 3. 16) Answer – (D)
So, the can contained 21 litres of A. Explanation :
C.P of a unit quantity of 1st kind = 310p (in paise)
12) Answer – (B) C.P of a unit quantity of 2nd kind = 360p
Explanation : Mean price = 325p
C.P of 1 Kg C.P of 1 Kg C.P of unit quantity C.P of unit quantity
rice of 1st rice of 2nd of 1st kind of 2nd kind
kind (930p) kind (1080p) (310p) (360p)

\ / \ /
Mean Price Mean Price
(1000p) (325p)
/ \ / \

(1080 – 1000) : (1000 - 930) (360 – 325) : (325 – 310)


80 70 35 15
Thus, required ratio = 80 : 70 = 8 : 7
Required ratio = 35 : 15 = 7 : 3
13) Answer: Option B They must be mixed in the ratio 7 : 3
Explanation: 17) Answer: Option A
By the rule of allegation: Explanation:
S.P. of 1 kg of the mixture = Rs. 68.20, Gain = 10%.
𝟏𝟎𝟎
C.P. of 1 kg of the mixture = Rs. ×68.20=𝑅𝑠.62.
𝟏𝟏𝟎
By the rule of allegation, we have:

Required ratio = 60 : 90 = 2 : 3.

14) Answer : A
Explanation :
Required ratio = 3 : 2.
By the rule of allegation:
18) Answer: Option C

For details on our programs: - Contact: +91 9930202329, +91, 8291924300 or visit www.campuscredentials.com (95)
Explanation:
Since first and second varieties are mixed in equal proportions.
𝟏𝟐𝟔+𝟏𝟑𝟓
So, their average price = Rs. ( )= 𝑅𝑠. 130.50
𝟐
So, the mixture is formed by mixing two varieties, one at Rs.
130.50 per kg and the other at say, Rs. x per kg in the ratio 2 :
2, i.e., 1 : 1. We have to find x. (20 - x) 2
By the rule of alligation, we have: =
(x - 15) 3
60 - 3x = 2x - 30
5x = 90
x = 18.

23) Answer: Option C


𝒙−𝟏𝟓𝟑
Explanation:
∴ 𝟐𝟐.𝟓𝟎 =1 Suppose the can initially contains 7x and 5x of mixtures A and
x - 153 = 22.50 B respectively.
x = 175.50 𝟕
Quantity of A in mixture left =(𝟕𝒙 − × 𝟗)𝑙𝑖𝑡𝑟𝑒𝑠 =(𝟕𝒙 −
𝟏𝟐
𝟐𝟏
20) Answer : D )𝑙𝑖𝑡𝑟𝑒𝑠
𝟒
Explanation : 𝟓
Quantity of B in mixture left =(𝟓𝒙 − × 𝟗)𝑙𝑖𝑡𝑟𝑒𝑠 =(𝟓𝒙 −
𝟏𝟐
Let the C.P. of sprit be Re. 1 per litre. 𝟏𝟓
𝟓 )𝑙𝑖𝑡𝑟𝑒𝑠
Spirit in 1 litre mix. of A = litre; C.P. of 1 litre mix. in A = Re. 𝟒
𝟕 𝟐𝟏
𝟓 (𝟕𝒙− ) 𝟕
𝟒
. ∴ 𝟏𝟓 =
𝟗
𝟕 (𝟓𝒙− )+𝟗
𝟕 𝟒
Spirit in 1 litre mix. of B = litre; C.P. of 1 litre mix. in B = Re. 𝟐𝟖𝒙 − 𝟐𝟏 𝟕
𝟏𝟑 =
𝟕 𝟐𝟎𝒙 + 𝟐𝟏 𝟗
𝟏𝟑 252x - 189 = 140x + 147
𝟖 𝟖 112x = 336
Spirit in 1 litre mix. of C = litre; Mean price = Re
𝟏𝟑 𝟏𝟑
x = 3.
By the rule of alligation, we have :
So, the can contained 21 litres of A.

24) Answer : A
Explanation :
By the rule of allegation:

𝟏 𝟗
Required ratio = : =7:9
𝟏𝟑 𝟗𝟏
Required rate = 750 : 250 = 3 : 1
21) Answer:: :
Explanation : 25) Answer:: :
By the rule of allegation: Explanation :
Profit of first part Profit of second part By the rule of allegation:
Cost of 1 kg tea of 1st kind Cost of 1 kg tea of
2nd kind

So, ratio of 1st and 2nd parts = 4 : 6 = 2 : 3.


Quantity of 2nd kind = (3/5 x 1000)kg = 600 kg.
Required ratio = 750 : 250 = 3 : 1
22) Answer: Option A
Explanation: 26) Answer: Option A
Let the price of the mixed variety be Rs. x per kg. Explanation:
Let C.P. of 1 litre milk be Re. 1.
By rule of allegation, we have:
50
S.P. of 1 litre of mixture = Re.1, Gain = %.
3
C.P. of 1 litre of mixture = 100 x3 x 1 =6

For details on our programs: - Contact: +91 9930202329, +91, 8291924300 or visit www.campuscredentials.com (96)
𝟖
Milk in 1 litre mix. in A = litre; C.P. of 1 litre mix. in A = Re.
350 7 𝟏𝟑
𝟖
.
𝟏𝟑
By the rule of allegation, we have: 𝟓 𝟓
Milk in 1 litre mix. of B = litre; C.P. of 1 litre mix. in B = Re.
𝟕 𝟕
𝟗𝟎𝟎 𝟏 𝟗
Milk in 1 litre of final mix. = × ×1= litre; Mean price =
𝟏𝟑 𝟏𝟎𝟎 𝟏𝟑
𝟗
Re
𝟏𝟑
By the rule of allegation, we have

16
Ratio of water and milk = : = 1 : 6.
77

27) Answer:: :
Explanation :
Let the quantity of the wine in the cask originally be x litres
then, quantity of wine left in cask after 4 operations
= x(1- 8/x)4 litres. 𝟐 𝟏
Therefore x(1- 8/x)4 / x = 16/81 Required ratio = : =2 : 7
𝟗𝟏 𝟏𝟑
=› (1- 8/x)4 = (2/3)2
=› (x - 8 / x) = 2/3 Chapter 08 : Time & Work
=› 3x - 24 = 2x Practice set 1 Answers:
=› x = 24. 1) Answer:: A
Explanation :
28) Answer – (C) A's 1 day's work = 1/18
Explanation : and B's 1 day's work = 1/9
C.P of a unit quantity of 1st kind = Rs. 3.20 (A+B)'s 1 day's work = (1/18+1/9) = 1/6
C.P of a unit quantity of 2nd kind = Rs. 2.90
Mean price = Rs.3.08 2) Answer:: Option C :
C.P of unit quantity C.P of unit quantity Explanation :
of 1st kind of 2nd kind When A runs 5 rounds, B runs 4 rounds (ratio of speeds)
(Rs. 3.20) (Rs. 2.90) A passes B each time A has run 5 rounds or 5 x 1/4 = 5/4km. =
1 1/4km
\ / 1 1/4km is contained in 4 km 3 times. Hence A passes B thrice.
Mean Price
(Rs.3.08) 3) Answer:: Option B
/ \ Explanation :
𝟏 𝟏 𝟏 𝟏 𝟕 𝟏
C's 1 day's work = − ( + )= − =
(3.08 – 2.90) : (3.20 – 3.08) 𝟑 𝟔 𝟖 𝟑 𝟐𝟒 𝟐𝟒
𝟏 𝟏 𝟏
0.18 0.12 A's wages : B's wages : C's wages = : : =4:3:1.
𝟔 𝟖 𝟐𝟒
Required ratio = 0.18 : 0.12 = 3 : 2 𝟏
C's share (for 3 days) = Rs. (𝟑 × × 𝟑𝟐𝟎𝟎)=𝑅𝑠.400
29) Answer: Option D 𝟐𝟒

Explanation:
S.P. of 1 kg of mixture = Rs. 9.24, Gain 10%. 4) Answer: : C
𝟏𝟎𝟎 Explanation :
C.P. of 1 kg of mixture = Rs. ×9.24== 𝑅𝑠. 8.40
𝟏𝟏𝟎
By the rule of allegation, we have:
B works for half
day only
So, A and B together will complete the work in 15 days.

5) Answer: - B (11 days)


st nd
Ratio of quantities of 1 and 2 kind = 14 : 6 = 7 : 3. Explanation :
Let x kg of sugar of 1st be mixed with 27 kg of 2nd kind. (A's 1 day's work) : (B's 1 day's work) = 7/4 : 1 = 7:4
Then, 7 : 3 = x : 27 Let A's and B's 1 day's work be 7x and 4x respectively.
𝑥=
𝟕×𝟐𝟕
= = 63 𝑘𝑔. Then, 7x + 4x = 1/7
𝟑 11x = 1/7
X = 1/77
30) Answer : A A's 1 day's work = (1/77) * 7 = 1/11
Explanation :
Let cost of 1 litre milk be Re. 1. 6) Answer:: B
Explanation :
(A+B)'s 1 day's work = 1/4

For details on our programs: - Contact: +91 9930202329, +91, 8291924300 or visit www.campuscredentials.com (97)
A's 1 day's work = 1/12 𝟏 𝟏 𝟑 𝟏
(A + B)'s 1 day's work = + = =
𝟔 𝟏𝟐 𝟏𝟐 𝟒
B's 1 day's work = (1/4 - 1/12) = 1/6.
So, A and B together can finish the work in 4 days.
Hence, B alone can complete the work in 6 days.
14) Answer:: C
7) Answer:: Option C
Explanation :
Explanation :
(A+B+C)'s 1 day's work = (1/24+1/6+1/12) = 7/24.
1
1 woman's 1 day's work = So, A,B and C together will complete the job in 24/7 = 3x3/7
70
days.
1
1 child's 1 day's work =
140
15) Answer:: Option C
(5 women + 10 children)'s day's work 5 10 1 1 1
+ = + = Explanation :
= 70 140 14 14 7 (A + B + C)'s 1 day's work = ¼
5 women and 10 children will complete the work in 7 days. A's 1 day's work= 1/16
B's 1 day's work = 1/12
8) Answer: : A 𝟏 𝟏 𝟏 𝟏 𝟕 𝟓
C's 1 day's work = −( + )= ( + )= .
Explanation : 𝟒 𝟏𝟔 𝟏𝟐 𝟒 𝟒𝟖 𝟒𝟖
𝟒𝟖 𝟑
So, C alone can do the work in =9 𝑑𝑎𝑦𝑠.
𝟓 𝟓

16) Answer: : C
Explanation :
𝟏 𝟐
B's 10 day's work = ×10= .
: 𝟏𝟓 𝟑
𝟐
Remaining work =1−
𝟑
9) Answer: - B (3 3/7 days) 𝟏
Now, work is done by A in 1 day.
Explanation : 𝟏𝟖
𝟏 𝟏
𝟏 𝟏 𝟏 𝟕 = work is done by A in 18× =6 𝑑𝑎𝑦𝑠.
(A + B + C)'s 1 day's work = + + = 𝟑 𝟑
𝟐𝟒 𝟔 𝟏𝟐 𝟐𝟒
𝟕 𝟑
So, A, B and C together will complete the job in =3 days. 17) Answer: - D (10 days)
𝟐𝟒 𝟕
Explanation :
10) Answer:: B 𝟏 𝟏 𝟕
(B + C)'s 1 day's work = + =
Explanation : 𝟗 𝟏𝟐 𝟑𝟔
𝟕 𝟕
A's 1 day's work = 1/8 Work done by B and C in 3 days = ×3=
𝟑𝟔 𝟏𝟐
B's 1 day's work = 1/10 𝟕 𝟓
Remaining work = 1− =
𝟏𝟐 𝟏𝟐
(A+B)'s 1 day's work = (1/8 + 1/10) = 9/40. 𝟏
Both A and B will finish the work in = 40 / 9 = 4 x 4/9 Now,
𝟐𝟒
𝑤𝑜𝑟𝑘 𝑖𝑠 𝑑𝑜𝑛𝑒 𝑏𝑦 𝐴 𝑖𝑛 1 𝑑𝑎𝑦
𝟓 𝟓
11) Answer:: Option B So, 𝑤𝑜𝑟𝑘 𝑖𝑠 𝑑𝑜𝑛𝑒 𝑏𝑦 𝐴 𝑖𝑛 24×𝟏𝟐=10 𝑑𝑎𝑦𝑠.
𝟏𝟐
Explanation :
18) Answer:: C
Explanation :
Number of pages typed by ronald in 1 hour = 32/6 = 16/3.
Number of pages typed by elan in 1 hour = 40/5 = 8.
Number of pages typed by both in 1 hour = (16/3+8) =40/3.
Time taken by both to type 110 pages = (110×3/40)hrs =
8×1/4hrs = 8 hrs 15 min.

12) Answer: : B 19) Answer:: Option D


Explanation : Explanation :
𝟏 𝟏 𝟑𝟏 𝟏 𝟐
(A + B)'s 1 day's work = + = . (A + B)'s 20 day's work = ( × 𝟐𝟎)=
𝟏𝟏 𝟐𝟎 𝟐𝟐𝟎 𝟑𝟎 𝟑
𝟏 𝟏 𝟔 𝟐 𝟏
= (A + C)'s 1 day's work = + = Remaining work = 1− =
𝟏𝟏 𝟓𝟓 𝟓𝟓 𝟑 3
𝟑𝟏 𝟔 𝟓𝟓 𝟏 𝟏
Work done in 2 days = + = = Now. 𝑤𝑜𝑟𝑘 𝑖𝑠 𝑑𝑜𝑛𝑒 𝑏𝑦 𝐴 𝑖𝑛 20 𝑑𝑎𝑦𝑠
𝟐𝟐𝟎 𝟓𝟓 𝟐𝟐𝟎 𝟒 𝟑
Now, ¼ work is done by A in 2 days Therefore, the whole work will be done by A in (20 x 3) = 60
Whole work will be done in (2 x 4) = 8 days. days.

13) Answer: - A (4 days) 20) Answer: : C


Explanation : Explanation :
𝟏 𝟏
Ratio of rates of working of A and B = 2 : 1. (A + B)'s 1 day's work = ; (B + C)'s 1 day's work = ; (A +
𝟏𝟐 𝟏𝟓
So, ratio of times taken = 1 : 2. 𝟏
𝟏 C)'s 1 day's work =
B's 1 day's work = . 𝟐𝟎
𝟏𝟐
𝟏
A's 1 day's work = ; (2 times of B's work)
𝟔

For details on our programs: - Contact: +91 9930202329, +91, 8291924300 or visit www.campuscredentials.com (98)
𝟏 𝟏 𝟏 𝟏𝟐 𝟏 𝟏 𝟏 𝟏𝟓 𝟏
Adding, we get: 2(A + B + C)’s 1 day’s work = + + = = 2(A + B + C)’s 1 day’s work = + + = =
𝟏𝟐 𝟏𝟓 𝟐𝟎 𝟔𝟎 𝟑𝟎 𝟐𝟒 𝟐𝟎 𝟏𝟐𝟎 𝟖
𝟏 𝟏
𝟓
(A + B + C)'s 1 day's work =
𝟏𝟔
𝟏 𝟏𝟎 𝟓
(A + B + C)'s 1 day's work = Work done by A, B and C in 10 days = =
𝟏𝟎 𝟏𝟔 𝟖
So, A, B and C together can complete the work in 10 days. 𝟓 𝟑
Remaining work =1− =
𝟖 𝟖
𝟏 𝟏 𝟏
21) Answer: - B (4 : 3) A's 1 day's work = − =
𝟏𝟔 𝟐𝟒 𝟒𝟖
𝟏
Explanation : Now, 𝑊𝑜𝑟𝑘 𝑤𝑖𝑙𝑙 𝑏𝑒 𝑑𝑜𝑛𝑒 𝑏𝑦 𝐴 𝑖𝑛 1 𝑑𝑎𝑦.
𝟒𝟖
(20 x 16) women can complete the work in 1 day. 𝟑 𝟑
So, W8ork is done by A in 48× =18 𝑑𝑎𝑦𝑠.
𝟖 𝟖
𝟏
1 woman's 1 day's work =
𝟑𝟐𝟎 27) Answer:: D
(16 x 15) men can complete the work in 1 day.
𝟏
Explanation :
1 woman's 1 day's work = A's 1 day's work = 1/15
𝟐𝟒𝟎
𝟏 𝟏 B's 1 day's work = 1/20
or, required ratio = :
𝟐𝟒𝟎 𝟑𝟐𝟎
𝟏 𝟏 (A+B)'s 1 day's work = (1/15 + 1/20) = 7/60.
= : =4:3 (𝑐𝑟𝑜𝑠𝑠 𝑚𝑢𝑙𝑡𝑖𝑝𝑙𝑖𝑒𝑑) (A+B)'s 4 day's work = (7/60x4) = 7/15.
𝟑 𝟒
Remaining Work = (1-7/15) = 8/15.
22) Answer:: Option A
Explanation : 28) Answer: – (C)
Let 1 man's 1 day's work = x and 1 boy's 1 day's work = y. Explanation :
Then, 6x + 8y = 1/10 and 26x + 48y = ½ Formula Used:
Solving these two equations, we get : x =1/100 and y =1/200 Subtraction of fraction
(15 men + 20 boy)'s 1 day's work = (15/100) + (20/200 = 1/4) 𝒂 𝒄 𝒂𝒅−𝒄𝒃
− =
𝒃 𝒅 𝒃𝒅
15 men and 20 boys can do the work in 4 days.
A and B finish one work with company = 35 days
𝟏
23) Answer: : B = (A + B)’s one day’s work =
𝟑𝟓
Explanation : A alone finish the same work = 60 days
𝟏
Suppose A, B and C take x/2, and x/3 hours respectively to = A’s one day’s work =
𝟔𝟎
finish the work.
= B’s one day’s work = (A + B)’s one day’s work - A’s one
day’s work
𝟏 𝟏
= − (take LCM)
So, B takes 6 hours to finish the work. 𝟑𝟓 𝟔𝟎
𝟏𝟐−𝟕 𝟓 𝟏
= = =
𝟒𝟐𝟎 𝟒𝟐𝟎 𝟖𝟒
24) Answer: - A (13 1/3 days) = B alone can complete the work in 84 days
Explanation :
𝟏 𝟏
Work done by X in 8 days = ×8= 29) Answer:: Option A
𝟒𝟎 𝟓
𝟏 𝟒 Explanation :
Remaining work =1- =
𝟓 𝟓 P can complete the work in (12 x 8) hrs. = 96 hrs.
𝟒
Now, work is done by Y in 16 days. Q can complete the work in (8 x 10) hrs. = 80 hrs.
𝟓
𝟓 P's1 hour's work = 1/96 and Q's 1 hour's work = 1/80
Whole work will be done by Y in 16× =20 𝑑𝑎𝑦𝑠 𝟏 𝟏 𝟏𝟏
𝟒
𝟏 𝟏 (P + Q)'s 1 hour's work = + =
𝟗𝟔 𝟖𝟎 𝟒𝟖𝟎
X's 1 day's work = , Y's 1 day's work= 𝟒𝟖𝟎
𝟒𝟎 𝟐𝟎
𝟏 𝟏 𝟑 So, both P and Q will finish the work in hrs
𝟏𝟏
(X + Y)'s 1 day's work = + = 𝟒𝟖𝟎 𝟔𝟎 𝟓
𝟒𝟎 𝟐𝟎 𝟒𝟎
𝟒𝟎 𝟏 Number of days of 8 hours each = × 𝑑𝑎𝑦𝑠=5 𝑑𝑎𝑦𝑠
𝟏𝟏 𝟏𝟏 𝟏𝟏
Hence, X and Y will together complete the work in =13
𝟑 𝟑
𝑑𝑎𝑦𝑠. 30) Answer: : D
Explanation :
25) Answer:: Option B Whole work is done by A in (3 x 4) = 12 days.
Explanation : Whole work is done by B in (4 x 6) = 24 days.
Let 1 man's 1 day's work = x and 1 woman's 1 day's work = y. A’s wages : B’s wages = A’s 1 day’s work : B’s 1 day’s work =
Then, 4x + 6y = 1/8 and 3x + 7y = 1/10
𝟏𝟏 𝟏
Solving the two equations, we get: x = ,y= .
𝟒𝟎𝟎 𝟒𝟎𝟎
𝟏
1 woman's 1 day's work .
𝟒𝟎𝟎 A's share = Rs. (2/3) x 180 = Rs. 120.
𝟏 𝟏
10 women's 1 day's work = ( × 𝟏𝟎)=
𝟒𝟎𝟎 𝟒𝟎
Hence, 10 women will complete the work in 40 days. Practice set 2 Answers:
1) Solution: The Correct answer is (B)
26) Answer: : A Explanation:
Explanation : Worker A completes the task in 8 days. So, in one day, he will
complete1/8 part of the task.

For details on our programs: - Contact: +91 9930202329, +91, 8291924300 or visit www.campuscredentials.com (99)
So, A's one day work = (A+B+C)'s one day work =
Therefore, C's one day work = (A+B+C)'s one day work
Similarly, B's one day work = (A+B)'s one day work

∴ (A+B)'s one day work = So, C's one day work =

of the task is completed in one day so both will complete the

whole task in days So, C will complete the work in 15 days.

2) Solution: The Correct answer is (C) 6) Solution: The Correct answer is (A)
Explanation: Explanation:
Vikas and Mohan together can complete the task in 6 days. So,
in one day, they will complete 1/6 part of the task. A's one day work =

Therefore, (Vikas + Mohan)'s one day work will be = B's one day work =

Similarly, Vikas's one day work = C's one day work =

Therefore, Mohan's one day work = (A+B+C)'s one day work =


Work done in three days will be the sum of A's two-day work
and (A+B+C)'s one day work.
In one day Mohan completes the 1/15part of the work so he will
complete the entire work in 15 days. A's two-day work =
Therefore, the work is done in three days =
3) Solution: The Correct answer is (A)
Explanation:
3/8 part of the job is done in 3 days.
A's one day work =
The entire job will be done by A in = 3 x
B's one day work =
7) Solution: The Correct answer is (A)
(A + B)'s one day work = Explanation:
5 men colour 50-meter long cloth in 5 days.

1 man will colour = 10-meter long cloth in five days.

A and B's four day work = So, in one day one man can colour = 2-meter long cloth.
Now, as per question 4 men are supposed to colour the 40-
meter long cloth.
Therefore, the remaining work =
One man has to colour the = 10 meter cloth
4) Solution: Solution: The Correct answer is (B)
For 2-meter long cloth one man takes 1 day
Explanation:
(Peter + Tom)'s one-day work = 1/16 So, to paint 10-meter long cloth he will take =
As per the question, Peter can finish twice as much work as As, 4 men are working together so 40 m long cloth will be
finished by Tom in a given duration of time. coloured in 5 days
Quicker method:
Therefore, of their one day's work will be completed by Peter Apply formula: M1D1W2 = M2D2W1
5*5*40 = 4*D2*50
and of their one day work will be completed by Tom.
D2=
So, Tom's one day work will be =
So, Tom will take 48 days to complete the task. 8) Solution: The Correct answer is (B)
Explanation:
5) Solution: The Correct answer is (A) 4 men can finish 4 times of work in four days.
Explanation: Therefore, one man can finish the one time of work in four
days.
A's one day work = So, 6 men will finish the six times of work in the same time (4
days)
B's one day work = Quicker method:

For details on our programs: - Contact: +91 9930202329, +91, 8291924300 or visit www.campuscredentials.com (100)
Apply formula: M1D1W2 = M2D2W1 Both work 8 hours a day so the number of days required to
Let the work be X
Work done by 4 men, W1 = 4X
Work done by 6 men, W2 = 6X complete the work = = 2 days
4*4*6X = 6*D2*4X
96X = D2 *24X 12) Solution: The Correct answer is (A)
D2= 4 days Explanation:
A's one hour work = 1/6
9) Solution: The Correct answer is (B) B's one hour work = 1/8
Explanation: (A +B)'s one hour work = 1/6 + 1/8 = 7/24
(A+B+C)'s one day work = 1/3
Therefore, C's one day work = 1/3 – 7/24 = 1/24
B is 50% more efficient than A so he will take less time to
The ratio of A, B and C's wages will be equal to the ratio of
do a piece of work.
work done by them in one day.
A's wages: B's wages: C's wages = 1/6 : 1/8 : 1/24
Therefore, the ratio of the time taken by A and B =
C's share of three days = 3 * C's share for one day
Let B takes X days to do the job.
= 3 ∗ (1/24 ∗ 2800) = 350

13) Solution: The Correct answer is (A)


3X =20 Explanation:
X = 6.6 days 5 men completed half of the work in 18 days so the entire work
will be completed in 36 days.
10) Solution: The Correct answer is (D) 5 men' one day work will be = 1/36
Explanation:

A's one day work = One man's one day work =


Two men drop out, so the three men have to complete the
A's ten-day work = remaining work.

So the remaining work would be =


Three men's one day work will be =

B's one day work =


part of the work is completed by three men in one day
Therefore, the remaining ½ part of the work will be completed
A and B's one day work =

of the job will be done by them in one day. in = 30 Days.

14) Solution: The Correct answer is (C)


So, the remaining job = 8 days
Explanation:
Therefore, the total number of days required to do the job
Using formula: M1D1W2 =M2D2W1
would be = 10 + 8 = 18 days
We have = 5∗9∗W2 = 3∗D2∗W1
11) Solution: The Correct answer is (C) W2 = W1 as the task is the same in both the cases, so the
Explanation: amount of work to be done would be the same.
A can complete the work in, 6 x 8 = 48 hour Therefore, we have 5 ∗ 9 = 3 ∗ D2
B can complete the same work in = 4 x 6 = 24 hours 45 = 3∗ D2
D2 = 45 = 15days
3
A's one hour work =
15) Solution: The Correct answer is (D)
B's one hour work = Explanation:
Let the original number of workers = X
X workers can complete the work in 30 days. And (X - 50)
(A +B)'s one hour work = complete the same task in 40 days.
Apply formula: M1D1W2 =M2D2W1
W1=W2 as the task is the same in both the cases.
They will complete Therefore, X * 30 = (X - 5) * 40
So, the entire work will be completed in 1/9 ∗ 144 30 X = 40X - 200
/9 = 144 hours 200 = 40X -30X
200 = 10 X
X = 200/10 = 20days

For details on our programs: - Contact: +91 9930202329, +91, 8291924300 or visit www.campuscredentials.com (101)
16) Solution: The Correct answer is (C) Sum of each man one day's work
Explanation: =1 + 1 + 1 = 5 = 1
Let the work will be completed in X days.
Apply formula: M1 D1 T1 W2 = M2 D2 T2 W1 60 60 120 120 24
W1=W2 Therefore, they will do the work in 24 days
Therefore, 40 * 15 * 4 = 30 * D2 * 5
2400 = 150 * D2 21) The correct answer is: D
D2 = 2400 = 16days Explanation:
150 ATQ,
The efficiency of A = B+C
17) Solution: The Correct answer is (D) (A+B) can finish the work in 10 days, and C can finish the work
Explanation: in 50 days.
Let the original number of men = X Now, A+B = 10 days
X men do the work in 50 days. C = 50 days
As per the question, X + 6 men can do the work in 40 days (50 - Note: Assume the total work = LCM of the given days
10). Take the LCM of days = LCM of (10 and 50) = 50
Apply formula: M1D1W2 = M2D2W1 Let the total work = 50
W1=W2 Note: One day work = (total work/ days)
Therefore, X * 50 = (X + 6) * 40 Now,
50 X = 40X + 240 C's one day work = 50/50 = 1
10X = 240 (A+B)'s one day work = 5
X = 240 = 24 men i.e., (A+B+C)'s one day work = 5+1=6
10 i.e., A+A = 6
Or, A's one day work efficiency = 3
18) Solution: The Correct answer is (C) A+B = 5, i.e., 3+B = 5
Explanation: Or, B's one day efficiency=2
A is twice as good as workman as B so he will take less time to Hence, B alone can work in 50/2 = 25 days
do a piece of work than B and the ratio of time taken by A and
B would be = 1:2 22) Solution: The correct answer is : B
So, if there is a difference of one day (2-1), B takes 2 days. Explanation:
As per question the difference in time taken by A and B is 40 ATQ,
days. A+B = 12 days
So, if the difference is of 40 days, B will take 2 * 40 = 80 days. B+C = 16 days
A takes 40 days less than B. So, A will take 40 days (80 - 40) to Note: Assume the total work = LCM of the given days
do the work. Take the LCM of days = LCM of (12 and 16) = 48
A's one day work = 1 Let the total work = 48
40 Note: One day work = (total work/ days)
B's one day work = 1 Now,
80 (A+B)'s one day work = 48/12 = 4 unit
(A + B)'s one day work = 1 + 1 = 120 = 3 (B+C)'s one day work = 48/16 = 3 unit
40 80 3200 80 As per the question:
So, working together they will do the work in 80 days A works for 5 days
3 B works for 7 days or (5 + 2) days, that means B works 5 days
with A and remaining 2 days with C.
19) Solution: The Correct answer is (A) C works 13 days or (2+11) days, that means C works 2 days
Explanation: with B and remaining 11 days alone.
As per question: That means total work done by (A+B) in 5 days
Work done by A in 1 day: work done by B in 1 day = 2:1 A+B = 5 days * 4 unit = 20 units
A and B working together can complete the work in 20 days. And, total work done by (B+C) = 2 days * 3 unit = 6 units
So, (A+ B)'s one day work = 1 So, A+B+C finish the 26 units of work.
20 Remaining work = 48-26 = 22 unit work
Therefore, A's one day work will be = 1 ∗ 2 = 2 = 1 And C completes the remaining work in 11 days.
20 3 60 30 i.e., C's one day's works = 22/11 = 2 units.
Therefore, A alone can finish the work in 30 days C alone can finish total work in [total work/ C's one day work]
= [48/2] = 24 days.
20) Solution: The Correct answer is (A)
Explanation: 23) Solution: The correct Answer is: C
3 men's one day work = 1 Explanation:
20 Note: Assume the total work = LCM of the given days
One man works for half day, so his one day's work = 1 = 1 Take the LCM of days = LCM of (10 and 15) = 30
60∗2 120 Let the total work = 30
Note: One day work = (total work/ days)

For details on our programs: - Contact: +91 9930202329, +91, 8291924300 or visit www.campuscredentials.com (102)
Now, 1) Answer: D
A's one day work = 30/10 = 3 unit Explanation :
B's one day work = 30/15 = 2 unit Total distance travelled = (50 x 2x1/2)+(70x1x1/2)
That means A and B works 5 unit works in 2 days = (125 + 105) miles
i.e., 2 days = 5 unit work = 230miles.
To complete the 30 work, multiply both sides with 6.
i.e., 2 days * 6 = 5 unit work * 6 2) Answer:
Or, 12 days = 30 works Explanation :
So, A and B require 12 days to finish the work. - B (11 : 9)
Explanation - In the same time, they cover 110 km and 90 km
24) Solution: The correct Answer is: D respectively.
Explanation: Ratio of their speeds = 110 : 90 = 11 : 9
Note: Assume the total work = LCM of the given days
Take the LCM of days = LCM of (10, 15, and 30) = 30 3) Answer: : C
Let the total work = 30 Explanation :
Note: One day work = (total work/ days) Ratio of speeds =
Now,
A's one day work = 30/10 = 3 unit
B's one day work = 30/15 = 2 unit
C's one day work = 30/30= 1 unit 4) Answer: : Option B
ATQ, A work continuously and B and C works alternatively Explanation:
i.e., (A+B)'s one day work = 3+2 = 5 unit
And (A+C)'s one day work = 3+1 = 4 unit
Or, 2 days work = 9 unit
To complete the 27 units work, multiply both sides with 3.
i.e., 2 days * 3 = 9 unit * 3
Or, 6 days = 27 units of work
Now, on the 7th day, the remaining work is done by A + B.
If (A+B)'s one day work = 5 unit work
i.e.,
5) Answer: : Option B
1 work = 1/5 days
Explanation:
Or, 3 work = 3/5 days
Due to stoppages, it covers 9 km less.
Hence, the work will be finished in 6+3/5 days or 6[3/5] days.

25) Solution: the correct Answer is : B


Explanation: 6) Answer: – (C)
Note: Assume the total work = LCM of the given days Explanation:
Take the LCM of days = LCM of (10, 15, and 30) = 30 Distance = Speed * time
𝟑 𝟏𝟏
Let the total work = 30 Here time = 2hr 45 min = 2 hr = hr
𝟒 𝟒
Note: One day work = (total work/ days) 𝟏𝟏
distance =4* =11 km
Now, 𝟒

A's one day work = 30/10 = 3 unit New Speed =16.5 kmph
𝑫 𝟏𝟏
B's one day work = 30/15 = 2 unit Therefore time = = = 40 min
𝑺 𝟏𝟔.𝟓
C's one day work = 30/30= 1 unit
Let A works for days = D 7) Answer: – (A)
Note: man * days = total work Explanation :
ATQ, (A+B+C) works 6 units work per day till D days, so their In same time, they cover 110km & 90 km respectively.
total work = 6 * D For the same time speed and distance is inversely proportional.
(B+C) works 3 unit work per day till 1 day, so their total work so ratio of their speed =110:90= 11: 9
=3*1=3
C works 1 unit work per day till 3 days, so his total work = 1 * 8) Answer: A
3=3 Explanation:
Or, 6*D + 3*1 + 1*3 = 30 (total work) Time taken to cover 600 km
6D = 30-6 = (600/100)hrs = 6 hrs.
D = 24/6 Number of stoppages = 600/75 – 1 = 7.
D= 4 Total Time of stoppages = (3 x 7)min = 21 min.
So, A works for 4 days, B works for 4+1=5 days, and C works Hence,total time taken = 6 hrs 21 min.
for 4+1+3= 8 days.
9) Answer:
Chapter 09: Speed, Time & Distance Explanation - C (11 am)
Suppose they meet x hrs after 8 a.m.

For details on our programs: - Contact: +91 9930202329, +91, 8291924300 or visit www.campuscredentials.com (103)
Then, (Distance moved by first in x hrs) + Distance moved by Let the length of journey be x km.
second in (x-1) hrs = 330
60x + 75(x – 1) = 330
x=3
8x - 7x = 70
So, they meet at (8 + 3), i.e. 11 a.m
x = 70 km
10) Answer: : C)
17) Answer: : Option B
Explanation :
Explanation:
Let the correct time to complete the journey be x min
Distance covered in (x + 11) min. at 40 kmph = Distance
covered in (x + 5) min. at 50 kmph

x = 19 min

11) Answer: : Option A


18) Answer: – (C)
Explanation:
Explanation:
Let the actual distance travelled be x km.
𝒙 𝒙+𝟐𝟎 Let the normal speed be 's' km/hr
Then, = Then new speed =(s+5) km/hr
𝟏𝟎 𝟏𝟒
14x = 10x + 200 𝟑𝟎𝟎 𝟑𝟎𝟎
−2=
4x = 200 𝑺 𝑺+𝟓
On solving this equation we get:
x = 50 km.
s = 25 km/hr
12) Answer: – (D)
19) Answer: B
Explanation:
Distance = (1100 x 11/5)feet
Due to stoppages, it covers 9 km less.
𝟗 = 2420 feet.
Time taken to cover 9 km = *60 min = 10 min.
𝟓𝟒
20) Answer:
13) Answer: – (B) Explanation - C (60 kmph)
Explanation: Explanation - Number of gaps between 21 telephone posts =
A ----------------- C ---------------B 20 Distance traveled in 1 minute = (50 x 20) m = 1000 m = 1
7am ------------------------------ 8am km Speed = 60 km/hr
AC =20 km, CB =90 km
Distance travelled in 1 hour =20 km 21) Answer: : B)
Remaining distance =110−20=90 km Explanation :
𝑫𝒊𝒔𝒕𝒂𝒏𝒄𝒆
Time taken= Total distance travelled = (10 + 12) km/hr = 22 km/hr
𝑹𝒆𝒍𝒂𝒕𝒊𝒗𝒆 𝒔𝒑𝒆𝒆𝒅
𝟗𝟎
= = 2 hr
𝟐𝟎+𝟐𝟓
So, time = 8 am + 2 = 10 am

14) Answer: C
Explanation : 22) Answer: : Option C
Total distance travelled in 12 hours = (35 + 37 + 39 +..... upto Explanation:
12 terms) Let speed of the car be x kmph.
This is an A.P with first term, a = 35, number of terms, n = 12,
d = 2.
Required distance = 12/22 x 35 + {12 - 1) x 2)
= 6(70 + 23)
= 552 kms.

15) Answer:
Explanation - C (7)
Explanation - Relative speed = (2 + 3) = 5 rounds per hour 23) Answer: – (B)
So, they cross each other 5 times in an hour and 2 times in half Explanation:
an hour Hence, they cross each other 7 times before 9.30 a.m. Let total distance be S
total time = 1 hr 24 min = 84 min
16) Answer: : C) 𝟖𝟒 𝟐𝟏
=
𝟔𝟎
ℎ𝑟=𝟏𝟓ℎ𝑟
Explanation :
Let Vikas travels from A->T->S
A ------------------------ T ------------ S
Difference between timings = 15 min = 𝟐 𝟏
<---------- S ------------><---- 𝑆---->
𝟑 𝟑

For details on our programs: - Contact: +91 9930202329, +91, 8291924300 or visit www.campuscredentials.com (104)
A to T :: speed = 4 kmph Speed of the train = 16.5 m/sec = 16.5 * (18/5) km/hr = 59.4
𝟐 km/hr
Distance = ×𝑆
𝟑
T to S :: speed = 5 km
𝟐 𝟏 30) Answer: : Option D
Distance =1− ) 𝑆 = 𝑆 Explanation:
𝟑 𝟑
𝟐 𝟏
𝟐𝟏 𝑺 𝑺 Distance = (240 x 5) = 1200 km.
Total time: ℎ𝑟𝟑𝟒 +𝟑𝟓
𝟏𝟓 Speed = Distance/Time
84=10S+4S {Multiply both sides by 15*4} 𝟐
𝟖𝟒
Speed = 1200/(5/3) km/hr. We can write 1 hours as 5/3
𝟑
S=
𝟏𝟒 hours)
= 6 km Required speed = (1200 x 3) km/hr = 720 km/hr

24) Answer: B 31)Answer: C) 127.5 meters


Explanation : Explanation:
Speed =(600 / 5 x 60)m/sec When A runs 1000 meters, B runs 900 meters and when B runs
= 2 m/sec 800 meters, C runs 700 meters.
=(2 x 18/5)km/hr Therefore, when B runs 900 meters, the distance that C runs =
= 7.2 km/hr. (900 x 700)/800 = 6300/8 = 787.5 meters.
So, in a race of 1000 meters, A beats C by (1000 - 787.5) =
25) Answer: : C) 212.5 meters to C.
Explanation : So, in a race of 600 meters, the number of meters by Which A
Let the distance traveled on foot be x km beats C = (600 x 212.5)/1000 = 127.5 meters.
Then, distance traveled on bicycle = (61 – x) km
32) Answer: A) 200 m
Explanation:
9x + 4 (61 - x) = 9 x 36 Ratio of the speeds of A and B =5 : 1 = 5 :3
= 5x = 80 Thus, in race of 5 m, A gains 2 m over B.
= x = 16 km 2 m are gained by A in a race of 5 m.
80 m will be gained by A in race of 5/2*80 m =200m
26) Answer: : Option D => Winning post is 200 m away from the starting point.
Explanation:
Let the speed of two trains be 7x and 8x km/hr 33) Answer: B) 111.12 meters
𝟒𝟎𝟎 Explanation:
𝑇ℎ𝑒𝑛, 8𝑥= ( 𝟒
)=100
𝟏𝟎𝟎
A runs 1000 meters while B runs 900 meters and C runs 800
→𝑥=( )=12.5 meters.
𝟖
Speed of first train = (7 x 12.5) km/hr = 87.5 km/hr Therefore, B runs 900 meters while C runs 800 meters.
So, the number of meters that C runs when B runs 1000 meters
27) Answer: – (B) = (1000 x 800)/900 = 8000/9 = 888.88 meters
Explanation: Thus, B can give C (1000 - 888.88) = 111.12 meters start
𝟓𝟎 𝟓
Time = hr = hr
𝟔𝟎 𝟔 34) Answer: C) 33sec
Speed = 48mph
𝟓
Explanation:
distance =S*T=48*( )=40 km B runs 35 m in 7 sec.
𝟔
𝟒𝟎 𝟐 B covers 200 m in (7/35*200) = 40 sec.
time = hr = hr
𝟔𝟎 𝟑
𝟐 B's time over the course = 40 sec.
New speed =40*( ) kmph = 60kmph A's time over the course (40 - 7) sec = 33 sec.
𝟑

28) Answer: – (C) 35) Answer :- c)20m


Explanation: Explanation:
Let the speed of the current be x km/hr As the speed of A and B r given in ratio 3:4 consider speed
Thus upward speed =(4.5+x) km/hr of A as 3x and that of B as 4x
and downward speed =(4.5−x) km/hr A is already ahead by 140m so he has to cover a distance of
Let distance travelled be y, then 500–140=360m … time required by A to complete this distance
𝒚 𝟐𝒚
= => x= 1.5 km/hr is 360/3x {time=distance/speed} at that same time B will travel
(𝟒.𝟓−𝒙) (𝟒.𝟓+𝒙)
a distance= 4x *360/3x= 480 m
As total distance for B is 500m A wins the race by 500 – 480
29) Answer: : C)
=20 m
Explanation :
Let the speed of the train be x m/sec. Then,
36) Answer - a. 240, 60 Secs
Distance travelled by the train in 10 min. = Distance travelled
Explanation:
by sound in 30 sec
Length of the track L=1200m
x x 10 x 60 = 330 x 30
Speed of A = 27×51827×518 = 7.5 m/s
x = 16.5

For details on our programs: - Contact: +91 9930202329, +91, 8291924300 or visit www.campuscredentials.com (105)
Speed of B = 45×51845×518 = 12.5 m/s
(i) same direction Chapter 10: Pipe & Cistern
Time = L / Relative Speed = 120012.5−7.5120012.5−7.5 = 240 1) Answer : D
sec Explanation :
(ii) Opposite Direction 𝟏 𝟏 𝟏
Part filled by (A+B) in 1 hour = + =
Time = L / Relative Speed = 120012.5+7.5120012.5+7.5 = 60 𝟓 𝟐𝟎 𝟒
sec So, A and B together can fill the tank in 4 hours.
𝟏 𝟐 𝟏
Work done by the leak in 1 hour = − = .
𝟒 𝟗 𝟑𝟔
37) Answer - b. 480, 480 Secs Leak will empty the tank in 36 hrs.
Explanation:
Length of the track L = 1200 m 2) Answer : C
Speed of A = 18×51818×518 = 5 m/s Explanation :
Speed of B = 27×51827×518 = 7.5 m/s 𝒙
Let capacity of P be x liters. Then, capacity of Q = 𝑙𝑖𝑡𝑒𝑟𝑠
Time taken by A to complete one round = 1200512005= 240 𝟑
Capacity of the drum = 60x liters.
sec 𝟔𝟎𝒙
Time take by B to complete one round = 12007.512007.5 = 160 Required number of turns = 𝒙
𝒙+
𝟑
sec 𝟑
= 60𝑥× =45
(i) Same direction: 𝟒𝒙
They will meet at the starting point at a time which is the LCM
of the timings taken by each of them to complete one full round. 3) Answer : A
i.e., the LCM of 160s and 240 s which is 480 Sec Explanation :
(ii) Opposite Direction: Let the filling capacity of the pump be x m3 / min.
They will meet at the starting point at a time which is the LCM Then, empting capacity of the pump = (x + 10) m3 / min
𝟐𝟒𝟎𝟎 𝟐𝟒𝟎𝟎
of the timings taken by each of them to complete one full So, −(𝒙+𝟏𝟎)=8
𝒙
round. i.e., the LCM of 160s and 240s which is 480 Sec x2 + 10x - 3000 = 0
(x - 50) (x + 60) = 0
38) Answer - b. 480 sec x = 50.
Explanation: neglecting the –ve value of x
L = 1200 m
Speed of A (a) = 9 x 5/18 = 2.5 m/sec 4) Answer : B
Speed of B (b) = 18 x 5/18 = 5 m/sec Explanation :
Speed of C (c) = 27 x 5/18 = 7.5 m/sec Work done by the third pipe in 1 min.
They will meet for the first time at a time which is the L/a−b, 𝟏 𝟏 𝟏
= − +
L/b−c seconds 𝟓𝟎 𝟔𝟎 𝟕𝟓
𝟏 𝟑 𝟏
L/a−b = 1200/5−2.5 = 480 sec = − =− − 𝑣𝑒 𝑆𝑖𝑔𝑛 𝑚𝑒𝑎𝑛𝑠 𝑒𝑚𝑝𝑡𝑦𝑖𝑛𝑔
𝟓𝟎 𝟏𝟎𝟎 𝟏𝟎𝟎
L/b−c = 1200/7.5−5 = 480 sec The third pipe alone can empty the cistern in 100 min.
So they will meet for the first time after 480 sec. i.e., 8 min after
they start. 5) Answer : C
Explanation :
39) Answer - b. 480 sec 𝟏 𝟏 𝟗 𝟑
(A + B)'s 1 hour's work = + = =
Explanation : 𝟏𝟐 𝟏𝟓 𝟔𝟎 𝟐𝟎
𝟏 𝟏 𝟖 𝟐
L = 1200 m (A + C)'s 1 hour's work = + = =
𝟏𝟐 𝟐𝟎 𝟔𝟎 𝟏𝟓
Speed of P(p) = 9 x 5/18 = 2.5 m/sec 𝟑 𝟐 𝟏𝟕
Part filled in 2 hrs = + = ;
Speed of Q (q) = 18 x 5/18 = 5 m/sec 𝟐𝟎 𝟏𝟓 𝟔𝟎
𝟏𝟕 𝟏𝟕
Speed of R (r) = 27 x 5/18 = 7.5 m/sec Part filled in 6 hrs = 3× =
𝟔𝟎 𝟐𝟎
They will meet for the first time at a time which is the LCM of 𝟏𝟕 𝟑
Remaining part = 1− =
𝟐𝟎 𝟐𝟎
L/p, L/a, L/r 𝟑
L/p = 1200 / 2.5 = 480 Now, it is the turn of A and B and Part is filled by A and B in
𝟐𝟎
L/q = 1200 / 5 = 240 1 hour.
L/r = 1200 / 7.5 = 160 Total time taken to fill the tank = (6 + 1) hrs = 7 hrs
LCM of 480, 240, 160 is 480 Sec. So they meet after 8 min.
6) Answer:
40) Answer - d. None of these Explanation :
Explanation: part filled by (A+B+C) in 1 hour = (1/5 + 1/6 + 1/30) = 1/3.
When B has completed 10 rounds, A would have completed 10 x All the three pipes together will fill the tank in 3 hours
15/6 = 25 rounds.
When running in same direction, this would mean A having run 7) Answer:
15 rounds more than B and would thus have met 15 times (For Explanation :
every one round that A runs more than B, A meets B) Net aprt filled in 1 hour = (1/10+1/12-1/20)
When running in Opposite direction, this would mean A and B = 8/60
together having run 35 rounds and thus would have met 35 = 2/15.
times. Therefore the tank will be full in 15/2 hours = 7 hrs 30 min.

For details on our programs: - Contact: +91 9930202329, +91, 8291924300 or visit www.campuscredentials.com (106)
Let original length = l, breadth = b, so area = lb
8) Answer: When l and b increased by 1:
Explanation : (l+1)(b+1) = lb + 21
Work done by the inlet in 1 hour = (1/8 – 1/12) = 1/24. Solve, l + b = 20
Work done by the inlet in 1 min. = (1/24 × 1/60) = 1/1440. When l increased by 1, b decreased by 1:
Volume of 1/1440 part = 6 litres. (l+1)(b-1) = lb – 5
Therefore, Volume of whole = (1440×6) = 8640 litres Solve, l – b = 6
Now solve both equations, l = 13, b = 7
9) Answer: Perimeter = 2(13+7)
Explanation :
Capacity of the tank = (12×13.5) litres = 162 litres. 2) Answer - d) Rs 3440
Capacity of each bucket = 9 litres. Explanation:
Number of buckets needed = (162/9) = 18. Given 2(l+b) = 340
1 m broad boundary means increase in l and b by 2 m
10) Answer: So area of the boundary will be [(l+2)(b+2) – lb] = 2(l+b) + 4
Explanation : = 340 + 4 = 344
Net part filled in 1 hour = (1/4 – 1/9) = 5/36. So cost of gardening = 344*10
Therefore,the cistern will be filled in 36/5 hrs i.e, 7.2 hrs.
3) Answer - e) 72 cm
11) Answer: Explanation:
Explanation : Sides 3x, 4x, 5x
Let the slower pipe alone fill the tank in x minutes. So semi-perimeter, s = (3x+4x+5x)/2 = 6x
Then, faster pipes will fill it in x/3 minutes. Area = √s(s-a)(s-b)(s-c)
Therefore, 1/x + 3/x = 1/36 = √6x*3x*2x*x = 6x2 cm2
= 4/x = 1/36 So 6x2 = 216, this gives x = 6
= x = 144 min. Perimeter = 12x = 12*6

12) Answer: 4) Answer - d) 25% decrease


Explanation : Explanation:
A's work in 1 hour = 1/6. Area of triangle = (1/2) * base * height
B's work in 1 hour = 1/4. So effect on area = +50 + (-50) + (50)(-50)/100 = -25%
(A+B)'s 2 hour's work when opened alternately = (1/6 + 1/4) =
5/12. 5) Answer - b) 72 cm
(A+B)'s 4 hour's work when opened alternately = 10/12 = 5/6. Explanation:
Remaining part = (1 – 5/6) = 1/6. Length of wire = 2ᴨr = 264 which should be equal to the
Therefore, total tank to fill the tank (4+1) hrs = 5 hrs. perimeter of rectangle in which it is bent.
So 2(6x + 5x) = 264
13) Answer: Option B Solve, x= 12
Explanation: Largest side = 6x = 6*12
𝟏 𝟏
Part filled by (A + B + C) in 3 minutes =3 ( + +
𝟑𝟎 𝟐𝟎
𝟏 𝟏𝟏 𝟏𝟏
6) Answer - a) 100 cm
)=(𝟑 × )= Explanation:
𝟏𝟎 𝟔𝟎 𝟐𝟎
𝟑 Sides = x and 5x
Part filled by C in 3 minutes = .
𝟏𝟎
𝟑 𝟐𝟎 𝟔
Now vol. of rectangle = vol. of cube
Required ratio = ( × )= x * 5x * (0.5) = 10*10 *10
𝟏𝟎 𝟏𝟏 𝟏𝟏
Solve, x = 20
14) Answer: Option C Largest side = 5x = 5*20
Explanation:
7) Answer - b) 1538.5 cm3
Explanation:
Given 2ᴨrh + ᴨr2 = 616 and r = (1/2) * h
So 2ᴨ × (1/2)h × h + ᴨ × (1/4)h2 = 616
Solve, h = 28/√5
15) Answer: Option D Volume = ᴨr2h = (22/7) * (1/4) * h2 * h
Explanation: Put h = 28/√5, vol. ≈ 1538.5
𝟏 𝟑 𝟏
Work done by the leak in 1 hour =( − )=
𝟐 𝟕 𝟏𝟒 8)Answer - c) 17 : 9
Leak will empty the tank in 14 hrs Explanation:
Let radius = 5x and height = 12x
Chapter 11 - Mensuration Then slant height = √[(5x)2 + (12x)2]= 13x
1)Answer - c) 40 m Required ratio = 2ᴨr(h+r) : ᴨr(l+r)
Explanation:

For details on our programs: - Contact: +91 9930202329, +91, 8291924300 or visit www.campuscredentials.com (107)
9) Answer - d) 2376 cm2 17) Answer - c) 168 cm2
Explanation:
Slant height of cone, l = √(122 + 52) = 13 cm
Total surface area of final figure = curved surface area of cone
+ curved surface area of cylinder + area of base
= ᴨrl + 2ᴨrh + ᴨr2
= ᴨr (l + 2h + r) Explanation:
= (22/7) * 12 (13 + 2*19 +12) Required area = area of square shown in figure – 4*areas of
1/4th parts of each circle, so
10) Answer - b) 48 Required area = 28*28 – 4*(1/4)*(22/7)*14*14
Explanation:
Let radius of rod = r, then height = 8r 18) Answer – a) (400√3) /9
Radius of 1 spherical ball = r/2 Explanation :
So number of balls = Vol. of cylindrical rod/Vol. of 1 spherical Diagonal of a square = a√2 = 10√2
ball so a = 10, perimeter of square = 4*10 = 40 = 3x (x is the
= ᴨ × r2 × 8r / (4/3) × ᴨ × (r/2)3 length of each side of triangle)
x = 40/3, so are of equilateral triangle = √3/4*40/3*40/3 =
11) Answer - e) 250 (400√3)/9 cm^2
Explanation:
Distance to be covered in 1 min = 66*(1000/60) = 1100 m 19) Answer – c) 13,11
70 cm = 0.70 m Explanation :
Circumference of wheel = 2*(22/7)*0.70 = 4.4 m Length = l and breadth = b,
Number of revolutions = (1100/4.4) = 250 (l +10)*(b-4) = lb and (l-5)*(b+7) = lb
Solve both equation to get l and b
12) Answer - b) 20 m
Explanation: 20) Answer – b) 16.67
3x, 2x Explanation :
So (3x+5)*2x = 2600 let length = 100 and breadth = 100
6x2 + 10x = 2600 now new length = 120 and let breadth = b
3x2 + 5x – 1300 = 0 so, 100*100 = 120*b
3x2 – 60x + 65x – 1300 = 0 b = 250/3, so % decrease = 100 – 250/3 = 50/3 = 16.67%
3x(x-20) + 65(x-20) = 0
Solve, x = 20 21) Answer – b) 2376
So breadth = 20 m Explanation :
total surface area = curved surface area of cone + curved
13) Answer - b) 125% surface area of cylinder + base area
Explanation: = (22/7)*12*13 + (22/7)*12*19 + (22/7)*12*12 = 2376 cm^2
Surface area of cube = 6a2
A increases by 50% 22) Answer – a) 3080
So area increases by 50 + 50 + (50)(50)/100 = 125% Explanation :
r + h = 19 m
14) Answer - a) 3080 cm2 2πr(r + h) = 1672
Explanation: r = 1672 * 7/ 2 * 22 * 19 = 14
(22/7)*(2x)2*3x = 12936 r = 14 ; h = 5
Solve, x = 7 volume of the cylinder = πr²h = (22/7) * 14 * 14 * 5 = 3080 m³
So radius = 7 cm, height = 21 cm
Total surface area = 2*(22/7)*14*(21+14) = 3080 23) Answer – b) 1 : 4
Explanation :
15) Answer - c) 120 b = 2a
Explanation: a = b/2
20 cm = 0.2 m Area of square = b²/4 = Area of rectangle
So number of boxes that can fit = 1.6 × 1 × 0.6/0.2 × 0.2 × 0.2 l * b = b²/4 => l = b/4
l / b =(b/4)/b => 1:4
16) Answer - e) 9 times
Explanation: 24) Answer – b)314.2 Sq cm
Wire will be taken as cylinder. Explanation :
R = r/3 Base of the cone(r) =10/2 = 5 cm
ᴨr2h = ᴨR2H Height of the cone (h)= √169-25 =√144 = 12
r2h = (r/3)2H Volume = 1/3πr2h
so H = 9h = 22*5*5*12/7*3 = 314.2 Sq cm

For details on our programs: - Contact: +91 9930202329, +91, 8291924300 or visit www.campuscredentials.com (108)
25) Answer – d)1/4 cm
Explanation : 4) Answer – B. 510 × 45
Total volume displaced by 8 men= 120×5 cm cube Explanation :
However volume=lxbxh=80x30xh 10 questions with 5 choices = 510
80x30xh=120×5 20 questions with 4 choices = 45
h=120*5/80*30 = 600/2400 = 0.25.
so the water level rises by 0.25 cm = ¼ cm 5) Answer – C. 2419200
Explanation :
26) Answer – c) 1:4 5
P 3 *8 P 3* 6 P 3 = 60*336*120 = 2419200
Explanation :
v1/v2 = (r1²) * h1 / (r2²) * h2 (h1=h2) 6) Answer – B) 10800
(r)²/(2r)² Explanation :
v1/v2 = 1:4 5 men can sit in 5! Ways and adjacent to men total 6 vacant
places are there, at there we can arrange 3 women in 6P3 ways.
27) Answer - b)98m 5!* 6P3= 10800
Explanation :
100*2 = 200cm measured long 7) Answer – D.72
Correct length = 100 – (200/100) = 100 -2 = 98m Explanation :
3!*3! + 3!*3! = 36+36 = 72
28) Answer - c)45 rounds
Explanation :N = 2000/(14*22/7) = 2000/44 = 45.45 = 45 8) Answer – A.1024
rounds Explanation :
No of way =45 = 1024
29) Answer – b)26m*3m
Explanation : 9) Answer - B) 720
L =20 – 2b Explanation:
area of the garden = 78 sq m, AUTHOR contains 6 letters, so total 6! ways.
L * b =78
b*(20-2b) = 78 10) Answer - C) 2401
20b – 2b2 = 78 Explanation:
b2 -10b+ 39 =0 LEADING is 7 letters.
b= 3 b=13 We have 4 places where letters are to be placed.
b = 3 then L = 26 For first letter there are 7 choices, since repetition is allowed,
b=13 then L = 6 for second, third and fourth letter also we have 7 choices each,
Dimension = 26m * 3 m so total of 7*7*7*7 ways = 2401 ways.

30) Answer – d)1192 sq cm 11) Answer - B) 2880


Explanation : Explanation:
Length of one side of rhombus= 300/4=75cm First make IIIE in a circle. So we have
Other diagonal= 2 * root of (75)^2 – 8^2
= 149
Area= ½ * 16 * 149

Now we have N, V, S, B, L and box, their arrangements can be


Chapter 12 - Permutation and Combination done in 6!
Practice Set 1 Answer: Letters inside circle are also to be arranged, we have I, I, I, E
1) Answer – D.3600 so ways are 4!/3!
Explanation : Total ways 6! * 4!/3!
Number of consonants = 5
Number of Vowels = 3 12) Answer - B) 210
We can arrange the vowels by 5! ways. In between these Explanation:
consonants 6 spaces are there we have to arrange 3 vowels at 6
We have to select 2 men from 4 men, and 4 women from 7
places. We can do that by 6P3 ways. women
5! × 6𝑃3
Total number of ways = = 3600 So total ways = 4C2*7C4
2!×2!

2) Answer – A.1320 13) Answer - C) 1722


Explanation : Explanation:
12*11*10 = 1320 Case 1: 3 men and 2 women
8
C3*7C2 = 1176
3) Answer – C.256 Case 2: 4 men and 1 women
8
Explanation : C4*7C1 = 490
4*4*4*4 = 256 Case 3: all 5 men

For details on our programs: - Contact: +91 9930202329, +91, 8291924300 or visit www.campuscredentials.com (109)
8
C5 = 56 digit (0-9 except 2)
Add all the cases. So numbers = 1*9*9 = 81
Case 2: 2 is placed at second place
14) Answer - D) 220 8 choices for the first place (1-9 except 2), 1 choice for the 2nd
Explanation: digit and 9 choices for the 3rd digit (0-9 except 2)
There are total 13 people, a particular woman is to be included, So numbers = 8*1*9 = 72
so now 12 people are left to chosen from and 3 members to be Case 3: 2 is placed at third place
12
chosen. So ways are C3. 8 choices for the first place (1-9 except 2), 9 choices for the 2nd
digit (0-9 except 2) and 1 choice for the 3rd digit
15) Answer - B) 20 So numbers = 8*9*1 = 72
Explanation: So total numbers = 81+72+72 = 225
Total 8 people, 2 men are to excluded, so 6 men left to be
chosen from and 3 members to be chosen. So ways are 6C3. 23) Answer – b) 4! 5!
Explanation :
16) Answer – C.465 First 5 African are seated along the circular table in (5-1)!
Explanation : Ways = 4!. Now Indian can be seated in 5! Ways, so 4! 5!
2 4-1 = 16 -1 = 15
2 5-1 = 32 -1 = 31 24) Answer – c) 91
15*31 = 465 Explanation :
From 15 points number of lines formed = 15C2
17) Answer – D.360 6 points are collinear, number of lines formed by these = 6C2
Explanation : So total lines = 15C2 – 6C2 + 1 = 91
No of way in Necklace = (n-1)!/2 = 6!/2
= 720/2 = 360 25) Answer – b) 16
Explanation :
18) Answer – A. 8467200 N
C2 = 120 (N is the number of persons)
Explanation :
No of ways = 7!*8P4 Chapter 13 - Probability
7! = 5040 Practice Set 1 Answer:
8P4 = 8*7*6*5 = 1680 1) Answer -A) 1/2
No of ways = 5040*1680 = 8467200 Explanation :
Total no of balls = 6+ 4 = 10
19) Answer – D. 11!/2! 3 balls drawn = 10C3 =(10×9×8)/(3×2×1) =120
Explanation : one ball is red and other 2 are white = 4C1×6C2
No of ways = 11!/2! 4
C1×6C2 =(4×6×5)/(1×2)= 60
P = 60/120 = ½
20) Answer - A) 60
Explanation: 2) Answer -B) 1/5
Take vowels in a box together as one – IIU, M, N, M, M Explanation :
So there are 5 that to be placed for this 5!, now 3 Ms, so 5!/3!, 5 digit number = 5! = 120
so arrangement of vowels inside box gives 3!/2! Divisible by 5 then the last digit should be 0
So total = 5!/3! * 3!/2! Then the remaining position have the possibility = 4! =24
P = (4!/5!) = 24/120 = 1/5
21) Answer - B) 225
Explanation: 3) Answer -B) 11/36
A number is divisible by 4 when its last two digits are divisible Explanation:
by 4 Number of favourable outcomes = (1,4), (2,4), (3,4), (4,4),
For this the numbers should have their last two digits as 00, 04, (5,4), (6,4), (4,1), (4,2), (4,3), (4,5), (4,6). = 11
08, 12, 16, … 96 Number of all possible outcomes = 36
By the formula, an = a + (n-1)d Probability = 11/36
96 = 0 + (n-1)*4
n = 25 4) Answer - C)11/21
so there are 25 choices for last 2 digits and 9 choices (1-9) for Explanation :
the 1st digit P(At least one woman is selected) = 1 - P(No woman is
so total 9*25 selected)
= 1 - 5C2/7C2
22) Answer - A) 225 = 11/21
Explanation:
0 cannot be placed at first digit to make it a 3 digit number. 5) Answer -B) 15/52
3 cases: Explanation :
Case 1: 2 is placed at first place n(S) = 16C7 = 11440
1 choice for the first place, 9 choices each for the 2nd and 3rd

For details on our programs: - Contact: +91 9930202329, +91, 8291924300 or visit www.campuscredentials.com (110)
n(E) = 11C4 ×5C3 = 330×10 = 3300 15) Answer – B) 13/18
P = n(E)/ n(S) = 3300/11440 = 15/52 Solution :
1 – 1/2*2/3*5/6 = 13/18
6) Answer -D) 4/11
Explanation : 16) Answer – B) 120/343
Total no of balls =8+6+8 = 22 Solution :
P(B) =8/22 Total number of outcomes = 7*7*7*7
P(neither red nor green) = 8/22 = 4/11 Favourable outcome = 7*6*5*4
P = (7*6*5*4)/7*7*7*7 = 120/343
7) Answer - C)7/12
Explanation : 17) Answer - C) 4/52*4/51
P Q Solution :4/52*4/51
1 123456
2 23456 18) Answer - B) 2/5
3 3456 Explanation:
4 456 Prob. of math = 30/100 = 3/10, Prob. of computers = 20/100 =
5 56 1/5, prob. for both = 10/100 = 1/10
6 6 So required prob. = 3/5 + 1/5 – 1/10
n(E) = 21
n(S) = 36 19) Answer - A) 55/221
P = 21/36 = 7/12 Explanation:
Prob. of both red = 26C2/52C2
8) Answer - C)7/10 Prob. of both kings = 4C2/52C2
Explanation : Since there are also cads which are both red and king, so we
P(a) = 10/100 = 1/10 will subtract there prob. There are 2 red cards which are kings
P(b) = 70/100 = 7/10 Prob. of both red and king = 2C2/52C2
P(both) = 10/100 = 1/10 So required prob. = 26C2/52C2 + 4C2/52C2 – 2C2/52C2 = 55/221
P(a U b) = (1+7-1)/10 = 7/10
20) Answer - C) 49/225
9) Answer - B)1/5525 Explanation:
Explanation : 7/15 * 7/15
n(S) = 52C3 = 132600/6 = 22100
n(E) = 4C3 = 24/6 = 4 1) Answer - A) 24
p = 4/22100 = 1/5525 Explanation :
We know,
10) Answer - A)1/3 P(getting red ball) = 1/4
Explanation : P(getting blue ball) = 1/3
6
C2 = 6*5/2*1 = 30/2 = 15 Let the no. of balls = x
10
C2 = 10*9/1*2 = 45 Therefore, P(getting orange ball) = 10/x
P =15/45 = 1/3 We know,
P(getting red ball) + P(getting blue ball) + P(getting orange
11) Answer - A)6/7 ball) = 1
Explanation : ⇒1/4 + 1/3 + 10/x = 1
7
C2 = 7*6/2 = 21 ⇒10/x = 1 -1/4 -1/3
4
C1*3C1 + 4C2 = 4*3 + (4*3)/2 = 12+6 = 18 ⇒10/x = 1 - 7/12
P = 18/21 = 6/7 ⇒10/x = 5/12
⇒10 * 12 = 5 * x
12) Answer - C)505/1001 ⇒x = 24
Explanation : Therefore, the no. of balls = x = 24
14
C4 = 14*13*12*11/4*3*2*1 = 1001
4 No positive+4 no negative + (2 no positive * 2 no negative) 22) Answer - C) 63/253
= 6C4 + 8C4 +(6C2 ×8C2) = 15+70+15*28 = 505 Explanation:
P = 505/1001 From 1 to 24, there are 8 numbers which are multiple of 3
Case 1: 2 are multiple of 3, and one any other number from
13) Answer - C)1/12 (24-8) = 16 tickets
Explanation :1/2 * 1/6 = 1/12 8
C2*16C1 / 24C3 = 56/253
Case 2: all are multiples of 3.
14) Answer – B) 8/15 8
C3 / 24C3 = 7/253
Solution : Add both cases.
PF + FP (P = pass, F = fail)
1/3*2/5 + 2/3*3/5 = 8/15

For details on our programs: - Contact: +91 9930202329, +91, 8291924300 or visit www.campuscredentials.com (111)
23) Answer - B)1/680
Explanation :
17
C3 = 17*16*15/3*2*1 = 4080/6 = 680
3
C3 = 1
P = 1/680

24) Answer - A)33/442


Explanation :
17
C5 = 17*16*15*14*13/5*4*3*2*1 = 742560/120 = 6188
11
C5 = 11*10*9*8*7/5*4*3*2*1 = 55440/120 = 462
P = 462/6188 = 33/442

25) Answer - D)3/85


Explanation :
17
C3 = 17*16*15/3*2*1 = 4080/6 = 680
4
C1 × 4C2 = 4*4*3/2*1 = 24
P = 24/680 = 3/85

For details on our programs: - Contact: +91 9930202329, +91, 8291924300 or visit www.campuscredentials.com (112)

You might also like